Mental Health Exam

Ace your homework & exams now with Quizwiz!

A cognitive therapist would help a client restructure the thought "I am stupid!" to which statement?

"What I did was stupid"

The case manager is demonstrating an understanding of the primary goals of managed care when engaging in which client intervention? A) Arranging for the client to have a screening for prostate cancer B) Notifying the family that the client will require a wheelchair when discharged C) Providing the client with organizations that help defray the cost of prescribed drug D) Arranging for respite care when the client's family needs to attend an out-of-state affair

A The goal of managed care is to provide coordination of all health services with an emphasis on preventive care. While appropriate interventions, none of the remaining options focus on preventive care.

Which statement regarding clients' rights after being voluntarily admitted to a behavioral health unit is true? A) All rights remain intact. B) Only rights that do not involve decision making remain intact. C) The right to refuse treatment is no longer guaranteed. D) All rights are temporarily suspended.

A The hospitalized client is not a convicted criminal thus all civil rights remain intact.

How does Harry Stack Sullivan Interpersonal Theory view anxiety?

A painful emotion arising from social insecurity

When considering the ongoing, crucial responsibilities of nurses working on an inpatient psychiatric unit, which activity has highest priority? A) Fostering research B) Maintaining a therapeutic milieu C) Providing sympathetic listening D) Providing constructive negative feedback

B Nursing is the discipline primarily responsible for maintenance of a therapeutic milieu, an environment that serves as a real-life training ground for learning about self and practicing communication and coping skills in preparation for a return to the community outside the hospital. While the remaining options are nursing responsibilities, none has the priority of maintaining a therapeutic milieu.

Which imaging technique can provide information about brain function?

PET Scan

Which assessment should the nurse perform to evaluate the pharmacokinetic affect of a MAOI's antidepressant medication?

The results of the liver function test

Avoidance

changing routines to escape similar situations to trauma. avoids places, events or objects that remind them of trauma

10. Chronic obstructive pulmonary disease, spinal injury, seizure disorder, and pregnancy are conditions that: a. Frequently result in out of control behavior. b. Respond well to therapeutic holding. c. Necessitate the use of only two-point restraint. d. Contraindicate restraint and seclusion.

d. Contraindicate restraint and seclusion.

psychological effects of sexual assault

depression suicides anxiety fear difficult daily functioning low self esteem sexual dysfunction somatic complaints

effects of incest

negative self image eating disorder depression personality disorder self destructive behavior substance abuse

What term is used to identify the structures that respond to stimuli, conduct electrical impulses, and release neurotransmitters?

neurons

Modeling

perhaps one of the most potent means of value education because it presents a vivid example of values in action. We all need role models to guide us in negotiating life's many choices. Young people in particular are hungry for role models and will find them among peers as well as adults. As nurses, parents, bosses, coworkers, friends, lovers, teachers, spouses, and singles, we are constantly (in either a positive or a negative manner) being a role model to others.

PTSD

result if ASD symptoms extend beyond 1 year

Process recordings

written records of a segment of the nurse-patient session that reflect as closely as possible the verbal and nonverbal behaviors of both patient and nurse

An adult seeks counseling after the spouse was murdered. The adult angrily says, "I hate the beast that did this. It has ruined my life. During the trial, I don't know what I'll do if the jury doesn't return a guilty verdict." What is the nurse's highest priority response a."Would you like to talk to a psychiatrist about some medication to help you cope during the trial" b."What resources do you need to help you cope with this situation" c."Do you have enough support from your family and friends" d."Are you having thoughts of hurting yourself or others"

"Are you having thoughts of hurting yourself or others"

A victim of a violent rape was treated in the emergency department. As discharge preparation begins, the victim says softly, "I will never be the same again. I can't face my friends. There is no reason to go on." Select the nurse's most appropriate response.

"Are you thinking of harming yourself?" The patient's words suggest hopelessness. Whenever hopelessness is present, so is suicide risk. The nurse should directly address the possibility of suicidal ideation with the patient. The other options attempt to offer reassurance before making an assessment.

A nurse, active in local consumer mental health groups and in local and state mental health associations, keeps aware of state and national legislation affecting mental illness treatment. How can this nurse positively affect the climate for effective, mental health treatment?

"By educating the public on the effects that stigmatizing has on mental health clients"

A nurse in the emergency department assesses an unresponsive victim of rape. The victim's friend reports, "That guy gave her salty water before he raped her." Which question is most important for the nurse to ask of the victim's friend?

"Has the victim consumed any alcohol?" Salty water is a slang/street name for GHB (γ-hydroxy-butyric acid), a Schedule III central nervous system depressant associated with rape. Use of alcohol would produce an increased risk for respiratory depression. GHB has a duration of 1-12 hours, but the duration is less important that the potential for respiratory depression. Seeking evidence is less important than the victim's physiologic stability.

The nurse is conducting an educational seminar for health care providers on mental health coverage and reimbursement for clients. Which statement is most important to include in the teaching? "Health insurance is employer based, and serious mental illness can lead to job loss." "Health insurance readily pays for physical problems, so always use a physical diagnosis as the primary problem." "Medicare and Medicaid do not cover mental illnesses." "Private insurance always covers mental illnesses."

"Health insurance is employer based, and serious mental illness can lead to job loss." Most health insurance is supplied through employers. Clients with mental disorders often have difficultly holding down employment due to their symptoms. Health care providers must be aware that clients with mental illness are at risk for losing their jobs and health insurance. The client's diagnosis should be consistent with his or her presentation and may not always be physical. Other options such as Medicare and Medicaid may be available in these situations for qualifying clients. Private insurance does not always cover mental illnesses. p. 58

A patient comes to the crisis clinic after an unexpected job termination. The patient paces around the room sobbing, cringes when approached, and responds to questions with only shrugs or monosyllables. Choose the nurse's best initial comment to this patient. a."Everything is going to be all right. You are here at the clinic, and the staff will keep you safe." b."I see you are feeling upset. I'm going to stay and talk with you to help you feel better." c."You need to try to stop crying and pacing so we can talk about your problems." d."Let's set some guidelines and goals for your visit here."

"I see you are feeling upset. I'm going to stay and talk with you to help you feel better."

Consider both Sullivan's term security operations and Freud's term defense mechanisms. Which statement suggests that the client's specialized treatment goal has been successfully met?

"I'm experiencing much less anxiety about school now."

During the initial interview at the crisis center, a patient says, "I've been served with divorce papers. I'm so upset and anxious that I can't think clearly." Which comment should the nurse use to assess personal coping skills a."In the past, how have you handled difficult or stressful situations" b."What would you like us to do to help you feel more relaxed" c."Tell me more about how it feels to be anxious and upset." d."Can you describe your role in the marital relationship"

"In the past, how have you handled difficult or stressful situations"

Resilience is characteristics of mental health that allows people to adapt to tragedies, trauma, and loss. Which client behavior demonstrates this characteristic?

"Losing my job was hard but my skills will help me get another one"

A nurse interviews a 17-year-old male victim of sexual assault. The victim is reluctant to talk about the experience. Which comment should the nurse offer to this victim?

"Male victims of sexual assault often experience physical injuries and are assaulted by more than one person." Few rape survivors seek help, even with serious injury; so, it is important for the nurse to help the victim discuss the experience. The correct response therapeutically gives information to this victim. A male rape victim is more likely to experience physical trauma and to have been victimized by several assailants. Males experience the same devastation, physical injury, and emotional consequences as females. Although they may cover their responses, they too benefit from care and treatment. "Why" questions represent probing, which is a non-therapeutic communication technique. The victim may or may not have friends who have had this experience, but it's important to talk about his feelings rather than theirs

A 38-year-old patient diagnosed with major depression states, "my provider said something about the medicine I've been prescribed will affect my neurotransmitters. What exactly are neurotransmitters?" What is the nurse's best response to the patient's question?

"Neurotransmitters are chemical messengers in the brain that help regulate specific functions such as depression."

The student nurse is describing characteristics of furniture on the psychiatric unit. Which description is most accurate? "The furniture for inpatient rooms tends to be lightweight, so if it is thrown it will not cause too much injury." "The furniture for inpatient rooms has rounded corners so that head injuries do not occur." "The furniture for inpatient rooms is heavy and durable." "There is no furniture in the inpatient rooms to prevent injuries."

"The furniture for inpatient rooms is heavy and durable." Furniture for inpatient rooms tends to be heavy and durable so that it cannot be thrown or dismantled and used for weapons. Lightweight furniture is not used because it is easier to lift and throw, and it is easy to disassemble and used as weaponry. Furniture edges should be rounded or padded for protection of small children. Seclusion rooms contain no furniture and have padded walls to prevent injury. Inpatient rooms have furniture. p. 56

A nurse's identification badge includes the term, "Psychiatric Mental Health Nurse". A client with a history of paranoia asks, "What does that title mean?" Keeping in mind the diagnosis of the patient, how should the nurse respond to this question?

"We have the specialized skills needed to care for those with mental illnesses."

A patient being seen in the clinic for superficial cuts on both wrists is pacing and sobbing. After a few minutes, the patient is calmer. The nurse attempts to determine the patient's perception of the precipitating event by asking: a."Tell me why you were crying." b."How did your wrists get injured" c."How can I help you feel more comfortable" d."What was happening just before you started to feel this way"

"What was happening just before you started to feel this way"

A nurse assesses a patient in crisis. Select the most appropriate question for the nurse to ask to assess this patient's situational support. a."Has anything upsetting occurred in the past few days" b."Who can be helpful to you during this time" c."How does this problem affect your life" d."What led you to seek help at this time"

"Who can be helpful to you during this time"

Which comment by the nurse would best support relationship building with a survivor of intimate partner abuse?

"You are feeling violated because you thought you could trust your partner." The correct option uses the therapeutic technique of reflection. It shows empathy, an important nursing attribute for establishing rapport and building a relationship.

A rape victim tells the nurse, "I should not have been out on the street alone." Select the nurse's most therapeutic response.

"You feel as though this would not have happened if you had not been alone." A reflective communication technique is most helpful. Looking at one's role in the event serves to explain events that the victim would otherwise find incomprehensible. The distracters discount the victim's perceived role and interfere with further discussion.

A new nurse has accepted a position as staff nurse on a psychiatric unit. Which statement made by the new nurse requires additional instructions regarding the therapies provided on the unit?

"You will attend a psychotherapy group that I lead that will help you care for yourself"

A team of nurses report to the community after a category 5 hurricane devastates many homes and businesses. The nurses provide emergency supplies of insulin to persons with diabetes and help transfer patients in skilled nursing facilities to sites that have electrical power. Which aspects of disaster management have these nurses fulfilled Select all that apply. a.Preparedness b.Mitigation c.Response d.Recovery e.Evaluation

-.Mitigation -Response

A patient was abducted and raped at gunpoint by an unknown assailant. Which nursing interventions are appropriate while caring for the patient in the emergency department?

-Allow the patient to talk at a comfortable pace. -Place the patient in a private room with a caregiver. -Pose questions in nonjudgmental, empathetic ways. Neutral, nonjudgmental care and emotional support are critical to crisis management for the rape victim. The rape victim should have privacy but not be left alone. The rape victim's anxiety may escalate when touched by a stranger, even when the stranger is a nurse. Some rape victims prefer not to have family involved. The patient's privacy may be compromised by family presence.

Which aspects of assessment have priority when a nurse interviews a rape victim in an acute setting?

-Coping mechanisms the patient is using -Signs and symptoms of emotional and physical trauma -Adequacy and availability of the patient's support system

The nurse is assessing a client for eligibility to receive psychiatric home care. What criteria must the client meet to receive reimbursement for psychiatric home care? Select all that apply. Current psychiatric diagnosis Age 65 years or older Wheelchair-bound status Under the care of a primary care physician (PCP) Homebound status Eligible for Medicare

-Current psychiatric diagnosis -Under the care of a primary care physician (PCP) -Homebound status Criteria for reimbursement of psychiatric home care include a psychiatric diagnosis, being under the care of a PCP, and homebound status. Homebound means the client cannot safely leave home, leaving home causes undue stress, or the nature of the illness results in refusal to leave home. It is not a stipulation for clients to be 65 years or older, wheelchair bound, or eligible for Medicare. Test-Taking Tip: Be alert for details about what you are being asked to do. In this question type, you are asked to select all options that apply to a given situation or patient. All options likely relate to the situation, but only some of the options may relate directly to the situation. p. 54

A nurse driving home after work comes upon a serious automobile accident. The driver gets out of the car with no apparent physical injuries. Which assessment findings would the nurse expect from the driver immediately after this event Select all that apply. a.Difficulty using a cell phone b.Long-term memory losses c.Fecal incontinence d.Rapid speech e.Trembling

-Difficulty using a cell phone -Rapid speech -Trembling

Based on the current understanding of brain physiology, which neurotransmitter would be the expected target of medication prescribed to mange depression?

-Dopamine -Serotonin -Norepinephrine

According to DSM-V, which statement made by an adult client supports the criteria for generalized anxiety disorder?

-I've been really anxious for at least 2 years now -My marriage is in trouble because I'm always so irritable -I've had a good physical and my health care providers says I'm in good health

The nurse caring for a client prescribed an antidepressant medication that produces anticholinergic side effects should assess for which possible side effects?

-blurred vision -dry mouth -constipation

The nursing caring for a client risperidone observes the client carefully for which possible side effects?

-daytime sleepiness -sexual dysfunction -a weight gain

A rape victim tells the emergency nurse, "I feel so dirty. Help me take a shower before I get examined." The nurse should:

-explain that bathing destroys evidence. -offer the victim a shower after evidence is collected.

Which organ secretes hormones that are a normal component of the body's general response to stress?

-hypothalamus -pituitary gland -adrenal glands

A rape victim visited a rape crisis counselor weekly for 8 weeks. At the end of this counseling period, which comment by the victim best demonstrates that reorganization was successful?

. "I'm sleeping better although I still have an occasional nightmare." Rape-trauma syndrome is a variant of posttraumatic stress disorder. The absence of signs and symptoms of posttraumatic stress disorder suggest that the long-term reorganization phase was successfully completed. The victim's sleep has stabilized; occasional nightmares occur, even in reorganization. The distracters suggest somatic symptoms, appetite disturbances, and self-blame, all of which are indicators that the process is ongoing.

An adult has cared for a debilitated parent for 10 years. The parent's condition recently declined, and the health care provider recommended placement in a skilled nursing facility. The adult says, "I've always been able to care for my parents. Nursing home placement goes against everything I believe." Successful resolution of this person's crisis will most closely relate to: a.resolving the feelings associated with the threat to the person's self-concept. b.ability of the person to identify situational supports in the community. c.reliance on assistance from role models within the person's culture. d.mobilization of automatic relief behaviors by the person.

.resolving the feelings associated with the threat to the person's self-concept.

5 steps to sexual assault examination

1-head to toe 2-detailed genital examination 3-evidence collection 4-documentation of physical findings 5-TREATMENT prophylactic STD treatment psychotherapy crisis counseling discharge planning follow up care

The nurse working at a Veteran's Administration (VA) clinic assesses a client with posttraumatic stress disorder (PTSD). What is the approximate prevalence of PTSD in veterans of the Iraq and Afghanistan wars? 3% 7% 14% 24%

14% For veterans of the Iraq and Afghanistan wars, PTSD prevalence is about 14%. More veterans than 3% experienced PTSD. The prevalence of PTSD in the general population is approximately 7%. Fewer veterans than 24% experienced PTSD. Test-Taking Tip: The most reliable way to ensure that you select the correct response to a multiple choice question is to recall it. Depend on your learning and memory to furnish the answer to the question. To do this, read the stem, and then stop! Do not look at the response options yet. Try to recall what you know and, based on this, what you would give as the answer. After you have taken a few seconds to do this, then look at all of the choices and select the one that most nearly matches the answer you recalled. It is important that you consider all the choices and not just choose the first option that seems to fit the answer you recall. Remember the distractors. The second choice may look okay, but the fourth choice may be worded in a way that makes it a slightly better choice. If you do not weigh all the choices, you are not maximizing your chances of correctly answering each question. p. 57

A patient who is visiting the crisis clinic for the first time asks, "How long will I be coming here" The nurse's reply should consider that the usual duration of crisis intervention is: a.1 to 2 weeks. b.3 to 4 weeks. c.4 to 8 weeks. d.8 to 12 weeks.

4 to 8 weeks.

A 16-year-old wants to drive, but the parents will not allow it. A 14-year-old sibling was invited to several sleepovers, but the parents found reasons to deny permission. Both teens are annoyed because the parents buy clothes for them that are more suitable for younger children. The parents say, "We don't want our kids to grow up too fast." Which term best describes this family's boundaries? a. Rigid b. Clear c. Enmeshed d. Differentiated

A

Which comment by a mother during a family therapy session shows evidence of scapegoating? a. "Our youngest child always starts arguments and upsets everyone else." b. "We all express our feelings openly except when we think it might upset my husband." c. "Our oldest child knows that my husband and I are doing all we can for the others." d. "After my husband has been drinking, I have to get everyone up and ready for school."

A

Most clients who are diagnosed with chronic mental illness are not likely to have their psychiatric mental health experiences covered by which payment method? A) Private insurance B) Medicare C) Medicaid D) Private pay

A Because most health insurance is employer based, few chronically ill clients have private insurance. The other options are examples of ways patients pay for their needed mental health services.

A therapeutic inpatient milieu will include which characteristic? A) It provides for the client's safety and comfort. B) Voluntarily admitted clients are generally allowed additional privileges. C) Rules and behavioral limits are flexibly enforced. D) Staff provide frequent and ongoing negative feedback to clients.

A Because the acuity level on inpatient units is high, nurses are responsible for ensuring that the environment is safe and that elopement and self-harm opportunities are minimized. The other choices are undesirable characteristics of a therapeutic milieu.

Which of the following structural safety precautions is most important when attempting to prevent a common type of inpatient suicide? A) Break-away closet bars to prevent hanging B) Bedroom and dining areas with locked windows to prevent jumping C) Double-locked doors to prevent escaping from the unit D) Platform beds to prevent crush injuries

A Hangings are the most common method of inpatient suicide. The other options are important safety measures but don't directly address the suicide method of hanging.

In addition to physicians, what other members of the mental health disciplines have been identified as having the knowledge, skills, ability, and legal authority to intervene in the full range of mental health care? A) Nurses B) Social workers C) Clinical psychologists D) Chemical dependency counselors

A Nurses are the only caregivers listed who can provide both physical and psychological care for mental health clients.

When considering mental illness, recovery is best described to a client by which statement? A) Working, living, and participating in the community B) Never having to visit a mental health provider again C) Being able to understand the nature of the diagnosed illness D) A period of time when signs and symptoms are being managed

A Recovery is described as the ability of the individual to work, live, and participate in the community. Never having to visit a mental health provider is unrealistic. While important to recovery understanding of the disorder is not a demonstration of recovery. Remission is a period of time when signs and symptoms are being managed.

Which scenario is an example of an adventitious crisis a.The death of a child from sudden infant death syndrome b.Being fired from a job because of company downsizing c.Retirement of a 55-year-old person d.A riot at a rock concert

A riot at a rock concert

Which situation describes consensual sex rather than rape?

A woman's lover pleads with her to have oral sex. She gives in but later regrets the decision. Only the key describes a scenario in which the sexual contact is consensual. Consensual sex is not considered rape if the participants are of legal age.

A parent was recently hospitalized with severe depression. Family members say, "We're falling apart. Nobody knows what to expect, who should make decisions, or how to keep the family together." Which interventions should the nurse use when working with this family? Select all that apply. a. Help the family set realistic expectations. b. Provide empathy, acceptance, and support. c. Empower the family by teaching problem solving. d. Negotiate role flexibility amongst family members. e. Focus planning on the family rather than on the patient.:

A, B, C, D

Which situations are most likely to place severe, disabling stress on a family? Select all that apply. a. A parent needs long-term care after sustaining a severe brain injury. b. The youngest child in a family leaves for college in another state. c. A spouse is diagnosed with liver failure and needs a transplant. d. Parents of three children, aged 9, 7, and 2 years, get a divorce. e. A parent retires after working at the same job for 28 years.

A, C, D

You are working on an adolescent psychiatric unit. Katy, aged 16 years, has been angry all day because her boyfriend was not allowed to visit last night. Katy is in the hallway and begins yelling, "It's not fair! You all hate me! I hate this place!" She begins pounding her fists on the wall. To deal with the situation and prevent further escalation, your best response would be to say: A. "Katy, I will help you calm down. Do you want to go to your room and talk or go to the quiet room?" B. "Katy, you may yell and bang your fists but you must do it in your own room so you don't upset the other patients." C. "Katy, stop that right now! You will not be allowed to behave like that!" D. "Katy, you will have to go into seclusion and restraints right now." a

A. "Katy, I will help you calm down. Do you want to go to your room and talk or go to the quiet room?" Approaching the patient in a calm manner and giving choices may de-escalate the situation and gives the patient some control. The patient would not be allowed to yell or possibly hurt herself alone in her room. Commands such as "stop that right now!" could further escalate the situation. Seclusion and restraint may be premature because the situation may be able to be resolved using least restrictive means.Cognitive Level: Analyze (Analysis)Nursing Process: ImplementationNCLEX: Safe and Effective Care EnvironmentText page: 520

A nurse is caring for a client who is screaming at staff members and other clients. Which of the following is a therapeutic response by the nurse to the client/ A. "Stop screaming, and walk with me outside" B. "Why are you so angry and screaming at everyone?" C. "You will not get your way by screaming" D. "What was going through your mind when you started screaming?"

A. "Stop screaming, and walk with me outside"

Andie is a patient anxiously waiting her turn to speak with you. As you are very busy, you ask Andie if she can wait a few minutes so that you can finish your task. Unfortunately the task takes longer than anticipated and you are delayed getting back to Andie. On seeing you approach her, Andie accuses you of lying and refuses to speak with you. Which response is most likely to be therapeutic at this time? A. "you are angry that I didn't speak with you when I promised I would" B. "I'm sorry for being late, but screaming at me is not the best way to handle it." C. "You are too angry to talk right now. I'll come back in 20 minutes and we can try again." D. "Why are you angry? I told you I was busy and would get to you as soon as I could."

A. "you are angry that I didn't speak with you when I promised I would"

Which assessment finding is the best predictor of violence in a newly admitted client? A. A recent assault on a drinking companion B. A family history of bipolar disorder C. The nurse's subjective feeling that the client is uncooperative. D. A childhood history of being bullied at school

A. A recent assault on a drinking companion. The best predictor of violence is past episodes of violent behavior.REF: 517

The nurse notes that a patient's medication history includes clopidogrel. The nurse recognizes that this medication belongs to which drug class? A. Antiplatelet B. Opioid analgesic C. Vitamin K antagonist D. Nonsteroidal antiinflammatory drug (NSAID)

A. Antiplatelet Clopidogrel (Plavix) is an antiplatelet medication, used commonly in patients with cardiovascular disease. Plavix is not an opioid analgesic, NSAID, or vitamin K antagonist. p. 715

Which intervention strategy should be avoided by staff working with a client who is shouting and flailing his arms? A. Defusing the situation by laughing or making a joke of the challenge B. Saying "Let's go to your room to talk about this" C. Moving a few staff close together as a group to provide a show of force D. Allowing one staff person to speak to the client while others provide support

A. Defusing the situation by laughing or making a joke of the challenge Ridiculing a client should always be avoided. The other options are constructive approaches to deescalation.REF: Page 520

You respond to a loud, angry voice coming from the day room, where you find Alex is pacing and shouting that he isn't "going to take his (expletive) anymore." which of the following responses is likely to be helpful in de-escalating Alex? Select all that apply A. Remain calm, quiet, and in control B. Tell Alex that his actions are unacceptable and that he must go to his room C. Match Alex's volume so that he is able to hear over his own shouting D. Ask Alex if he can tell you what is upsetting him so you may be able to help E. Sand close to Alex so you can intervene physically if needed to protect others F. Tell Alex that he could be placed in seclusion if he cannot control himself so that the patient is aware of the negative consequences

A. Remain calm, quiet, and in control D. Ask Alex if he can tell you what is upsetting him so you may be able to help

Which would be the most appropriate response by the nurse to help a client who is demonstrating escalating anger? A. Walk the client to his room and help him practice stress-reduction techniques, such as deep breathing or muscle relaxation B. Suggest that the client spend some time in the gym with a punching bag to relieve his stress C. Suggest that the client spend some time pacing rapidly in the hallway until he feels less stressed D. Sit with the client in the day room so that he can vent his anger and not isolate himself

A. Walk the client to his room and help him practice stress-reduction techniques, such as deep breathing or muscle relaxation In settings in which the staff can reasonably expect episodes of client anger and aggression, regular teaching and practice of verbal and nonverbal interventions are essential. The most appropriate response by the nurse would be to help the client to a quiet environment and teach or coach the client to use positive coping skills.REF: 520

The most restrictive method for dealing with an aggressive client who is out of control is A. seclusion. B. a show of force. C. verbal intervention. D. antipsychotic medication.

A. seclusion Seclusion is the most restrictive method listed, because it curtails the client's freedom of ambulation.REF: 521-522

The more a nurse's intervention is prompted by emotion A. the less likely it is to be therapeutic B. the less likely it is to be aggressive C. the more likely it is to be effective. D. the more likely it is to be empathetic.

A. the less likely it is to be therapeutic. One study reported in the text found that the nurse's response to anger from a client varied according to the interpretation given to the client's anger and to the nurse's self-appraised ability to manage the situation. Only when self-efficacy was perceived as adequate did the nurse move to help the client. When self-efficacy was not seen as adequate, nurses showed a decreased ability to process the client's message and a decreased ability to problem-solve.REF: 518-519

16. Which statement provides the best rationale for closely monitoring a severely depressed patient during antidepressant medication therapy? a. As depression lifts, physical energy becomes available to carry out suicide. b. Patients who previously had suicidal thoughts need to discuss their feelings. c. For most patients, antidepressant medication results in increased suicidal thinking. d. Suicide is an impulsive act. Antidepressant medication does not alter impulsivity.

ANS: A Antidepressant medication has the objective of relieving depression. Risk for suicide is greater as the depression lifts, primarily because the patient has more physical energy at a time when he or she may still have suicidal ideation. The other options have little to do with nursing interventions relating to antidepressant medication therapy. PTS: 1 DIF: Cognitive Level: Understand (Comprehension) REF: Page 486-487 TOP: Nursing Process: Planning MSC: Client Needs: Safe, Effective Care Environment

A patient diagnosed with major depression has lost 20 pounds in one month, has chronic low self-esteem, and a plan for suicide. The patient has taken an antidepressant medication for 1 week. Which nursing intervention has the highest priority? a. Implement suicide precautions. b. Offer high-calorie snacks and fluids frequently. c. Assist the patient to identify three personal strengths. d. Observe patient for therapeutic effects of antidepressant medication.

ANS: A Implementing suicide precautions is the only option related to patient safety. The other options, related to nutrition, self-esteem, and medication therapy, are important but are not priorities.

21. The feeling experienced by a patient that should be assessed by the nurse as most predictive of elevated suicide risk is a. hopelessness. b. sadness. c. elation. d. anger.

ANS: A Of the feelings listed, hopelessness is most closely associated with increased suicide risk. Depression, aggression, impulsivity, and shame are other feelings noted as risk factors for suicide. PTS: 1 DIF: Cognitive Level: Understand (Comprehension) REF: Page 487-490 (Table 25-3) and (Nursing Care Plan 25-1) TOP: Nursing Process: Assessment MSC: Client Needs: Psychosocial Integrity

18. A depressed patient says, Nothing matters anymore. What is the most appropriate response by the nurse? a. Are you having thoughts of suicide? b. I am not sure I understand what you are trying to say. c. Try to stay hopeful. Things have a way of working out. d. Tell me more about what interested you before you became depressed.

ANS: A The nurse must make overt what is covert; that is, the possibility of suicide must be openly addressed. The patient often feels relieved to be able to talk about suicidal ideation. PTS: 1 DIF: Cognitive Level: Apply (Application) REF: Page 486-487 | Page 488-490 (Nursing Care Plan 25-1) and (Table 25-3) TOP: Nursing Process: Implementation MSC: Client Needs: Safe, Effective Care Environment

9. A college student who attempted suicide by overdose was hospitalized. When the parents were contacted, they responded, We should have seen this coming. We did not do enough. The parents reaction reflects: a. guilt. b. denial. c. shame. d. rescue feelings.

ANS: A The parents statements indicate guilt. Guilt is evident from the parents self-chastisement. The feelings suggested in the distracters are not clearly described in the scenario. PTS: 1 DIF: Cognitive Level: Understand (Comprehension) REF: Page 493-494 TOP: Nursing Process: Assessment MSC: Client Needs: Psychosocial Integrity

11. It has been 5 days since a suicidal patient was hospitalized and prescribed an antidepressant medication. The patient is now more talkative and shows increased energy. Select the highest priority nursing intervention. a. Supervise the patient 24 hours a day. b. Begin discharge planning for the patient. c. Refer the patient to art and music therapists. d. Consider discontinuation of suicide precautions.

ANS: A The patient now has more energy and may have decided on suicide, especially given the prior suicide attempt history. The patient must be supervised 24 hours per day. The patient is still a suicide risk. PTS: 1 DIF: Cognitive Level: Apply (Application) REF: Page 486-487 TOP: Nursing Process: Assessment MSC: Client Needs: Safe, Effective Care Environment

When a nurse assesses an older adult patient, answers seem vague or unrelated to the questions. The patient also leans forward and frowns, listening intently to the nurse. An appropriate question for the nurse to ask would be: a. "Are you having difficulty hearing when I speak?" b. "How can I make this assessment interview easier for you?" c. "I notice you are frowning. Are you feeling annoyed with me?" d. "You're having trouble focusing on what I'm saying. What is distracting you?"

ANS: A The patient's behaviors may indicate difficulty hearing. Identifying any physical need the patient may have at the onset of the interview and making accommodations are important considerations. By asking if the patient is annoyed, the nurse is jumping to conclusions. Asking how to make the interview easier for the patient may not elicit a concrete answer. Asking about distractions is a way of asking about auditory hallucinations, which is not appropriate because the nurse has observed that the patient seems to be listening intently.

24. After one of their identical twin daughters commits suicide, the parents express concern that the other twin may also have suicidal tendencies. Which reply should the nurse provide? a. Genetics are associated with suicide risk. Monitoring and support are important. b. Apathy underlies suicide. Instilling motivation is the key to health maintenance. c. Your child is unlikely to act out suicide when identifying with a suicide victim. d. Fraternal twins are at higher risk for suicide than identical twins.

ANS: A Twin studies suggest the presence of genetic factors in suicide; however, separating genetic predisposition to suicide from predisposition to depression or alcoholism is difficult. Primary interventions can be helpful in promoting and maintaining health and possibly counteracting genetic load. The incorrect options are untrue statements or an oversimplification. PTS: 1 DIF: Cognitive Level: Apply (Application) REF: Page 483-484 TOP: Nursing Process: Implementation MSC: Client Needs: Psychosocial Integrity

A nurse and social worker co-lead a reminiscence group for six "elite-old" adults. Which activity is appropriate to include in the group? a. Singing a song from World War II b. Learning to send and receive email c. Discussing national leadership during the Vietnam War d. Identifying the most troubling story in today's newspaper

ANS: A "Elite-old" adults are persons over 94 years of age; they were young during World War II. Reminiscence groups share memories of the past. Sending and receiving email is not an aspect of reminiscence. The other incorrect options are less relevant to this age group.

A patient asks the nurse, "What advantage does a durable power of attorney for health care have over a living will?" The nurse should reply, "A durable power of attorney for health care: a. gives your agent the authority to make decisions about your care if you are unable to during any illness." b. can be given only to a relative, usually the next of kin, who has your best interests at heart." c. authorizes your physician to make decisions about your care that are in your best interest." d. can be used only if you have a terminal illness and become incapacitated."

ANS: A A durable power of attorney for health care is an instrument that appoints a person other than a health care provider to act as an individual's agent in the event that he or she is unable to make medical decisions. The patient does not have to be terminally ill or incompetent for the appointed person to act on his or her behalf.

A patient says, "Please don't share information about me with the other people." How should the nurse respond? a. "I will not share information with your family or friends without your permission, but I share information about you with other staff." b. "A therapeutic relationship is just between the nurse and the patient. It is up to you to tell others what you want them to know." c. "It depends on what you choose to tell me. I will be glad to disclose at the end of each session what I will report to others." d. "I cannot tell anyone about you. It will be as though I am talking about my own problems, and we can help each other by keeping it between us."

ANS: A A patient has the right to know with whom the nurse will share information and that confidentiality will be protected. Although the relationship is primarily between the nurse and patient, other staff needs to know pertinent data. The other incorrect responses promote incomplete disclosure on the part of the patient, require daily renegotiation of an issue that should be resolved as the nurse-patient contract is established, and suggest mutual problem solving. The relationship must be patient centered. See relationship to audience response question.

A 78-year-old nursing home resident diagnosed with hypertension and cardiac disease is usually alert and oriented. This morning, however, the resident says, "My family visited during the night. They stood by the bed and talked to me." In reality, the patient's family lives 200 miles away. The nurse should first suspect that the resident: a. may be experiencing side effects associated with medications. b. may be developing Alzheimer disease associated with advanced age. c. had a transient ischemic attack and developed sensory perceptual alterations. d. has previously unidentified alcohol abuse and is beginning alcohol withdrawal delirium.

ANS: A A resident taking medications is at high risk for becoming confused because of medication side effects, drug interactions, and delayed excretion. The nurse should report the event and continue to assess for cognitive impairment. Symptoms of dementia develop slowly but persist over time. Alcohol abuse and withdrawal are not the nurse's first suspicion in this scenario.

If an older adult patient must be physically restrained, who is responsible for the patient's safety? a. Nurse assigned to care for the patient b. Nursing assistant who applies the restraint c. Health care provider who ordered the application of restraint d. Family member who agrees to the application of the restraint

ANS: A Although restraint is ordered by a health care provider, it is carried out by a nursing staff member. The nurse caring for the patient is responsible for the safe application of restraining devices and for providing safe care while the patient is restrained. Nurses may delegate the application of restraining devices and the care of the patient in restraint but remain responsible for outcomes. Even when the family agrees to restraint, nurses are responsible for ensuring safe outcomes.

A patient in a support group says, "I'm tired of being sick. Everyone always helps me, but I will be glad when I can help someone else." This statement reflects: a. altruism. b. universality. c. cohesiveness. d. corrective recapitulation.

ANS: A Altruism refers to the experience of being helpful or useful to others, a condition that the patient anticipates will happen. The other options are also therapeutic factors identified by Yalom.

The highest priority for assessment by nurses caring for older adults who self-administer medications is: a. use of multiple drugs with anticholinergic effects. b. overuse of medications for erectile dysfunction. c. misuse of antihypertensive medications. d. trading medications with acquaintances.

ANS: A Anticholinergic effects are cumulative in older adults and often have adverse consequences related to accidents and injuries. The incorrect options may be relevant but are not of the highest priority.

A selective serotonin reuptake inhibitor (SSRI) is prescribed for an older adult patient diagnosed with major depressive disorder. Nursing assessment should include careful collection of information regarding: a. use of other prescribed medications and over-the-counter products. b. evidence of pseudoparkinsonism or tardive dyskinesia. c. history of psoriasis and any other skin disorders. d. history of diarrhea and electrolyte imbalances.

ANS: A Drug interactions, with both prescription and over-the-counter products, can be problematic for the geriatric patient taking an SSRI. Careful collection of information is important. The incorrect options do not pose problems with SSRIs.

Which statement shows a nurse has empathy for a patient who made a suicide attempt? a. "You must have been very upset when you tried to hurt yourself." b. "It makes me sad to see you going through such a difficult experience." c. "If you tell me what is troubling you, I can help you solve your problems." d. "Suicide is a drastic solution to a problem that may not be such a serious matter."

ANS: A Empathy permits the nurse to see an event from the patient's perspective, understand the patient's feelings, and communicate this to the patient. The incorrect responses are nurse- centered (focusing on the nurse's feelings rather than the patient's), belittling, and sympathetic.

A nurse explains to the family of a mentally ill patient how a nurse-patient relationship differs from social relationships. Which is the best explanation? a. "The focus is on the patient. Problems are discussed by the nurse and patient, but solutions are implemented by the patient." b. "The focus shifts from nurse to patient as the relationship develops. Advice is given by both, and solutions are implemented." c. "The focus of the relationship is socialization. Mutual needs are met, and feelings are shared openly." d. "The focus is creation of a partnership in which each member is concerned with growth and satisfaction of the other."

ANS: A Only the correct response describes elements of a therapeutic relationship. The remaining responses describe events that occur in social or intimate relationships.

A nurse leads a psychoeducational group for patients in the community diagnosed with schizophrenia. A realistic outcome for group members is that they will: a. discuss ways to manage their illness. b. develop a high level of trust and cohesiveness. c. understand unconscious motivation for behavior. d. demonstrate insight about development of their illness.

ANS: A Patients with schizophrenia almost universally have problems associated with everyday living in the community, so discussing ways to manage the illness would be an important aspect of psychoeducation. Discussing concerns about daily life would be a goal to which each could relate. Developing trust and cohesion is desirable but is not the priority outcome of a psychoeducational group. Understanding unconscious motivation would not be addressed. Insight would be difficult for a patient with residual schizophrenia because of the tendency toward concrete thinking.

A group begins the working phase. One member has a childhood history of neglect and ridicule by parents. Which comment would the group leader expect from this member? a. "My boss is always expecting more of me than the others, but talking to him would only make it worse." b. "I'm sorry for talking all the time, but there is so much going on in my life. I can't remember what I already said." c. "Thanks for the suggestions everyone. Maybe some of them will help. It won't hurt to give them a try." d. "This group is stupid. Nobody here can help anybody else because we are all so confused. It's a waste."

ANS: A People who frequently complain, yet reject help or suggestions when offered, tend to have histories of severe deprivation as children, often accompanied by neglect or abuse. The other comments reflect dynamics other than the help-rejecting complainer, such as the monopolizer who apologizes for talking too much, the person who is insightful and agrees to try a peer's suggestion, and the demoralizing member.

A health care provider writes these new prescriptions for a resident in a skilled care facility: "2 g sodium diet; restraint as needed; limit fluids to 2000 ml daily; 1 dose milk of magnesia 30 ml orally if no bowel movement occurs for 3 days." Which prescription should the nurse question? a. Restraint b. Fluid restriction c. Milk of magnesia d. Sodium restriction

ANS: A Restraints may be applied only on the written order of the health care provider that specifies the duration during which the restraints can be used. The Joint Commission guidelines and Omnibus Budget Reconciliation Act regulations also mandate a number of other conditions that must be considered and documented before restraints are used. The other orders may be appropriate for implementation.

A nurse asks the following questions while assessing an older adult. The nurse will add the Geriatric Depression Scale as part of the assessment if the patient answers "yes" to which question? a. "Would you say your mood is often sad?" b. "Are you having any trouble with your memory?" c. "Have you noticed an increase in your alcohol use?" d. "Do you often experience moderate-to-severe pain?"

ANS: A Sadness may be a symptom of depression. Sad moods occurring with regularity should signal the need to assess further for other symptoms of depression. The incorrect options do not focus on mood.

When admitting older adult patients, health care agencies receiving federal funds must provide written information about: a. advance health care directives. b. the financial status of the institution. c. how to sign out against medical advice. d. the institution's policy on the use of restraints.

ANS: A The Patient Self-Determination Act of 1990 requires that patients have the opportunity to prepare advance directives.

At what point in the nurse-patient relationship should a nurse plan to first address termination? a. During the orientation phase b. At the end of the working phase c. Near the beginning of the termination phase d. When the patient initially brings up the topic

ANS: A The patient has a right to know the conditions of the nurse-patient relationship. If the relationship is to be time-limited, the patient should be informed of the number of sessions. If it is open-ended, the termination date will not be known at the outset, and the patient should know that the issue will be negotiated at a later date. The nurse is responsible for bringing up the topic of termination early in the relationship, usually during the orientation phase.

A community mental health nurse plans an educational program for staff members at a home health agency that specializes in the care of older adults. A topic of high priority should be: a. identifying clinical depression in older adults. b. providing cost-effective foot care for older adults. c. identifying nutritional deficiencies in older adults. d. psychosocial stimulation for those who live alone.

ANS: A The topic of greatest immediacy is identification of clinical depression in older adults. Home health staff are often better versed in the physical aspects of care and less knowledgeable about mental health topics. Statistics show that older adult patients with mental health problems are less likely than young adults to be diagnosed accurately. This is especially true for those with depression and anxiety, both of which are likely to be misinterpreted as normal aging. Undiagnosed and untreated depression and anxiety result in unnecessary suffering. The other options are of lesser importance.

A nurse wants to perform a preliminary assessment for suicidal ideation in an older adult patient. Which question would obtain the desired data? a. "What thoughts do you have about a person's right to take his or her own life?" b. "If you felt suicidal, would you communicate your feelings to anyone?" c. "Do you have any risk factors that potentially contribute to suicide?" d. "Do you think you are vulnerable to developing a depressed mood?"

ANS: A This question is clear, direct, and respectful. It will produce information relative to the acceptability of suicide as an option to the patient. If the patient deems suicide unacceptable, then no further assessment is necessary. If the patient deems suicide as acceptable, then the nurse can continue to assess the patient's intent, plan, and means to carry out the plan, as well as the lethality of the chosen method. The incorrect options are less direct.

A patient says, "Please don't share information about me with the other people." How should the nurse respond? a. "I won't share information with others without your permission, but I will share information about you with other staff members." b. "A therapeutic relationship is just between the nurse and the patient. It's up to you to tell others what you want them to know." c. "It really depends on what you choose to tell me. I will be glad to disclose at the end of each session what I will report to others." d. "I cannot tell anyone about you. It will be as though I am talking about my own problems, and we can help each other by keeping it between us."

ANS: A A patient has the right to know with whom the nurse will share information and that confidentiality will be protected. Although the relationship is primarily between the nurse and patient, other staff members need to know pertinent data. The other incorrect responses promote incomplete disclosure on the part of the patient, require daily renegotiation of an issue that should be resolved as the nurse-patient contract is established, and suggest mutual problem solving. The relationship must be patient centered.

A community mental health nurse has worked with a patient for 3 years but is moving out of the city and terminates the relationship. What is the role of the new nurse who begins working with this patient? a. Beginning at the orientation phase b. Resuming the working relationship c. Entering into a social relationship d. Returning to the emotional catharsis phase

ANS: A After the termination of a long-term relationship, the patient and new nurse usually have to begin at ground zero, the orientation phase, to build a new relationship. If termination is successfully completed, then the orientation phase sometimes progresses quickly to the working phase. Other times, even after successful termination, the orientation phase may be prolonged.

A patient says, "I've done a lot of cheating and manipulating in my relationships." What nonjudgmental response by the nurse is most appropriate? a. "How do you feel about that?" b. "It's good that you realize this." c. "That's not a good way to behave." d. "Have you outgrown that type of behavior?"

ANS: A Asking a patient to reflect on feelings about his or her actions does not imply any judgment about those actions, and it encourages the patient to explore feelings and values. The remaining options offer negative judgments.

Which statement shows a nurse has empathy for a patient who made a suicide attempt? a. "You must have been very upset when you tried to hurt yourself." b. "It makes me sad to see you going through such a difficult experience." c. "If you tell me what is troubling you, I can help you solve your problems." d. "Suicide is a drastic solution to a problem that may not be such a serious matter."

ANS: A Empathy permits the nurse to see an event from the patient's perspective, understand the patient's feelings, and communicate this to the patient. The incorrect responses are nurse centered (focusing on the nurse's feelings rather than the patient's), belittling, and sympathetic.

A nurse designs a plan of exercise for a patient experiencing stress. What rationale should the nurse cite when presenting this plan to the treatment team? a. Exercise will stimulate endorphins and improve the patient's feelings of well-being. b. Exercise prevents damage from overstimulation of the sympathetic nervous system. c. Exercise detoxifies the body by removing metabolic wastes and other toxins. d. Exercise will prevent exacerbation of the stress by the limbic system.

ANS: A Exercise is a stress reduction strategy that stimulates endorphins and improves patient's feelings of well-being. The other options are not accurate.

A soldier returns to the United States from active duty in a combat zone. The soldier is diagnosed with posttraumatic stress disorder (PTSD). The nurse's highest priority is to screen this soldier for which problem? a. Major depressive disorder b. Bipolar disorder c. Schizophrenia d. Dementia

ANS: A Major depressive disorder frequently co-occurs with PTSD. The incidence of the disorders identified in the distractors is similar to the general population.

. A nurse explains to the family of a patient who is mentally ill how the nurse-patient relationship differs from social relationships. Which is the best explanation? a. "The focus is on the patient. Problems are discussed by the nurse and patient, but solutions are implemented by the patient." b. "The focus shifts from nurse to patient as the relationship develops. Advice is given by both, and solutions are implemented." c. "The focus of the relationship is socialization. Mutual needs are met, and feelings are openly shared." d. "The focus is the creation of a partnership in which each member is concerned with the growth and satisfaction of the other."

ANS: A Only the correct response describes the elements of a therapeutic relationship. The remaining responses describe events that occur in social or intimate relationships.

. A veteran of military combat tells the nurse, "I saw a child get blown up over a year ago, and now I keep seeing bits of flesh everywhere. I see something red and the visions race back to my mind." Which phenomenon associated with posttraumatic stress disorder (PTSD) is this veteran describing? a. Re-experiencing b. Hyperarousal c. Avoidance d. Psychosis

ANS: A Spontaneous or cued recurrent, involuntary, and intrusive distressing memories of the traumatic events are often associated with PTSD. The veteran has described intrusive thoughts and visions associated with re-experiencing the traumatic event. This description does not indicate psychosis, hypervigilance, or avoidance.

. A nurse talks with the caregiver of a combat veteran diagnosed with severe traumatic brain injuries. The caregiver says, "I don't know how much longer I can do it. My whole life is consumed with taking care of my partner." Which response best addresses the needs of the caregiver? a. "How are you taking care of yourself?" b. "Let's review your partner's diagnostic results." c. "I have some web-based programs for you to visit." d. "Your partner is lucky to have someone so devoted."

ANS: A The caregiver is the focus of the nurse's attention. The caregiver is suffering. The nurse must be empathetic and assess how the caregiver is caring for self. Reassurance and isolated computer activities do not help. The partner is already aware of the diagnostic results.

As a nurse discharges a patient, the patient gives the nurse a card of appreciation made in an arts and crafts group. What is the nurse's best action? a. Accept the card while recognizing the effectiveness of the relationship and the patient's thoughtfulness. b. Inform the patient that accepting gifts violates the policies of the facility. Decline the card regretfully. c. Acknowledge the patient's transition through the termination phase but decline the card. d. Accept the card and invite the patient to return to participate in other arts and crafts groups.

ANS: A The nurse must consider the meaning, timing, and value of the gift. In this instance, the nurse should accept the patient's expression of gratitude.

At what point in the nurse-patient relationship should a nurse plan to first address termination? a. In the orientation phase b. During the working phase c. In the termination phase d. When the patient initially brings up the topic

ANS: A The patient has a right to know the conditions of the nurse-patient relationship. If the relationship is to be time limited, then the patient should be informed of the number of sessions. If it is open ended, then the termination date will not be known at the outset and the patient will know that the issue will be negotiated at a later date. The nurse is responsible for bringing up the topic of termination early in the relationship, usually during the orientation phase.

A nurse says, "I am the only one who truly understands this patient. Other staff members are too critical." What does the nurse's statement indicate? a. Boundary blurring b. Sexual harassment c. Positive regard d. Advocacy

ANS: A When the role of the nurse and the role of the patient shift, boundary blurring may arise. In this situation, the nurse is becoming overinvolved with the patient as a probable result of unrecognized countertransference. When boundary issues occur, the need for supervision exists. The situation does not describe sexual harassment. Data are not present to suggest positive regard or advocacy

A nurse ends a relationship with a patient. Which actions by the nurse should be included in the termination phase? (Select all that apply.) a. Focus dialog with the patient on problems that may occur in the future. b. Help the patient express feelings about the relationship with the nurse. c. Help the patient prioritize and modify socially unacceptable behaviors. d. Reinforce expectations regarding the parameters of the relationship. e. Help the patient identify strengths, limitations, and problems.

ANS: A, B The correct actions are part of the termination phase. The other actions are used in the working and orientation phases.

A professor's 4-year-old child has a temperature of 101.6° F, diarrhea, and complains of stomach pain. The professor is scheduled to teach three classes today. Which actions by the professor demonstrate effective parenting? (Select all that apply.) a. Telephoning a grandparent to stay with the child at home for the day b. Telephoning a colleague to teach his classes and staying home with the sick child c. Taking the child to the university and keeping the child in a private office for the day d. Taking the child to a day care center and hoping day care workers will not notice the child is sick e. Giving the child one dose of ibuprofen (Motrin) and taking the child to the day care center

ANS: A, B The correct responses demonstrate fulfillment of the role as a parent. The distractors indicate the professor has not cared for the sick child in an effective way. Taking the child to a day care center exposes other children to a potential infection. Taking the child to one's office does not keep the child comfortable or provide for the child while the professor is teaching.

2. Which nursing interventions will be implemented for a patient who is actively suicidal? Select all that apply. a. Maintain arms-length, one-on-one direct observation at all times. b. Check all items brought by visitors and remove risk items. c. Use plastic eating utensils; count utensils upon collection. d. Remove the patients eyeglasses to prevent self-injury. e. Interact with the patient every 15 minutes.

ANS: A, B, C One-on-one observation is necessary for anyone who has limited or unreliable control over suicidal impulses. Finger foods allow the patient to eat without silverware; no silver or glassware orders restrict access to a potential means of self-harm. Every-15-minute checks are inadequate to assure the safety of an actively suicidal person. Placement in a public area is not a substitute for arms-length direct observation; some patients will attempt suicide even when others are nearby. Vision impairment requires eyeglasses (or contacts); although they could be used dangerously, watching the patient from arms length at all times would allow enough time to interrupt such an attempt and would prevent the disorientation and isolation that uncorrected visual impairment could create. PTS: 1 DIF: Cognitive Level: Apply (Application) REF: Page 491-492 (Table 25-5) and (Box 25-4) TOP: Nursing Process: Implementation MSC: Client Needs: Safe, Effective Care Environment

An individual has been diagnosed with a dissociative disorder. Which comorbid psychiatric disorders are most likely to accompany this type of mental illness? (Select all that apply.) a. Substance abuse disorders b. Depression c. Eating disorders d. Personality disorders e. Schizophrenia

ANS: A, B, C, D Comorbid psychiatric disorders associated with dissociative disorders include: substance use disorders, depression and anxiety disorders, eating disorders, PTSD, and personality disorders. Schizophrenia is not associated with this type of disorder.

MULTIPLE RESPONSE 1. A nurse assesses five newly hospitalized patients. Which patients have the highest suicide risk? Select all that apply. a. 82-year-old white male b. 17-year-old white female c. 22-year-old Hispanic male d. 19-year-old Native American male e. 39-year-old African American male

ANS: A, B, D Whites have suicide rates almost twice those of non-whites, and the rate is particularly high for older adult males, adolescents, and young adults. Other high-risk groups include young African American males, Native American males, and older Asian Americans. Rates are not high for Hispanic males. PTS: 1 DIF: Cognitive Level: Analyze (Analysis) REF: Page 481-485 TOP: Nursing Process: Assessment MSC: Client Needs: Psychosocial Integrity

A nurse leads a staff development session about ageism among health care workers. What information should the nurse include about the consequences of ageism? Select all that apply. a. Failure of older adults to receive necessary medical information b. Development of public policy that favors programs for older adults c. Staff shortages because caregivers prefer working with younger adults d. Perception that older adults consume a small share of medical resources e. More ancillary than professional personnel discriminate with regard to age

ANS: A, C Because of society's negative stereotyping of older adults as having little to offer, some staff members avoid working with older patients. Staff shortages in long-term care facilities are often greater than those for acute care settings. Older adult patients often receive less information about their conditions and are offered fewer treatment options than younger patients; some health care staff members perceive them as less able to understand. This problem exists among professional and ancillary personnel. Public policy discriminates against programs for older adults. Societal anger exists because older adults are perceived to consume a disproportionately large share of the medical resources.

A health care provider decided that the emotional distress of an older adult patient warrants the use of risperidone (Risperdal). Which interventions should the nurse add to the patient's plan of care? Select all that apply. a. Monitor for signs and symptoms of diabetes. b. Use disposable briefs for incontinence. c. Monitor for cerebrovascular changes. d. Implement a tyramine-free diet. e. Monitor for dehydration.

ANS: A, C Use of atypical antipsychotic medications increases the risk of diabetes and cerebrovascular events in the older adult population; therefore, the nurse should carefully monitor the patient for changes suggestive of these problems. This medication does not place the patient at great risk for the other options.

Which descriptors exemplify consistency regarding therapeutic nurse-patient relationships? (Select all that apply.) a. Having the same nurse care for a patient on a daily basis b. Encouraging a patient to share initial impressions of staff c. Providing a schedule of daily activities to a patient d. Setting a time for regular sessions with a patient e. Offering solutions to a patient's problems

ANS: A, C, D Consistency implies predictability. Having the same nurse see the patient daily, providing a daily schedule of patient activities, and setting a regular time for sessions help a patient to predict what will happen during each day and to develop a greater degree of security and comfort. Encouraging a patient to share initial impressions of staff and giving advice are not related to consistency and would not be considered a therapeutic intervention.

3. A college student is extremely upset after failing two examinations. The student said, No one understands how this will hurt my chances of getting into medical school. The student then suspends access to his social networking website and turns off his cell phone. Which suicide risk factors are evident? Select all that apply. a. Shame b. Panic attack c. Humiliation d. Self-imposed isolation e. Recent stressful life event

ANS: A, C, D, E Failing examinations in the academic major constitutes a recent stressful life event. Shame and humiliation related to the failure can be hypothesized. The statement, No one can understand, can be seen as recent lack of social support. Terminating access to ones social networking site and turning off the cell phone represents self-imposed isolation. The scenario does not provide evidence of panic attack. PTS: 1 DIF: Cognitive Level: Apply (Application) REF: Page 481-483 TOP: Nursing Process: Assessment MSC: Client Needs: Psychosocial Integrity

A new psychiatric nurse is providing care to a parent diagnosed with bipolar disorder. This nurse angrily recalls embarrassing events concerning the parent's behavior in the community. Select the best ways for this nurse to cope with these feelings. (Select all that apply.) a. Seeking ways to use the understanding gained from childhood to help patients cope with their own illnesses b. Recognizing that these feelings are unhealthy and try to suppress them when working with patients c. Recognizing that psychiatric nursing is not an appropriate career choice and explore other nursing specialties d. Beginning new patient relationships by saying, "My own parent had mental illness, so I accept it without stigma" e. Recognizing that the feelings may add sensitivity to the nurse's practice, but supervision is important

ANS: A, E The nurse needs to explore these feelings. An experienced psychiatric nurse is a resource who may be helpful. The knowledge and experience gained from the nurse's relationship with a parent who is mentally ill may contribute sensitivity to a compassionate practice. Self-disclosure and suppression are not adaptive coping strategies. The nurse should not give up on this area of practice without first seeking ways to cope with the memories.

A nurse asks a patient, "If you had fever and vomiting for 3 days, what would you do?" Which aspect of the mental status examination is the nurse assessing? a. Behavior b. Cognition c. Affect and mood d. Perceptual disturbances

ANS: B Assessing cognition involves determining a patient's judgment and decision making. In this case, the nurse would expect a response of "Call my doctor" if the patient's cognition and judgment are intact. If the patient responds, "I would stop eating" or "I would just wait and see what happened," the nurse would conclude that judgment is impaired. The other options refer to other aspects of the examination.

Before assessing a new patient, a nurse is told by another health care worker, "I know that patient. No matter how hard we work, there isn't much improvement by the time of discharge." The nurse's responsibility is to: a. document the other worker's assessment of the patient. b. assess the patient based on data collected from all sources. c. validate the worker's impression by contacting the patient's significant other. d. discuss the worker's impression with the patient during the assessment interview.

ANS: B Assessment should include data obtained from both the primary and reliable secondary sources. The nurse, bearing in mind the possible effects of counter-transference, should evaluate biased assessments by others as objectively as possible.

A patient presents to the emergency department with mixed psychiatric symptoms. The admission nurse suspects the symptoms may be the result of a medical problem. Lab results show elevated BUN (blood urea nitrogen) and creatinine. What is the nurse's next best action? a. Report the findings to the health care provider. b. Assess the patient for a history of renal problems. c. Assess the patient's family history for cardiac problems. d. Arrange for the patient's hospitalization on the psychiatric unit.

ANS: B Elevated BUN (blood urea nitrogen) and creatinine suggest renal problems. Renal dysfunction can often imitate psychiatric disorders. The nurse should further assess the patient's history for renal problems and then share the findings with the health care provider.

22. Which statement by a depressed patient will alert the nurse to the patients need for immediate, active intervention? a. I am mixed up, but I know I need help. b. I have no one to turn to for help or support. c. It is worse when you are a person of color. d. I tried to get attention before I cut myself last time.

ANS: B Hopelessness is evident. Lack of social support and social isolation increases the suicide risk. Willingness to seek help lowers risk. Being a person of color does not suggest higher risk because more whites commit suicide than do individuals of other racial groups. Attention seeking is not correlated with higher suicide risk. PTS: 1 DIF: Cognitive Level: Apply (Application) REF: Page 487-490 (Table 25-3) and (Nursing Care Plan 25-1) TOP: Nursing Process: Planning MSC: Client Needs: Safe, Effective Care Environment

23. A patient hospitalized for 2 weeks committed suicide during the night. Which initial nursing measure will be most important regarding this event? a. Ask the information technology manager to verify the hospital information system is secure. b. Hold a staff meeting to express feelings and plan care for the other patients. c. Ask the patients roommate not to discuss the event with other patients. d. Prepare a report of a sentinel event.

ANS: B Interventions should help the staff and patients come to terms with the loss and grow because of the incident. Then, a community meeting should occur to allow other patients to express their feelings and request help. Staff should be prepared to provide additional support and reassurance to patients and should seek opportunities for peer support. A sentinel event report can be prepared later. The other incorrect options will not control information or would result in unsafe care. PTS: 1 DIF: Cognitive Level: Apply (Application) REF: Page 487-488 | Page 493-494 TOP: Nursing Process: Implementation MSC: Client Needs: Safe, Effective Care Environment

Which entry in the medical record best meets the requirement for problem-oriented charting? a. "A: Pacing and muttering to self. P: Sensory perceptual alteration related to internal auditory stimulation. I: Given fluphenazine HCL (Prolixin) 2.5 mg po at 0900 and went to room to lie down. E: Calmer by 0930. Returned to lounge to watch TV." b. "S: States, 'I feel like I'm ready to blow up.' O: Pacing hall, mumbling to self. A: Auditory hallucinations. P: Offer haloperidol (Haldol) 2 mg po. I: Haloperidol (Haldol) 2 mg po given at 0900. E: Returned to lounge at 0930 and quietly watched TV." c. "Agitated behavior. D: Patient muttering to self as though answering an unseen person. A: Given haloperidol (Haldol) 2 mg po and went to room to lie down. E: Patient calmer. Returned to lounge to watch TV." d. "Pacing hall and muttering to self as though answering an unseen person. haloperidol (Haldol) 2 mg po administered at 0900 with calming effect in 30 minutes. Stated, 'I'm no longer bothered by the voices.'"

ANS: B Problem-oriented documentation uses the first letter of key words to organize data: S for subjective data, O for objective data, A for assessment, P for plan, I for intervention, and E for evaluation. The distracters offer examples of PIE charting, focus documentation, and narrative documentation.

4. Which change in the brains biochemical function is most associated with suicidal behavior? a. Dopamine excess b. Serotonin deficiency c. Acetylcholine excess d. Gamma-aminobutyric acid deficiency

ANS: B Research suggests that low levels of serotonin may play a role in the decision to commit suicide. The other neurotransmitter alterations have not been implicated in suicidality. PTS: 1 DIF: Cognitive Level: Understand (Comprehension) REF: Page 484-485 TOP: Nursing Process: Assessment MSC: Client Needs: Physiological Integrity

1. An adult outpatient diagnosed with major depression has a history of several suicide attempts by overdose. Given this patients history and diagnosis, which antidepressant medication would the nurse expect to be prescribed? a. Amitriptyline (Elavil), a sedating tricyclic medication b. Fluoxetine (Prozac), a selective serotonin reuptake inhibitor c. Desipramine (Norpramin), a stimulating tricyclic medication d. Tranylcypromine sulfate (Parnate), a monoamine oxidase inhibitor

ANS: B Selective serotonin reuptake inhibitor antidepressants are very safe in overdosage situations, which is not true of the other medications listed. Given this patients history of overdosing, it is important that the medication be as safe as possible in case she takes an overdose of her prescribed medication. PTS: 1 DIF: Cognitive Level: Analyze (Analysis) REF: Page 492 TOP: Nursing Process: Planning MSC: Client Needs: Physiological Integrity

6. A nurse uses the SAD PERSONS scale to interview a patient. This tool provides data relevant to: a. current stress level. b. suicide potential. c. mood disturbance. d. level of anxiety.

ANS: B The SAD PERSONS tool evaluates 10 major risk factors in suicide potential: sex, age, depression, previous attempt, ethanol use, rational thinking loss, social supports lacking, organized plan, no spouse, and sickness. The tool does not have categories to provide information on the other options listed. PTS: 1 DIF: Cognitive Level: Understand (Comprehension) REF: Page 486-487 (Table 25-2) TOP: Nursing Process: Assessment MSC: Client Needs: Psychosocial Integrity

10. Select the most critical question for the nurse to ask an adolescent who has threatened to take an overdose of pills. a. Why do you want to kill yourself? b. Do you have access to medications? c. Have you been taking drugs and alcohol? d. Did something happen with your parents?

ANS: B The nurse must assess the patients access to means to carry out the plan and, if there is access, alert the parents to remove from the home and take additional actions to assure the patients safety. The information in the other questions may be important to ask but are not the most critical. PTS: 1 DIF: Cognitive Level: Apply (Application) REF: Page 483 (Box 25-2) | Page 486-487 TOP: Nursing Process: Assessment MSC: Client Needs: Safe, Effective Care Environment

A nurse wants to assess an adult patient's recent memory. Which question would best yield the desired information? a. "Where did you go to elementary school?" b. "What did you have for breakfast this morning?" c. "Can you name the current president of the United States?" d. "A few minutes ago, I told you my name. Can you remember it?"

ANS: B The patient's recall of a meal provides evidence of recent memory. Two incorrect responses are useful to assess immediate and remote memory. The other distracter assesses the patient's fund of knowledge.

After formulating the nursing diagnoses for a new patient, what is a nurse's next action? a. Designing interventions to include in the plan of care b. Determining the goals and outcome criteria c. Implementing the nursing plan of care d. Completing the spiritual assessment

ANS: B The third step of the nursing process is planning and outcome identification. Outcomes cannot be determined until the nursing assessment is complete and nursing diagnoses have been formulated.

A group has two more sessions before it ends. One member was previously vocal and has shown much progress but has now grown silent. What explanation most likely underlies this behavior? The silent member: a. has participated in the group and now has nothing more to offer. b. is having trouble dealing with feelings about termination of this group. c. wants to give quieter members a chance to talk in the remaining sessions. d. is engaging in attention-seeking behavior aimed at continuation of the group.

ANS: B A chief task during the termination phase of a group is to take what has been learned in group and transition to life without the group. The end of a group can be a significant loss for members, who may experience loss and grief and respond with sadness or anger. It is unlikely he would have nothing to say; at the very least, he could be responding to the comments of others even if not focusing on his own issues. He may wish to give quieter members a chance to talk, but again, this would not require or explain his complete silence. Some members, faced with only two remaining sessions, may be becoming more dominant under this pressure of time, but here too this is unlikely to lead a previously active participant to fall completely silent. The member is not attention-seeking.

A student nurse visiting a senior center tells the instructor, "It's so depressing to see all these old people. They are so weak and frail. They are probably all confused." The student is expressing: a. reality. b. ageism. c. empathy. d. advocacy.

ANS: B Ageism is defined as a bias against older people because of their age. None of the other options can be identified from the ideas expressed by the student.

A clinic nurse interviews four patients between 70 and 80 years of age. Which patient should have further assessment regarding the risk of alcohol addiction? The patient: a. with a history of intermittent problems of alcohol misuse early in life and who now consumes one glass of wine nightly with dinner. b. with no history of alcohol-related problems until age 65 years, when the patient began to drink alcohol daily "to keep my mind off my arthritis." c. who drank socially throughout adult life and continues this pattern, saying, "I've earned the right to do as I please." d. who abused alcohol between the ages of 25 and 40 years but now abstains and occasionally attends Alcoholics Anonymous.

ANS: B Alcohol addiction can develop at any age, and the geriatric population is particularly at risk. The geriatric problem drinker is defined as someone who has no history of alcohol-related problems but develops an alcohol-abuse pattern in response to the stresses of aging. The incorrect responses profile alcohol use that is not problematic.

Which type of group is a staff nurse with 2 months' psychiatric experience best qualified to conduct? a. Psychodynamic/psychoanalytic group b. Medication education group c. Existential/Gestalt group d. Family therapy group

ANS: B All nurses receive information about patient teaching strategies and basic information about psychotropic medications, making a medication education group a logical group for a beginner to conduct. The other groups would need a leader with more education and experience.

During a support group, a patient diagnosed with schizophrenia says, "Sometimes I feel sad that I will never have a good job like my brother. Then I dwell on it and maybe I should not." Select the nurse leader's best comment to facilitate discussion of this issue. a. "It is often better to focus on our successes rather than our failures." b. "How have others in the group handled painful feelings like these?" c. "Grieving for what is lost is a normal part of having a mental disorder." d. "I wonder if you might also experience feelings of anger and helplessness."

ANS: B Asking others to share their experiences will facilitate discussion of an issue. Giving information may serve to close discussion of the issue because it sounds final. Suggesting a focus on the positives implies a discussion of the issue is not appropriate. Suggesting other possible feelings is inappropriate at this point, considering the patient has identified feelings of sadness and seems to have a desire to explore this feeling. Focusing on other feelings will derail discussion of the patient's grief for his perceived lost potential.

A 75-year-old patient comes to the clinic reporting frequent headaches. After an introduction at the beginning of the interview, the nurse should: a. initiate a neurologic assessment. b. ask if the patient can hear clearly as the nurse speaks. c. suggest that the patient lie down in a darkened room for a few minutes. d. administer medication to relieve the patient's pain before performing the assessment.

ANS: B Before proceeding, the nurse should assess the patient's ability to hear questions. Hearing ability often declines with age. Impaired hearing could lead to inaccurate answers. The nurse should not administer medication (an intervention) until after the assessment is complete

A nurse assesses a confused older adult. The nurse experiences sadness and reflects, "The patient is like one of my grandparents...so helpless." Which response is the nurse demonstrating? a. Transference c. Catastrophic reaction b. Countertransference d. Defensive coping reaction

ANS: B Countertransference is the nurse's transference or response to a patient that is based on the nurse's unconscious needs, conflicts, problems, or view of the world. See relationship to audience response question.

A nurse wants to demonstrate genuineness with a patient diagnosed with schizophrenia. The nurse should: a. restate what the patient says. b. use congruent communication strategies. c. use self-revelation in patient interactions. d. consistently interpret the patient's behaviors.

ANS: B Genuineness is a desirable characteristic involving awareness of one's own feelings as they arise and the ability to communicate them when appropriate. The incorrect options are undesirable in a therapeutic relationship.

A 74-year-old patient is regressed and apathetic. This patient responds to others only when they initiate the interaction. Which therapy would be most useful to promote resocialization? a. Life review b. Remotivation c. Group psychotherapy d. Individual psychotherapy

ANS: B Remotivation therapy is designed to resocialize patients who are regressed and apathetic by focusing on a single topic, creating a bridge to reality as group members talk about the world in which they live and work, and hobbies related to the topic. Group leaders give group members acceptance and appreciation

Which is the best comment for a nurse to use when beginning an interview with an older adult patient? a. "Hello, [call patient by first name]. I am going to ask you some questions to get to know you better." b. "Hello. My name is [nurse's name]. I am a nurse. Please tell me how you would like to be addressed by the staff." c. "I am going to ask you some questions about yourself. I would like to call you by your first name if you don't mind." d. "You look as though you are comfortable and ready to participate in an admission interview. Shall we get started?"

ANS: B The correct response identifies the nurse's role and politely seeks direction for addressing the patient in a way that will make him or her comfortable. This is particularly important when a considerable age difference exists between the nurse and the patient. The nurse should address a patient by name, but should not assume the patient wants to be called by his or her first name. The nurse should always introduce himself or herself.

An 80-year-old patient has difficulty walking because of arthritis and says, "It's awful to be old. Every day is a struggle. No one cares about old people." Which is the nurse's most therapeutic response? a. "Everyone here cares about old people. That's why we work here." b. "It sounds like you're having a difficult time. Tell me about it." c. "Let's not focus on the negative. Tell me something good." d. "You are still able to get around, and your mind is alert."

ANS: B The nurse uses empathic understanding to permit the patient to express frustration and clarify the "struggle" for the nurse. The other options are nontherapeutic and block communication.

In a sad voice, a patient tells the nurse of the recent deaths of a spouse of 50 years as well as an adult child in an automobile accident. The patient has no other family and only a few friends in the community. What is the priority nursing diagnosis? a. Spiritual distress, related to being angry with God for taking the family b. Risk for suicide, related to recent deaths of significant others c. Anxiety, related to sudden and abrupt lifestyle changes d. Social isolation, related to loss of existing family

ANS: B The patient appears to be experiencing normal grief related to the loss of the family; however, because of age and social isolation, the risk for suicide should be determined and has high priority. No defining characteristics exist for the diagnosis of anxiety or spiritual distress. Risk for suicide is a higher priority than social isolation

A 79-year-old white man tells a visiting nurse, "I've been feeling down lately. My family and friends are all dead. My money is running out, and my health is failing." The nurse should analyze this comment as: a. normal negativity of older adults. b. evidence of suicide risk. c. a cry for sympathy. d. normal grieving.

ANS: B The patient describes the loss of significant others, economic insecurity, and declining health. He describes mood alteration and expresses the thought that he has little to live for. Combined with his age, sex, and single status, each is a risk factor for suicide. Older adult white men have the highest risk for completed suicide

A patient in a detoxification unit asks, "What good it will do to go to Alcoholics Anonymous and talk to other people with the same problem?" The nurse's best response would be to explain that self-help groups such as AA provide opportunities for: a. newly discharged alcoholics to learn about the disease of alcoholism. b. people with common problems to share their experiences with alcoholism and recovery. c. patients with alcoholism to receive insight-oriented treatment about the etiology of their disease. d. professional counselors to provide guidance to individuals recovering from alcoholism.

ANS: B The patient needs basic information about the purpose of a self-help group. The basis of self-help groups is sharing by individuals with similar problems. Self-help is based on the belief that an individual with a problem can be truly understood and helped only by others who have the same problem. The other options fail to address this or provide incorrect information.

Why should the nurse introduce the matter of a contract during the first session with a new patient? a. To specify what the nurse will do for the patient b. To explain the participation and responsibilities of each party c. To indicate the feeling tone established between the participants d. To prevent either party from prematurely ending the relationship

ANS: B A contract emphasizes that the nurse works with the patient rather than doing something for the patient. "Working with" is a process that suggests each party is expected to participate and share responsibility for the outcomes. Contracts do not, however, stipulate roles or feeling tone, or that premature termination is forbidden

As part of the stress response, the HPA axis is stimulated. Which structures make up this system? a. Hippocampus, parietal lobe, and amygdala b. Hypothalamus, pituitary gland, and adrenal glands c. Hind brain, pyramidal nervous system, and anterior cerebrum d. Hepatic artery, parasympathetic nervous system, and acoustic nerve

ANS: B As part of the physiological response of stress, the hypothalamus stimulates the HPA axis, which is made up of the hypothalamus, pituitary gland, and adrenal glands.

Cortisol is released in response to a patient's prolonged stress. Which initial effect would the nurse expect to result from the increased cortisol level? a. Diuresis and electrolyte imbalance b. Focused and alert mental status c. Drowsiness and lethargy d. Restlessness and anxiety

ANS: B Cortisol is the primary stress hormone and is released in response to prolonged stress. Cortisol helps supply cells with amino acids and fatty acids for energy supply, as well as diverting glucose from muscles for use by the brain. As a result, the brain stays alert and focused. The distractors present effects that would not be expected.

A nurse assesses a confused older adult. The nurse experiences sadness and reflects, "The patient is like one of my grandparents, so helpless." What feelings does the nurse describe? a. Transference b. Countertransference c. Catastrophic reaction d. Defensive coping reaction

ANS: B Countertransference is the nurse's transference or response to a patient that is based on the nurse's unconscious needs, conflicts, problems, or view of the world.

Which scenario best demonstrates an example of eustress? a. A child loses a beloved family pet. b. A single male prepares to take a 1-week vacation to a tropical island with a group of close friends. c. A couple receives a bank notice there were insufficient funds in their account for a recent rent payment. d. A married female receives notification that their current employer is experiencing financial problems and some workers will be terminated.

ANS: B Eustress is beneficial stress; it motivates people to develop skills they need to solve problems and meet personal goals. Positive life experiences produce eustress. Going on a tropical vacation is an exciting, relaxing experience and an example of eustress. Losing the family pet, worrying about employment security, and having financial problems are examples of distress, a negative experience that drains energy and can lead to significant emotional problems.

A veteran of the war in Afghanistan was diagnosed with posttraumatic stress disorder (PTSD). The veteran says, "If there's a loud noise at night, I get under my bed because I think we're getting bombed." What type of experience has the veteran described? a. Illusion b. Flashback c. Nightmare d. Auditory hallucination

ANS: B Flashbacks are dissociative reactions in which an individual feels or acts as if the traumatic event were recurring. Illusions are misinterpretations of stimuli; although the experience is similar, the more accurate term is flashback because of the diagnosis of PTSD. Auditory hallucinations have no external stimuli. Nightmares commonly accompany PTSD, but this experience is stimulated by an actual environmental sound.

. How should the nurse who wants to demonstrate genuineness with a patient diagnosed with schizophrenia do so most effectively? a. By restating what the patient says. b. By using congruent communication strategies. c. By using self-disclosure in patient interactions. d. By consistently interpreting the patient's behaviors

ANS: B Genuineness is a desirable characteristic involving an awareness of one's own feelings as they arise and the ability to communicate them when appropriate. The incorrect options are undesirable in a therapeutic relationship.

. A nurse wants to enhance the growth of a patient by showing positive regard. What nursing action is consistent with this wish? a. Making patient rounds daily b. Staying with a tearful patient c. Administering daily medication as prescribed d. Examining personal feelings about a patient

ANS: B Staying with a crying patient offers support and shows positive regard. Administering daily medication and making rounds are tasks that could be part of an assignment and do not necessarily reflect positive regard. Examining feelings regarding a patient addresses the nurse's ability to be therapeutic.

A patient diagnosed with liver failure has been on the transplant waiting list 8 months. The patient says, "Why is it taking so long to have the surgery? Maybe I'm meant to die for all the bad things I've done." The nurse should document the patient's comment in which section of the assessment? a. Physical b. Spiritual c. Financial d. Psychological

ANS: B Stress can be evident in a person's spirituality. This patient's comment indicates questioning of one's place in the universe and consequences for wrongdoing, both are elements of spirituality. Stress can be related to psychological, physical, or psychosocial well-being, but spirituality is the best answer.

A soldier returns to the United States from active duty in a combat zone. The soldier is diagnosed with posttraumatic stress disorder (PTSD). Which comment by the soldier requires the nurse's immediate attention? a. "It's good to be home. I missed my family and friends." b. "I saw my best friend get killed by a roadside bomb. It should have been me instead." c. "Sometimes I think I hear bombs exploding, but it's just the noise of traffic in my hometown." d. "I want to continue my education but I'm not sure how I will fit in with other college students."

ANS: B The correct response indicates the soldier is thinking about death and feeling survivor's guilt. These emotions may accompany suicidal ideation, which warrants the nurse's follow-up assessment. Suicide is a high risk among military personnel diagnosed with PTSD. One distractor indicates flashbacks, which is common with individuals with PTSD but is not solely indicative of further problems. The other distractors are normal emotions associated with returning home and change.

What patient behavior is the desirable outcome for the orientation stage of a nurse-patient relationship? a. Gaining a sense of independence b. Building rapport and trust with the nurse c. Assuming self-responsibility and autonomy d. Effective resolution of feelings of transference

ANS: B The development of rapport and trust is necessary before the relationship can progress to the working phase. Behaviors indicating a greater sense of independence, self-responsibility, and resolved transference occur in the working phase.

. As a patient diagnosed with mental illness is being discharged from a facility, a nurse invites the patient to the annual staff picnic. What is the best analysis of this scenario? a. The invitation facilitates dependency on the nurse. b. The nurse's action blurs the boundaries of the therapeutic relationship. c. The invitation is therapeutic for the patient's diversional activity deficit. d. The nurse's action assists the patient's integration into community living.

ANS: B The invitation creates a social relationship rather than a therapeutic relationship.

An older patient drinks a six-pack of beer daily. The patient tells the community health nurse, "I've been having trouble with my arthritis lately, so I take acetaminophen four times a day for pain." What are the nurse's priority interventions? Select all that apply. a. Inquiring about sleep disturbances caused by mixing alcohol and analgesic medications. b. Determining the safety of the daily acetaminophen dose the patient is ingesting. c. Advising the patient of harmful effects of alcohol and acetaminophen on the liver. d. Suggesting an increase in the acetaminophen dose because alcohol causes faster excretion. e. Assessing the patient for declining functional status associated with medication-induced dementia.

ANS: B, C The nurse should be concerned with the patient's use of alcohol and acetaminophen because the toxicity of acetaminophen is enhanced by alcohol and by the age-related decrease in clearance. The nurse must determine whether the acetaminophen dose is within safe limits or is excessive and provide this information to the patient. Next, the nurse must provide health education regarding the danger of combined use of acetaminophen and alcohol. The patient will need to discontinue or reduce alcohol intake. Another analgesic with less hepatotoxicity could be used. Additional acetaminophen would cause greater liver damage. The scenario does not suggest dementia.

Which descriptors exemplify consistency regarding nurse-patient relationships? Select all that apply. a. Encouraging a patient to share initial impressions of staff b. Having the same nurse care for a patient on a daily basis c. Providing a schedule of daily activities to a patient d. Setting a time for regular sessions with a patient e. Offering solutions to a patient's problems

ANS: B, C, D Consistency implies predictability. Having the same nurse see the patient daily and provide a daily schedule of patient activities and a set time for regular sessions will help a patient predict what will happen during each day and develop a greater degree of security and comfort. Encouraging a patient to share initial impressions of staff and giving advice are not related to consistency and would not be considered a therapeutic intervention.

Which beliefs facilitate provision of safe, effective care for older adult patients? Select all that apply. a. Sexual interest declines with aging. b. Older adults are able to learn new tasks. c. Aging results in a decline in restorative sleep. d. Older adults are prone to become crime victims. e. Older adults are usually lonely and socially isolated.

ANS: B, C, D Myths about aging are common and can negatively impact the quality of care older patients receive. Older individuals are more prone to become crime victims. A decline in restorative sleep occurs as one ages. Learning continues long into life. These factors affect care delivery.

A new nurse tells a mentor, "I want to convey to my patients that I am interested in them and that I want to listen to what they have to say." Which behaviors are helpful in meeting the nurse's goal? (Select all that apply.) a. Sitting behind a desk, facing the patient b. Introducing self to a patient and identifying own role c. Using facial expressions that convey interest and encouragement d. Assuming an open body posture and sometimes mirror imaging e. Maintaining control of the topic under discussion by asking direct questions

ANS: B, C, D Trust is fostered when the nurse gives an introduction and identifies his or her role. Facial expressions that convey interest and encouragement support the nurse's verbal statements to that effect and strengthen the message. An open body posture conveys openness to listening to what the patient has to say. Mirror imaging enhances patient comfort. A desk would place a physical barrier between the nurse and patient. A face-to-face stance should be avoided when possible, and a less intense, 90- or 120-degree angle is used to permit either party to look away without discomfort. Once introductions have been made, the nurse focuses the interview on the patient by using open-ended questions, such as, "Where should we start?"

A nurse assessing an older adult patient for depression should include questions about mood as well as which other symptoms? Select all that apply. a. Increased appetite b. Sleep pattern changes c. Anhedonia and anergia d. Increased social isolation e. Increased concern with bodily functions

ANS: B, C, D, E These symptoms are often noted in older adult patients experiencing depression. Somatic symptoms are often present but are missed by nurses as being related to depression. Anorexia, rather than hyperphagia, is observed in major depressive disorder. Low self-esteem is more often associated with major depressive disorder

A patient is very suspicious and states, "The FBI has me under surveillance." Which strategies should a nurse use when gathering initial assessment data about this patient? Select all that apply. a. Tell the patient that medication will help this type of thinking. b. Ask the patient, "Tell me about the problem as you see it." c. Seek information about when the problem began. d. Tell the patient, "Your ideas are not realistic." e. Reassure the patient, "You are safe here."

ANS: B, C, E During the assessment interview, the nurse should listen attentively and accept the patient's statements in a nonjudgmental way. Because the patient is suspicious and fearful, reassuring safety may be helpful, although trust is unlikely so early in the relationship. Saying that medication will help or telling the patient that the ideas are not realistic will undermine development of trust between the nurse and patient.

The next-to-last meeting of an interpersonal therapy group is taking place. The leader should take which actions? Select all that apply. a. Support appropriate expressions of disagreement by the group's members. b. Facilitate discussion and resolution of feelings about the end of the group. c. Encourage members to reflect on their progress and that of the group itself. d. Remind members of the group's norms and rules, emphasizing confidentiality. e. Help members identify goals they would like to accomplish after the group ends. f. Promote the identification and development of new options for solving problems.

ANS: B, C, E The goals for the termination phase of groups are to prepare the group for separation, resolve related feelings, and prepare each member for the future. Contributions and accomplishments of members are elicited, post-group goals are identified, and feelings about the group's ending are discussed. Group norms are the focus of the orientation phase, and conflict and problem solving are emphasized in the working phase.

1. A nurse assessed a patient who reluctantly participated in activities, answered questions with minimal responses, and rarely made eye contact. What information should be included when documenting the assessment? Select all that apply. a. The patient was uncooperative b. The patient's subjective responses c. Only data obtained from the patient's verbal responses d. A description of the patient's behavior during the interview e. Analysis of why the patient was unresponsive during the interview

ANS: B, D Both content and process of the interview should be documented. Providing only the patient's verbal responses would create a skewed picture of the patient. Writing that the patient was uncooperative is subjectively worded. An objective description of patient behavior would be preferable. Analysis of the reasons for the patient's behavior would be speculation, which is inappropriate.

What information is conveyed by nursing diagnoses? Select all that apply. a. Medical judgments about the disorder b. Unmet patient needs currently present c. Goals and outcomes for the plan of care d. Supporting data that validate the diagnoses e. Probable causes that will be targets for nursing interventions

ANS: B, D, E Nursing diagnoses focus on phenomena of concern to nurses rather than on medical diagnoses.

A novice nurse tells a mentor, "I want to convey to my patients that I am interested in them and that I want to listen to what they have to say." Which behaviors will be helpful in meeting the nurse's goal? Select all that apply. a. Sitting behind a desk, facing the patient b. Introducing self to a patient and identifying own role c. Maintaining control of discussions by asking direct questions d. Using facial expressions to convey interest and encouragement e. Assuming an open body posture and sometimes mirror imaging

ANS: B, D, E Trust is fostered when the nurse gives an introduction and identifies his or her role. Facial expressions that convey interest and encouragement support the nurse's verbal statements to that effect and strengthen the message. An open body posture conveys openness to listening to what the patient has to say. Mirror imaging enhances patient comfort. A desk would place a physical barrier between the nurse and patient. A face-to-face stance should be avoided when possible and a less intense 90- or 120-degree angle used to permit either party to look away without discomfort.

A leader begins the discussion at the first meeting of a new group. Which comments should be included? Select all that apply. a. "We use groups to provide treatment because it's a more cost-effective use of staff in this time of budget constraints." b. "When someone shares a personal experience, it's important to keep the information confidential." c. "Talking to family members about our group discussions will help us achieve our goals." d. "Everyone is expected to share a personal experience at each group meeting." e. "It is important for everyone to arrive on time for our group."

ANS: B, E The leader must set ground rules for the group before members can effectively participate. Confidentiality of personal experiences should be maintained. Arriving on time is important to the group process. Talking to family members would jeopardize confidentiality. While groups are cost-effective, blaming the budget would not help members feel valued. Setting an expectation to share may be intimidating for a withdrawn patient.

An adolescent asks a nurse conducting an assessment interview, "Why should I tell you anything? You'll just tell my parents whatever you find out." Which response by the nurse is appropriate? a. "That isn't true. What you tell us is private and held in strict confidence. Your parents have no right to know." b. "Yes, your parents may find out what you say, but it is important that they know about your problems." c. "What you say about feelings is private, but some things, like suicidal thinking, must be reported to the treatment team." d. "It sounds as though you are not really ready to work on your problems and make changes."

ANS: C Adolescents are very concerned with confidentiality. The patient has a right to know that most information will be held in confidence but that certain material must be reported or shared with the treatment team, such as threats of suicide, homicide, use of illegal drugs, or issues of abuse. The incorrect responses are not true, will not inspire the confidence of the patient, or are confrontational.

5. A college student who failed two tests cried for hours and then tried to telephone a parent but got no answer. The student then gave several expensive sweaters to a roommate and asked to be left alone for a few hours. Which behavior provides the strongest clue of an impending suicide attempt? a. Calling parents b. Excessive crying c. Giving away sweaters d. Staying alone in dorm room

ANS: C Giving away prized possessions may signal that the individual thinks he or she will have no further need for the item, such as when a suicide plan has been formulated. Calling parents, remaining in a dorm, and crying do not provide direct clues to suicide. PTS: 1 DIF: Cognitive Level: Apply (Application) REF: Page 486-487 TOP: Nursing Process: Assessment MSC: Client Needs: Psychosocial Integrity

20. When assessing a patients plan for suicide, what aspect has priority? a. Patients financial and educational status b. Patients insight into suicidal motivation c. Availability of means and lethality of method d. Quality and availability of patients social support

ANS: C If a person has plans that include choosing a method of suicide readily available and if the method is one that is lethal (i.e., will cause the person to die with little probability for intervention), the suicide risk is high. These areas provide a better indication of risk than the areas mentioned in the other options. See relationship to audience response question. PTS: 1 DIF: Cognitive Level: Apply (Application) REF: Page 486-487 TOP: Nursing Process: Assessment MSC: Client Needs: Safe, Effective Care Environment

A patient begins a new program to assist with building social skills. In which part of the plan of care should a nurse record the item, "Encourage patient to attend one psychoeducational group daily"? a. Assessment b. Analysis c. Implementation d. Evaluation

ANS: C Interventions are the nursing prescriptions to achieve the outcomes. Interventions should be specific.

Select the most appropriate label to complete this nursing diagnosis: ___________ related to feelings of shyness and poorly developed social skills as evidenced by watching television alone at home every evening. a. Deficient knowledge b. Ineffective coping c. Social isolation d. Powerlessness

ANS: C Nursing diagnoses are selected based on the etiological factors and assessment findings, or evidence. In this instance, the evidence shows social isolation that is caused by shyness and poorly developed social skills.

A newly admitted patient diagnosed with major depression has gained 20 pounds over a few months and has suicidal ideation. The patient has taken an antidepressant medication for 1 week without remission of symptoms. Select the priority nursing diagnosis. a. Imbalanced nutrition: more than body requirements b. Chronic low self-esteem c. Risk for suicide d. Hopelessness

ANS: C Risk for suicide is the priority diagnosis when the patient has both suicidal ideation and a plan to carry out the suicidal intent. Imbalanced nutrition, hopelessness, and chronic low self-esteem may be applicable nursing diagnoses, but these problems do not affect patient safety as urgently as would a suicide attempt.

15. Which intervention will the nurse recommend for the distressed family and friends of someone who has committed suicide? a. Participating in reminiscence therapy b. Psychological postmortem assessment c. Attending a self-help group for survivors d. Contracting for at least two sessions of group therapy

ANS: C Survivors need outlets for their feelings about the loss and the deceased person. Self-help groups provide peer support while survivors work through feelings of loss, anger, and guilt. Psychological postmortem assessment would not provide the support necessary to work through feelings of loss associated with the suicide. Reminiscence therapy is not geared to loss resolution. Contracting for two sessions of group therapy would not provide sufficient time to work through the issues associated with a death by suicide. PTS: 1 DIF: Cognitive Level: Apply (Application) REF: Page 493-494 TOP: Nursing Process: Implementation MSC: Client Needs: Psychosocial Integrity

12. A nurse and patient construct a no-suicide contract. Select the preferable wording. a. I will not try to harm myself during the next 24 hours. b. I will not make a suicide attempt while I am hospitalized. c. For the next 24 hours, I will not in any way attempt to harm or kill myself. d. I will not kill myself until I call my primary nurse or a member of the staff.

ANS: C The correct answer leaves no loopholes. The wording about not harming oneself and not making an attempt leaves loopholes or can be ignored by the patient who thinks I am not going to harm myself, I am going to kill myself or I am not going to attempt suicide, I am going to commit suicide. A patient may call a therapist and leave the telephone to carry out the suicidal plan. PTS: 1 DIF: Cognitive Level: Apply (Application) REF: Page 491-492 (Table 26-5) TOP: Nursing Process: Implementation MSC: Client Needs: Safe, Effective Care Environment

14. A nurse interacts with an outpatient who has a history of multiple suicide attempts. Select the most helpful response for a nurse to make when the patient states, I am considering committing suicide. a. Im glad you shared this. Please do not worry. We will handle it together. b. I think you should admit yourself to the hospital to keep you safe. c. Bringing up these feelings is a very positive action on your part. d. We need to talk about the good things you have to live for.

ANS: C The correct response gives the patient reinforcement, recognition, and validation for making a positive response rather than acting out the suicidal impulse. It gives neither advice nor false reassurance, and it does not imply stereotypes such as You have a lot to live for. It uses the patients ambivalence and sets the stage for more realistic problem solving. PTS: 1 DIF: Cognitive Level: Apply (Application) REF: Page 484 | Page 488-491 (Nursing Care Plan 25-1) TOP: Nursing Process: Implementation MSC: Client Needs: Psychosocial Integrity

7. A person intentionally overdosed on antidepressants. Which nursing diagnosis has the highest priority? a. Powerlessness b. Social isolation c. Risk for suicide d. Compromised family coping

ANS: C This diagnosis is the only one with life-or-death ramifications and is therefore of higher priority than the other options. PTS: 1 DIF: Cognitive Level: Apply (Application) REF: Page 487-490 (Table 25-3) TOP: Nursing Process: Diagnosis/Analysis MSC: Client Needs: Psychosocial Integrity

13. A tearful, anxious patient at the outpatient clinic reports, I should be dead. The initial task of the nurse conducting the assessment interview is to: a. assess lethality of suicide plan. b. encourage expression of anger. c. establish rapport with the patient. d. determine risk factors for suicide.

ANS: C This scenario presents a potential crisis. Establishing rapport facilitates a therapeutic alliance that will allow the nurse to obtain relevant assessment data such as the presence of a suicide plan, lethality of plan, and presence of risk factors for suicide. PTS: 1 DIF: Cognitive Level: Analyze (Analysis) REF: Page 488-491 (Nursing Care Plan 25-1) TOP: Nursing Process: Implementation MSC: Client Needs: Psychosocial Integrity

17. A nurse assesses a patient who reports a 3-week history of depression and periods of uncontrolled crying. The patient says, My business is bankrupt, and I was served with divorce papers. Which subsequent statement by the patient alerts the nurse to a concealed suicidal message? a. I wish I were dead. b. Life is not worth living. c. I have a plan that will fix everything. d. My family will be better off without me.

ANS: C Verbal clues to suicide may be overt or covert. The incorrect options are overt references to suicide. The correct option is more veiled. It alludes to the patients suicide as being a way to fix everything but does not say it outright. PTS: 1 DIF: Cognitive Level: Apply (Application) REF: Page 485-486 | Page 490 (Table 25-3) TOP: Nursing Process: Assessment MSC: Client Needs: Psychosocial Integrity

A nurse and social worker co-lead a reminiscence group for eight "young-old" adults. Which activity is most appropriate to include in the group? a. Singing a song from World War II b. Learning how to join an online social network c. Discussing national leadership during the Vietnam War d. Identifying the most troubling story in today's newspaper

ANS: C "Young-old" adults are persons 65 to 74 years of age. These adults were attuned to conflicts in national leadership associated with the Vietnam War. Reminiscence groups share memories of the past. Learning how to join a social network would not be an aspect of reminiscence. Singing a song from World War II is more appropriate for an elite old reminiscence group. The other incorrect option is less relevant to this age group.

Which behavior shows that a nurse values autonomy? The nurse: a. suggests one-on-one supervision for a patient who has suicidal thoughts. b. informs a patient that the spouse will not be in during visiting hours. c. discusses options and helps the patient weigh the consequences. d. sets limits on a patient's romantic overtures toward the nurse.

ANS: C A high level of valuing is acting on one's belief. Autonomy is supported when the nurse helps a patient weigh alternatives and their consequences before the patient makes a decision. Autonomy or self-determination is not the issue in any of the other behaviors.

When making a distinction as to whether a patient is experiencing confusion related to depression or dementia, what information would be most important for the nurse to consider? a. The patient with dementia is persistently angry and hostile. b. Early morning agitation and hyperactivity occur in dementia. c. Confusion seems to worsen at night when dementia is present. d. A patient who is depressed is constantly preoccupied with somatic symptoms.

ANS: C Both dementia and depression in older adults may produce symptoms of confusion. Noting whether the confusion seems to increase at night, which occurs more often with dementia than with depression, will help distinguish whether depression or dementia is producing the confused behavior. The other options are not necessarily true.

What is the desirable outcome for the orientation stage of a nurse-patient relationship? The patient will demonstrate behaviors that indicate: a. self-responsibility and autonomy. c. rapport and trust with the nurse. b. a greater sense of independence. d. resolved transference.

ANS: C Development of rapport and trust is necessary before the relationship can progress to the working phase. Behaviors indicating a greater sense of independence, self-responsibility, and resolved transference occur in the working phase.

During which phase of the nurse-patient relationship can the nurse anticipate that identified patient issues will be explored and resolved? a. Preorientation c. Working b. Orientation d. Termination

ANS: C During the working phase, the nurse strives to assist the patient in making connections among dysfunctional behaviors, thinking, and emotions and offers support while alternative coping behaviors are tried.

A group is in the working phase. One member states, "That is the stupidest thing I've ever heard. Everyone whines and tells everyone else what to do. This group is a waste of my time." Which initial action by the group leader would be most therapeutic? a. Advise the member that hostility is inappropriate. Remove the member if it continues. b. Keep the group's focus on this member so the person can express the anger. c. Meet privately with the member outside of group to discuss the anger. d. Change to a more positive topic of discussion in this group session.

ANS: C Meeting privately with the member can convey interest and help defuse the anger so that it is less disruptive to the group. Removing the member would be a last resort and used only when the behavior is intolerably disruptive to the group process and all other interventions have failed. Decreasing the focus on the hostile member and focusing more on positive members can help soften the anger. Angry members often hide considerable vulnerability by using anger to keep others at a distance and intimidated. Changing the subject fails to respond to the behavior.

A soldier who served in a combat zone returned to the United States. The soldier's spouse complains to the nurse, "We had planned to start a family, but now he won't talk about it. He won't even look at children." The spouse is describing which symptom associated with posttraumatic stress disorder (PTSD)? a. Re-experiencing b. Hyperarousal c. Avoidance d. Psychosis

ANS: C Physiological reactions to reminders of the event include a persistent avoidance of the stimuli associated with the trauma; the individual avoids talking about the event or avoids activities, people, or places that arouse memories of the trauma. Avoidance is exemplified by a sense of foreshortened future and estrangement. No evidence suggests that this soldier is having a hyperarousal reaction or is re-experiencing war-related traumas. Psychosis is not evident.

After several therapeutic encounters with a patient who recently attempted suicide, which occurrence should cause the nurse to consider the possibility of countertransference? a. The patient's reactions toward the nurse seem realistic and appropriate. b. The patient states, "Talking to you feels like talking to my parents." c. The nurse feels unusually happy when the patient's mood begins to lift. d. The nurse develops a trusting relationship with the patient.

ANS: C Strong positive or negative reactions toward a patient or over-identification with the patient indicate possible countertransference. Nurses must carefully monitor their own feelings and reactions to detect countertransference and then seek supervision. Realistic and appropriate reactions from a patient toward a nurse are desirable. One incorrect response suggests transference. A trusting relationship with the patient is desirable. See relationship to audience response question.

Termination of a therapeutic nurse-patient relationship has been successful when the nurse: a. avoids upsetting the patient by shifting focus to other patients before the discharge. b. gives the patient a personal telephone number and permission to call after discharge. c. discusses with the patient changes that happened during the relationship and evaluates outcomes. d. offers to meet the patient for coffee and conversation three times a week after discharge.

ANS: C Summarizing and evaluating progress help validate the experience for the patient and the nurse and facilitate closure. Termination must be discussed; avoiding discussion by spending little time with the patient promotes feelings of abandonment. Successful termination requires that the relationship be brought to closure without the possibility of dependency-producing ongoing contact.

18. The nurse working in the role of educator with groups of clients may: a. Care for clients in acute care settings. b. Coordinate activities with staff members in community settings. c. Use learning principles to increase understanding about mental illness. d. Act as an advocate for mental health support groups.

ANS: C The educator role is foundational to health maintenance, health promotion, and community action.

A group is in the working phase. One member says, "That is the stupidest thing I've ever heard. Everyone whines and tells everyone else what to do. This group is a total waste of my time." Which comment by the group leader would be most therapeutic? a. "You seem to think you know a lot already. Since you know so much, perhaps you can tell everyone why you are back in the hospital?" b. "I think you have made your views clear, but I wonder if others feel the same way. How does everyone else feel about our group?" c. "It must be hard to be so angry." Direct this comment to another group member, "You were also angry at first but not now. What has helped you?" d. "I would like to remind you that one of our group rules is that everyone is to offer only positive responses to the comments of others."

ANS: C The member's comments demean the group and its members and suggest that the member is very angry. Labeling the emotion and conveying empathy would be therapeutic. Focusing on members who are likely to be more positive can balance the influence of demoralizing members. "You seem to know a lot..." conveys hostility from the leader, who confronts and challenges the member to explain how he came to be readmitted if he was so knowledgeable, implying that he is less knowledgeable than he claims. This comment suggests countertransference and is non-therapeutic. Shifting away from the complaining member to see if others agree seeks to have others express disagreement with this member, but that might not happen. In the face of his anger, they might be quiet or afraid to oppose him, or they could respond in kind by expressing hostility themselves. A rule that only positive exchanges are permitted would suppress conflict, reducing the effectiveness of the therapy group.

As a nurse escorts a patient being discharged after treatment for major depressive disorder, the patient gives the nurse a gold necklace with a heart pendant and says, "Thank you for helping mend my broken heart." Which is the nurse's best response? a. "Accepting gifts violates the policies and procedures of the facility." b. "I'm glad you feel so much better now. Thank you for the beautiful necklace." c. "I'm glad I could help you, but I can't accept the gift. My reward is seeing your renewed sense of hope." d. "Helping people is what nursing is all about. It's rewarding to me when patients recognize how hard we work."

ANS: C Accepting a gift creates a social rather than a therapeutic relationship with the patient and blurs the boundaries of the relationship. A caring nurse will acknowledge the patient's gesture of appreciation, but the gift should not be accepted.

During which phase of the nurse-patient relationship can the nurse anticipate that identified patient issues will be explored and resolved? a. Preorientation b. Orientation c. Working d. Termination

ANS: C During the working phase, the nurse strives to assist the patient in making connections among dysfunctional behaviors, thinking, and emotions and offers support while alternative coping behaviors are tried.

A patient says, "I'm still on restriction, but I want to attend some off-unit activities. Would you ask the doctor to change my privileges?" What is the nurse's best response? a. "Why are you asking me when you're able to speak for yourself?" b. "I will be glad to address it when I see your doctor later today." c. "That's a good topic for you to take up with your doctor." d. "Do you think you can't speak to a doctor?"

ANS: C Nurses should encourage patients to work at their optimal level of functioning. A nurse does not act for the patient unless it is necessary. Acting for a patient increases feelings of helplessness and dependency.

Which issues should a nurse address during the first interview with a patient diagnosed with a psychiatric disorder? a. Trust, congruence, attitudes, and boundaries b. Goals, resistance, unconscious motivations, and diversion c. Relationship parameters, the contract, confidentiality, and termination d. Transference, countertransference, intimacy, and developing resources

ANS: C Relationship parameters, the contract, confidentiality, and termination are issues that should be considered during the orientation phase of the relationship. The remaining options are issues that are dealt with later.

After several therapeutic encounters with a patient who recently attempted suicide, which occurrence should cause the nurse to consider the possibility of countertransference? a. The patient's reactions toward the nurse seem realistic and appropriate. b. The patient states, "Talking to you feels like talking to my parents." c. The nurse feels unusually happy when the patient's mood begins to lift. d. The nurse develops a trusting relationship with the patient.

ANS: C Strong positive or negative reactions toward a patient, or an overidentification with a patient signal possible countertransference. Nurses must carefully monitor their own feelings and reactions to detect countertransference and then seek supervision. Realistic and appropriate reactions from a patient toward a nurse are desirable. One incorrect response suggests transference. A trusting relationship with the patient is desirable.

Termination of a therapeutic nurse-patient relationship with a patient has been successful when nurse engages in what action? a. Avoids upsetting the patient by shifting focus to other patients before the discharge. b. Gives the patient a personal telephone number and permission to call after discharge. c. Discusses with the patient changes that have happened during the relationship and evaluates the outcomes. d. Offers to meet the patient for coffee and conversation three times a week after discharge.

ANS: C Summarizing and evaluating progress help validate the experience for the patient and the nurse and facilitate closure. Termination must be discussed; avoiding the discussion by spending little time with the patient promotes feelings of abandonment. Successful termination requires that the relationship be brought to closure without the possibility of dependency-producing ongoing contact.

A person with a fear of closes spaces enters into an elevator. Which division of the autonomic nervous system is stimulated in response to this experience? a. Limbic system b. Peripheral nervous system c. Sympathetic nervous system d. Parasympathetic nervous system

ANS: C The autonomic nervous system is made up of the sympathetic (fight-or-flight response) and parasympathetic (relaxation response) nervous systems. In times of stress, the sympathetic nervous system is stimulated. A person fearful of heights would experience stress associated with the experience of driving across a high bridge. The peripheral nervous system responds to messages from the sympathetic nervous system. The limbic system processes emotional responses but is not specifically part of the autonomic nervous system.

A person with a fear of heights drives across a high bridge. Which structure will stimulate a response from the autonomic nervous system? a. Thalamus b. Parietal lobe c. Hypothalamus d. Pituitary gland

ANS: C The individual will find this experience stressful. The hypothalamus functions as the command-and-control center when receiving stressful signals. The hypothalamus responds to signals of stress by engaging the autonomic nervous system. The parietal lobe is responsible for the interpretation of other sensations. The thalamus processes messages associated with pain and wakefulness. The pituitary gland may be involved in other aspects of the person's response but would not stimulate the autonomic nervous system.

Which experiences are most likely to precipitate posttraumatic stress disorder (PTSD)? (Select all that apply.) a. An 8-year-old child watches an R-rated movie with both parents. b. A young adult jumps from a bridge with a bungee cord with a best friend. c. An adolescent is kidnapped and held for 2 years in the home of a sexual predator. d. A passenger is in a bus that overturns on a sharp curve in the road, tumbling down an embankment. e. An adult is trapped for 3 hours at an angle in an elevator after a portion of the supporting cable breaks.

ANS: C, D, E PTSD usually follows a traumatic event that is outside the range of usual experience. Examples are childhood physical abuse, torture or kidnapping, military combat, sexual assault, and natural disasters such as floods, tornados, earthquakes, and tsunamis; human disasters such as a bus or elevator accident or crime-related events such being taken hostage are additional examples. The common element in these experiences is the individual's extraordinary helplessness or powerlessness in the face of such stressors. Bungee jumping by adolescents is part of the developmental task and might be frightening but in an exhilarating way rather than a harmful way. A child may be disturbed by an R-rated movie, but the presence of the parents would modify the experience in a positive way.

A nurse assesses the health status of veterans of the war in Afghanistan. Screening will be a priority for signs and symptoms of which health problems? (Select all that apply.) a. Schizophrenia b. Eating disorder c. Traumatic brain injury d. Seasonal affective disorder e. Posttraumatic stress disorder

ANS: C, E TBI and PTSD each occur in approximately 20% of soldiers who experienced combat. Some soldiers have both problems. The incidence of disorders identified in the distractors would be expected to parallel the general population.

The desired outcome for a patient experiencing insomnia is, "Patient will sleep for a minimum of 5 hours nightly within 7 days." At the end of 7 days, review of sleep data shows the patient sleeps an average of 4 hours nightly and takes a 2-hour afternoon nap. The nurse will document the outcome as: a. consistently demonstrated. b. often demonstrated. c. sometimes demonstrated. d. never demonstrated.

ANS: D Although the patient is sleeping 6 hours daily, the total is not one uninterrupted session at night. Therefore, the outcome must be evaluated as never demonstrated. See relationship to audience response question.

25. Which individual in the emergency department should be considered at highest risk for completing suicide? a. An adolescent Asian American girl with superior athletic and academic skills who has asthma b. A 38-year-old single, African American female church member with fibrocystic breast disease c. A 60-year-old married Hispanic man with twelve grandchildren who has type 2 diabetes d. A 79-year-old single, white male diagnosed recently with terminal cancer of the prostate

ANS: D High-risk factors include being an older adult, single, male, and having a co-occurring medical illness. Cancer is one of the somatic conditions associated with increased suicide risk. Protective factors for African American women and Hispanic individuals include strong religious and family ties. Asian Americans have a suicide rate that increases with age. PTS: 1 DIF: Cognitive Level: Analyze (Analysis) REF: Page 482-483 (Box 25-2) TOP: Nursing Process: Assessment MSC: Client Needs: Psychosocial Integrity

Nursing behaviors associated with the implementation phase of nursing process are concerned with: a. participating in mutual identification of patient outcomes. b. gathering accurate and sufficient patient-centered data. c. comparing patient responses and expected outcomes. d. carrying out interventions and coordinating care.

ANS: D Nursing behaviors relating to implementation include using available resources, performing interventions, finding alternatives when necessary, and coordinating care with other team members.

The desired outcome for a patient experiencing insomnia is, "Patient will sleep for a minimum of 5 hours nightly within 7 days." At the end of 7 days, review of sleep data shows the patient sleeps an average of 4 hours nightly and takes a 2-hour afternoon nap. What is the nurse's next action? a. Continue the current plan without changes. b. Remove this nursing diagnosis from the plan of care. c. Write a new nursing diagnosis that better reflects the problem. d. Examine interventions for possible revision of the target date.

ANS: D Sleeping a total of 5 hours at night remains a reasonable outcome. Extending the period for attaining the outcome may be appropriate. Examining interventions might result in planning an activity during the afternoon rather than permitting a nap. Continuing the current plan without changes is inappropriate. Removing this nursing diagnosis from the plan of care would be correct when the outcome was met and the problem resolved. Writing a new nursing diagnosis is inappropriate because no other nursing diagnosis relates to the problem.

8. A person who attempted suicide by overdose was treated in the emergency department and then hospitalized. The initial outcome is that the patient will: a. verbalize a will to live by the end of the second hospital day. b. describe two new coping mechanisms by the end of the third hospital day. c. accurately delineate personal strengths by the end of first week of hospitalization. d. exercise suicide self-restraint by refraining from attempts to harm self for 24 hours.

ANS: D Suicide self-restraint relates most directly to the priority problem of risk for self-directed violence. The other outcomes are related to hope, coping, and self-esteem. PTS: 1 DIF: Cognitive Level: Apply (Application) REF: Page 487 | Page 492 (Table 25-4) TOP: Nursing Process: Outcomes Identification MSC: Client Needs: Psychosocial Integrity

A nurse documents: "Patient is mute despite repeated efforts to elicit speech. Makes no eye contact. Inattentive to staff. Gazes off to the side or looks upward rather than at speaker." Which nursing diagnosis should be considered? a. Defensive coping b. Decisional conflict c. Risk for other-directed violence d. Impaired verbal communication

ANS: D The defining characteristics are more related to the nursing diagnosis of impaired verbal communication than to the other nursing diagnoses.

19. A nurse counsels a patient with recent suicidal ideation. Which is the nurses most therapeutic comment? a. Lets make a list of all your problems and think of solutions for each one. b. Im happy youre taking control of your problems and trying to find solutions. c. When you have bad feelings, try to focus on positive experiences from your life. d. Lets consider which problems are very important and which are less important.

ANS: D The nurse helps the patient develop effective coping skills. Assist the patient to reduce the overwhelming effects of problems by prioritizing them. The incorrect options continue to present overwhelming approaches to problem solving. PTS: 1 DIF: Cognitive Level: Apply (Application) REF: Page 484 | Page 488-489 (Nursing Care Plan 25-1) | Page 491-492 TOP: Nursing Process: Implementation MSC: Client Needs: Psychosocial Integrity

A nurse prepares to assess a new patient who moved to the United States from Central America three years ago. After introductions, what is the nurse's next comment? a. "How did you get to the United States?" b. "Would you like for a family member to help you talk with me?" c. "An interpreter is available. Would you like for me to make a request for these services?" d. "Are you comfortable conversing in English, or would you prefer to have a translator present?"

ANS: D The nurse should determine whether a translator is needed by first assessing the patient for language barriers. Accuracy of the assessment depends on the ability to communicate in a language that is familiar to the patient. Family members are not always reliable translators. An interpreter may change the patient's responses; a translator is a better resource.

A patient states, "I'm not worth anything. I have negative thoughts about myself. I feel anxious and shaky all the time. Sometimes I feel so sad that I want to go to sleep and never wake up." Which nursing intervention should have the highest priority? a. Self-esteem-building activities b. Anxiety self-control measures c. Sleep enhancement activitie d. Suicide precautions

ANS: D The nurse would place a priority on monitoring and reinforcing suicide self-restraint because it relates directly and immediately to patient safety. Patient safety is always a priority concern. The nurse should monitor and reinforce all patient attempts to control anxiety, improve sleep patterns, and develop self-esteem, while giving priority attention to suicide self-restraint.

Select the best outcome for a patient with the nursing diagnosis: Impaired social interaction related to sociocultural dissonance as evidenced by stating, "Although I'd like to, I don't join in because I don't speak the language very well." Patient will: a. show improved use of language. b. demonstrate improved social skills. c. become more independent in decision making. d. select and participate in one group activity per day.

ANS: D The outcome describes social involvement on the part of the patient. Neither cooperation nor independence has been an issue. The patient has already expressed a desire to interact with others. Outcomes must be measurable. Two of the distracters are not measurable.

Which statement made by a patient during an initial assessment interview should serve as the priority focus for the plan of care? a. "I can always trust my family." b. "It seems like I always have bad luck." c. "You never know who will turn against you." d. "I hear evil voices that tell me to do bad things."

ANS: D The statement regarding evil voices tells the nurse that the patient is experiencing auditory hallucinations and may create risks for violence. The other statements are vague and do not clearly identify the patient's chief symptom.

2. Four individuals have given information about their suicide plans. Which plan evidences the highest suicide risk? a. Turning on the oven and letting gas escape into the apartment during the night b. Cutting the wrists in the bathroom while the spouse reads in the next room c. Overdosing on aspirin with codeine while the spouse is out with friends d. Jumping from a railroad bridge located in a deserted area late at night

ANS: D This is a highly lethal method with little opportunity for rescue. The other options are lower lethality methods with higher rescue potential. See relationship to audience response question. PTS: 1 DIF: Cognitive Level: Analyze (Analysis) REF: Page 486-487 TOP: Nursing Process: Assessment MSC: Client Needs: Psychosocial Integrity

3. Which measure would be considered a form of primary prevention for suicide? a. Psychiatric hospitalization of a suicidal patient b. Referral of a formerly suicidal patient to a support group c. Suicide precautions for 24 hours for newly admitted patients d. Helping school children learn to manage stress and be resilient

ANS: D This measure promotes effective coping and reduces the likelihood that such children will become suicidal later in life. Admissions and suicide precautions are secondary prevention measures. Support group referral is a tertiary prevention measure. PTS: 1 DIF: Cognitive Level: Understand (Comprehension) REF: Page 487-488 TOP: Nursing Process: Implementation MSC: Client Needs: Safe, Effective Care Environment

At what point in an assessment interview would a nurse ask, "How does your faith help you in stressful situations?" During the assessment of: a. childhood growth and development b. substance use and abuse c. educational background d. coping strategies

ANS: D When discussing coping strategies, the nurse might ask what the patient does when upset, what usually relieves stress, and to whom the patient goes to talk about problems. The question regarding whether the patient's faith helps deal with stress fits well here. It would be out of place if introduced during exploration of the other topics.

A nurse assesses an older adult patient brought to the emergency department by a family member. The patient was wandering outside saying, "I can't find my way home." The patient is confused and unable to answer questions. Select the nurse's best action. a. Record the patient's answers to questions on the nursing assessment form. b. Ask an advanced practice nurse to perform the assessment interview. c. Call for a mental health advocate to maintain the patient's rights. d. Obtain important information from the family member.

ANS: D When the patient (primary source) is unable to provide information, secondary sources should be used, in this case, the family member. Later, more data may be obtained from other information sources familiar with the patient. An advanced practice nurse is not needed for this assessment; it is within the scope of practice of the staff nurse. Calling a mental health advocate is unnecessary. See relationship to audience response question.

An advance directive gives valid direction to health care providers when a patient is: a. aggressive. b. dehydrated. c. unable to verbally communicate. d. unable to make decisions for himself or herself.

ANS: D Advance directives are invoked when patients are unable to make their own decisions. Aggression, dehydration, or an inability to speak does not mean the patient is unable to make a decision.

An older adult patient brings a bag of medication to the clinic. The nurse finds one bottle labeled "Ativan" and one labeled "lorazepam," and both are labeled "Take two times daily." Bottles of hydrochlorothiazide, Inderal, and rofecoxib, each labeled "Take one daily," are also included. Which conclusion is accurate? a. Rofecoxib should not be taken with Ativan. b. The patient's blood pressure is likely to be very high. c. This patient should not self-administer any medication. d. Lorazepam and Ativan are the same drug; consequently, the dose is excessive.

ANS: D Lorazepam and Ativan are generic and trade names for the same drug, creating an accidental overdose situation. The patient needs medication education and help with proper, consistent labeling of bottles. No evidence suggests that the patient is unable to self-administer medication. The distractors are not factual statements.

An older adult patient diagnosed with major depressive disorder is being treated with sertraline (Zoloft). This medication is often chosen for older adult patients because it: a. has a high degree of sedation. b. is effective when given in smaller doses. c. has few adverse interactions with other drugs. d. is less affected by changes associated with aging.

ANS: D Older adults are particularly susceptible to side effects, so selecting a drug with a low side-effect profile is desirable. The pharmacokinetics of sertraline are less affected by changes associated with aging. The other options are either incorrect or of lesser relevance

Which statement about aging provides the best rationale for focused assessment of older adult patients? a. Older adults are often socially isolated and lonely. b. As people age, they become more rigid in their thinking. c. The majority of older adults sleep more than 12 hours per day. d. The senses of vision, hearing, touch, taste, and smell decline with age.

ANS: D Only the correct answer is true and cues the nurse to assess carefully the sensory functions of the older adult patient. The incorrect options are myths about aging.

Recognizing the risk for acquired immunodeficiency syndrome (AIDS) among older adults, nurses should provide health teaching aimed at: a. discouraging sexual expression. b. using birth control measures. c. avoiding blood transfusions. d. encouraging condom use.

ANS: D Safe sex continues to be important and should be taught to the older adult population. Because the risk for pregnancy is nonexistent in postmenopausal women, condom use is diminished, which places older adults at risk for AIDS and other sexually transmitted diseases. Sexual expression is a basic human need. Little to no danger exists from blood transfusions

Which remark by a patient indicates passage from orientation to the working phase of a nurse-patient relationship? a. "I don't have any problems." b. "It is so difficult for me to talk about problems." c. "I don't know how it will help to talk to you about my problems." d. "I want to find a way to deal with my anger without becoming violent."

ANS: D Thinking about a more constructive approach to dealing with anger indicates a readiness to make a behavioral change. Behavioral change is associated with the working phase of the relationship. Denial is often seen in the orientation phase. It is common early in the relationship, before rapport and trust are firmly established, for a patient to express difficulty in talking about problems. Stating skepticism about the effectiveness of the nurse-patient relationship is more typically a reaction during the orientation phase.

A nurse is talking with a patient, and 5 minutes remain in the session. The patient has been silent most of the session. Another patient comes to the door of the room, interrupts, and says to the nurse, "I really need to talk to you." The nurse should: a. invite the interrupting patient to join in the session with the current patient. b. say to the interrupting patient, "I am not available to talk with you at the present time." c. end the unproductive session with the current patient and spend time with the interrupting patient. d. tell the interrupting patient, "This session is 5 more minutes; then I will talk with you."

ANS: D When a specific duration for sessions has been set, the nurse must adhere to the schedule. Leaving the first patient would be equivalent to abandonment and would destroy any trust the patient had in the nurse. Adhering to the contract demonstrates that the nurse can be trusted and that the patient and the sessions are important. The incorrect responses preserve the nurse-patient relationship with the silent patient but may seem abrupt to the interrupting patient, abandon the silent patient, or fail to observe the contract with the silent patient.

Which behavior shows that a nurse values autonomy? a. Setting limits on a patient's romantic overtures toward the nurse b. Suggesting one-on-one supervision for a patient who is suicidal c. Informing a patient that the spouse will not be in during visiting hours d. Helping the patient weigh the consequences of their behaviors and decisions

ANS: D A high level of valuing is acting on one's belief. Autonomy is supported when the nurse helps the patient weigh alternatives and their consequences before the patient makes a decision. Autonomy or self-determination is not the issue in any of the other behaviors.

A soldier returned 1 year ago from Afghanistan and was diagnosed with posttraumatic stress disorder (PTSD). Which social event would most likely be disturbing for this soldier? a. Halloween festival with neighborhood children b. Singing carols around a Christmas tree c. Family outing to the seashore d. Fireworks display on July 4th

ANS: D Exploding noises associated with fireworks are most likely to provoke exaggerated responses for this soldier. The distractors are not associated with offensive sounds.

A soldier served in combat zones in Iraq in 2010 and was deployed to Afghanistan in 2014. When is it most important for the nurse to screen for signs and symptoms of posttraumatic stress disorder (PTSD)? a. Immediately upon return to the United States from Afghanistan b. Before departing Afghanistan to return to the United States c. Two years after returning from Afghanistan d. Screening should be ongoing

ANS: D PTSD can have a long lag time—months to years. Screening should be ongoing.

A soldier returned home last year after deployment to a war zone. The soldier's spouse reports, "We were going to start a family but now he won't talk about it. He will not look at children. I wonder if we're going to make it as a couple." What response best addresses the spouse's concerns? a. "Posttraumatic stress disorder often changes a person's sexual functioning." b. "I encourage you to continue to participate in social activities where children are present." c. "Have you talked with your spouse about these reactions? Sometimes we just need to confront behavior." d. "Posttraumatic stress disorder often strains relationships. I will suggest some community resources for help and support."

ANS: D Posttraumatic stress disorder (PSTD) precipitates changes that often lead to divorce. Providing support to both the veteran and spouse is important. Confrontation will not be effective. Although providing information is important, ongoing support is more effective.

A family member asks the nurse, "Do you think stress and physical illness are connected? Since my father's death, my mother has had shingles and the flu, but she's usually not one who gets sick." Which answer by the nurse best reflects current knowledge about long term effects of stress? a. "It is probably a coincidence. Emotions and physical responses travel on different tracts of the nervous system." b. "You may be paying more attention to your mother since your father died and noticing more things such as minor illnesses." c. "So far, research on emotions or stress and becoming ill more easily is unclear. We do not know for sure if there is a link." d. "Negative emotions and stress may interfere with the body's ability to protect itself and can increase the likelihood of infection."

ANS: D The correct answer best explains the research. Research supports a link between negative emotions and/or prolonged stress and impaired immune system functioning. Activation of the immune system signals the central nervous system to initiate myriad responses to stress. Prolonged stress suppresses the immune system and lowers resistance to infections.

During the first interview, a nurse notices that the patient does not make eye contact. What can the nurse correctly assume from this behavior? a. The patient is not truthful. b. The patient is feeling sad. c. The patient has a poor self-concept. d. The need for more information is required to draw a conclusion.

ANS: D The data are insufficient to draw a conclusion. The nurse must continue to assess.

A nurse caring for a withdrawn, suspicious patient recognizes the development of feelings of anger toward the patient. How should the nurse respond? a. By suppressing the angry feelings. b. By expressing the anger openly and directly with the patient. c. By telling the nurse manager to assign the patient to another nurse. d. By discussing the anger with a clinician during a supervisory session.

ANS: D The nurse is accountable for the relationship. Objectivity is threatened by strong positive or negative feelings toward a patient. Supervision is necessary to work through a countertransference of feelings.

A patient is brought to the emergency department after a motorcycle accident. The patient is alert, responsive, and diagnosed with a broken leg. The patient's vital signs are temperature (T), 98.6° F; pulse (P), 72 beats/min (bpm); and respirations (R), 16 breaths per minute. After being informed that surgery is required for the broken leg, which vital sign readings would be expected? a. T, 98.6°; P, 64; R, 14 b. T, 98.6°; P, 68; R, 12 c. T, 98.6°; P, 62; R, 16 d. T, 98.6°; P, 84; R, 22

ANS: D The patient would experience stress associated with the anticipation of surgery. In times of stress, the sympathetic nervous system takes over (fight-or-flight response) and sends signals to the adrenal glands, thereby releasing epinephrine. The circulating epinephrine increases the heart rate. Respirations increase, bringing more oxygen to the lungs.

Which remark by a patient indicates passage from the orientation phase to the working phase of a nurse-patient relationship? a. "I don't have any problems." b. "It is so difficult for me to talk about my problems." c. "I don't know how talking about things twice a week can help." d. "I want to find a way to deal with my anger without becoming violent."

ANS: D Thinking about a more constructive approach to dealing with anger indicates a readiness to make a behavioral change. Behavioral change is associated with the working phase of the relationship. Denial is often seen in the orientation phase. It is common early in the relationship, before rapport and trust are firmly established, for a patient to express difficulty in talking about problems. Stating skepticism about the effectiveness of the nurse-patient relationship is more typically a reaction during the orientation phase.

A patient says, "People should be allowed to commit suicide without interference from others." A nurse replies, "You're wrong. Nothing is bad enough to justify death." What is the best analysis of this interchange? a. The patient is correct. b. The nurse is correct. c. Neither person is totally correct. d. Differing values are reflected in the two statements.

ANS: D Values guide beliefs and actions. The individuals stating their positions place different values on life and autonomy. Nurses must be aware of their own values and be sensitive to the values of others.

A nurse is talking with a patient, and 5 minutes remain in the session. The patient has been silent for most of the session. Another patient comes to the door of the room, interrupts, and says to the nurse, "I really need to talk to you right now." What action is most appropriate? a. Saying to the interrupting patient, "I am not available to talk with you at the present time." b. Ending the unproductive session with the current patient and spend time with the patient who has just interrupted. c. Inviting the interrupting patient to join in the session with the current patient. d. Telling the patient who interrupted, "This session will end in 5 minutes; then, I will talk with you."

ANS: D When a specific duration for a session has been set, the nurse must adhere to the schedule. Leaving the first patient would be equivalent to abandonment and would destroy any trust the patient had in the nurse. Adhering to the contract demonstrates that the nurse can be trusted and that the patient and the sessions are important. The incorrect responses preserve the nurse-patient relationship with the silent patient but may seem abrupt to the interrupting patient, abandon the silent patient, or fail to observe the contract with the silent patient.

The nurse records this entry in a patient's progress notes: Patient escorted to unit by ER nurse at 2130. Patient's clothing was dirty. In interview room, patient sat with hands over face, sobbing softly. Did not acknowledge nurse or reply to questions. After several minutes, abruptly arose, ran to window, and pounded. Shouted repeatedly, "Let me out of here." Verbal intervention unsuccessful. Order for stat dose 2 mg haloperidol PO obtained; medication administered at 2150. By 2215, patient stopped shouting and returned to sit wordlessly in chair. Patient placed on one-to-one observation. How should this documentation be evaluated? a. Uses unapproved abbreviations b. Contains subjective material c. Too brief to be of value d. Excessively wordy e. Meets standards

ANS: E This narrative note describes patient appearance, behavior, and conversation. It mentions that less-restrictive measures were attempted before administering medication and documents patient response to medication. This note would probably meet standards. A complete nursing assessment would be in order as soon as the patient is able to participate. Subjective material is absent from the note. Abbreviations are acceptable.

A psychiatric client is participating in a partial hospitalization program (PHP). What is a typical client schedule in the PHP? Half day Monday through Friday About 6 hours Monday through Friday Half day each on Saturday and Sunday About 3 to 4 hours each on Saturday and Sunday

About 6 hours Monday through Friday The typical client schedule for PHP is Monday through Friday for about 6 hours, because they are "partially hospitalized." Half day Monday through Friday is a typical schedule for intensive outpatient programs (IOPs). Programs are usually done Monday through Friday and exclude weekend days. p. 54

An adolescent comes to the crisis clinic and reports sexual abuse by an uncle. The adolescent told both parents about the uncle's behavior, but the parents did not believe the adolescent. What type of crisis exists a.Maturational b.Adventitious c.Situational d.Organic

Adventitious

According to Freud, a client experiencing dysfunction of the conscious as part of the mind will have problems with which aspect of memory?

All memories

When treating mental illnesses with psychotropic drugs what is the focus of the treatment?

Altering brain neurochemistry

What is the inability to recognize the need for care due to disorganized thinking called? Stigma Denial Anosognosia Abstinence

Anosognosia Anosognosia means the inability to recognize the need for care due to disorganized thinking. Stigma is a negative perception of individuals with mental illness. Denial is the lack of acceptance or willingness to understand or deal with a problem. Abstinence is refraining from something. Test-Taking Tip: Multiple choice questions can be challenging, because students think they will recognize the right answer when they see it or that the right answer will somehow stand out from the other choices. This is a dangerous misconception. The more carefully the question is constructed, the more each of the choices will seem like the correct response. p. 52

The role of a case manager working with severely and persistently mentally ill clients who are homeless would include which intervention? A. Coordinate needed services B. Administer medication C. Teach the clients to function independently D. Ensure that the clients are not re-hospitalized

Answer: A Community mental health services are designed to provide outreach and case management for severely mentally ill persons who are homeless. A team approach is used to gain access to clients and connect them with the various services available to meet their needs. The role of the outreach worker is to be an advocate in all areas of client need and to foster client self-care. The role of the case manager does not include any of the other options.

Which statement is true of the relationship between serious mental illnesses (SMIs) and substance abuse? A. Of those diagnosed with SMI, substance abuse is high B. Substance abuse occurs at approximately the same rate as in the general population C. Substance abuse rarely occurs within this population D. Smoking has declined in this population at the same rate as the general public

Answer: A Comorbid substance abuse occurs in 30% of those with SMI. It may be a form of self-medication, countering the dysphoria or other symptoms caused by illness or its treatment (e.g., the sedation caused by one's medications) or a maladaptive response to boredom. Nicotine use has always been higher in the population of those with SMI and is not declining as it has been in the general population. Substance abuse contributes to comorbid physical health problems, reduced quality of life, incarceration, relapse, and reduced effectiveness of medications. Substance abuse in those with SMI is higher than in the general population. Smoking has not declined in this population at the same rate as for the general public.

The term dual diagnosis refers to having a severe mental illness and what other dysfunctional behavior? A. A substance abuse problem B. Medication noncompliance C. HIV infection D. A personality disorder

Answer: A Dual diagnosis is the term used to identify a client with severe mental illness and a substance abuse problem. Both problems must be treated if the client is to be successfully rehabilitated. None of the other options reflect an accurate description of the term dual diagnosis.

Serious mental illness (SMI) affects how many adults in the United States? A. 11 million B. 8 Million C. 4 Million D. 1 million

Answer: A SMI affects about 11.4 million adults in the United States. The other options are incorrect percentages.

The clinical nurse specialist should suggest which cognitive intervention initially for a client experiencing auditory hallucinations? A. Initiating a distracting technique B. Giving as-needed medication anxiety C. Seclusion when escalation begins D. Physical restraints when the client is disruptive

Answer: A Strategies have been successfully applied to treat hallucinations, delusions, and negative symptoms, making cognitive interventions an evidence-based practice. For example, distraction techniques can be taught when auditory hallucinations occur, such as listening to music or humming. The remaining options should only be considered when less restrictive interventions, like distraction, prove to be ineffective and the client is at risk for injury to self or to others.

Which interventions and/or goals related to planning for discharge of a client diagnosed with a serious mental illness (SMI) would support the recovery model of care? A. Interventions will focus on the client's stated wish for independent living B. Care plan interventions will focus on medication adherence C. The clients' parents will receive education on how to manage the patient's deficits D. Attending groups that teach how to cope with one's present illness

Answer: A The recovery model is patient centered, instills hope and empowerment, emphasizes the person and the future, encourages independence and self-determination, and focuses on achieving goals of the patient's choosing and meaningful living. The National Alliance on Mental Illness (NAMI) and the President's New Freedom Commission on Mental Health (2003) both support the recovery model of care rather than the rehabilitation model, which focuses on the illness and the present. The other options all follow the rehabilitation model, focusing on the illness.

An issue for severely and persistently mentally ill clients living in the community is inadequate long-term medication monitoring by community mental health workers. What is a remedy for this problem? A. Use client empowerment techniques to increase client autonomy B. Shift follow-up from social workers to the ACT model C. Develop tools to predict relapse and assess the potential for violence D. Discontinue antipsychotics that cause untoward side effects

Answer: B Adequate monitoring of medication effects by the community-based health care provider is often difficult but more achievable when the client is being monitored by the assertive community treatment (ACT) model. None of the other options present an effective remedy for this problem.

Which statement best reflects the way clients who are severely and persistently mentally ill generally perceive how others in the community see them? A. A large number are intensely hostile toward others B. Many feel stigmatized and alienated C. Most feel under-supported by family and friends D. The majority are incapable of such self-reflective thought

Answer: B Studies have shown that many clients experience stigmatization, alienation, loss of relationships, and loss of vocational opportunities. While some clients may have the perceptions described in the other options, none are as generally expressed as feeling stigmatized and alienated.

A client who has been prescribed an antipsychotic medication comes to the clinic 3 days after a scheduled visit and demonstrates evidence of restlessness and agitation. He states, "My medicine ran out, and I didn't remember where to get more." The client's case manager should initially implement which intervention to support medication adherence? A. Arrange for the client to get to the nearest emergency department for treatment B. Arrange for a dose of the client's medication immediately C. Arrange to have the client's nursing care plan reflect the need for a medication change D. Arrange for the client to see his psychiatrist as soon as the psychiatrist has an open appointment

Answer: B The role of the case manager is to coordinate access to psychiatric treatment, housing, rehabilitation or work setting, socialization, and medical care. The client's immediate need for medication is best addressed by arranging for an immediate dose. None of the other options addresses the client's needs as effectively.

A 20-year-old Amish client was diagnosed with paranoid schizophrenia 1 year ago who lives with his parents. When the nurse attempts to educate him about his diagnosis and the need for medication, the client persistently mumbles, "I don't have mental illness. No, I am not sick." What term is used to describe this response? A. Religiosity B. Apathy C. Anosognosia D. Resistance

Answer: C Anosognosia is the inability to recognize one's deficits as a result of one's illness. In serious mental illnesses (SMIs), the brain, the organ one needs to have insight and make good decisions, is the organ that is diseased. An illness that makes one unable to recognize that illness can understandably cause one to be resistant to treatment. Although the patient may be resistive to treatment, it does not best describe the patient's denial of the illness. Apathy is lack of caring. Nothing in the scenario depicts the patient being preoccupied with religion at this time.

Which statement by a young client diagnosed with a severe and persistent mental illness would alert the nurse to the need for psychoeducational intervention? A. "I hate having my thought so messed up all the time." B. "I am looking for a job washing dishes at a diner." C. "I hear that marijuana helps calm you down." D. "I like to watch cartoons every morning."

Answer: C Clients with mental illness should receive information about the dangerous negative impact of using illegal drugs. None of the other options suggest situations that are unsafe for the client.

Severely mentally ill (SMI) clients often express a strong desire to be employed. According to the evidence-based research, what is the most effective model of employment for these clients? A. Vocational rehabilitation B. A placement program of rehabilitation C. Supported employment D. Productive employment

Answer: C In the past, vocational rehabilitation programs required extensive evaluation procedures and training before attempting job placement. However, these programs were unsuccessful at helping severely mentally ill clients to maintain jobs. Research efforts have identified a more productive model called supported employment.

Institutionalization leads to what specific type of behaviors in adults old enough to have been confined to institutions before deinstitutionalization? A. Assertiveness and condor B. Fearfulness and paranoia C. Passivity and dependance D. Anger and aggression

Answer: C Medical paternalism, in which the health care provider made all decisions for patients with serious mental illnesses (SMIs), was pervasive at the time of common institutionalization for mental illness. As a result, patients became dependent on the services and structure of institutions and unable to function independently outside such institutions. It was difficult to distinguish whether behaviors such as regression were the result of the illness or institutionalization. The other options are incorrect regarding the common resulting behavior of institutionalized patients.

A client diagnosed with a severe and persistent mental illness tells the case manager, "I think people are laughing at me behind my back. I get real upset and anxious when I have to be around others in the group home. It's better when I just stay by myself." The nurse should consider which nursing diagnosis to address the client's concerns? A. Acute confusion B. Risk for activity intolerance C. Social isolation D. Impaired comfort

Answer: C Social isolation is aloneness experienced by the individual and perceived as imposed by others. None of the other options would be supported by the information provided in the question.

The goal of a nurse working in psychiatric rehabilitation would be to help clients in the community achieve which outcome? A. Live comfortably in a psychiatric treatment facility B. Learn to live with dependency C. Cope more effectively with their symptoms D. Complete mental health

Answer: C The long-term outcomes of rehabilitation for severely mentally ill clients include the concepts of illness management and recovery. Illness management refers to the focus in the early stage of treatment that assists the client to gain control over symptoms. Clients are taught to collaborate with professionals in mental health treatment, reduce susceptibility to relapse, and cope more effectively with symptoms. Complete mental health is not always achievable.

The mother of a client with severe, persistent schizophrenia tells the nurse, "My child has slipped so far away from me over the past few years. We really don't have a relationship anymore. I miss my child terribly." The nursing diagnosis that best describes the mother's feelings using which term? A. Ineffective coping B. Caregiver role strain C. Grieving D. Powerlessness

Answer: C The mother is mourning the loss of her son as she formerly knew him. Grief is a common experience for families with mentally ill members. The statement does not support any of the other options.

The nurse working with a client diagnosed with severe and persistent mental illness will implement rehabilitation principles by concentrating on which intervention? A. Considering the need to lower expectations periodically B. Reviewing earlier treatment plans for errors C. Assessment on the client's deficits D. Reinforcing the client's strengths

Answer: D Although deficits are assessed and addressed, implementation of rehabilitation is dependent on reinforcement of identified client strengths. None of the remaining options is fundamental to the rehab process.

Which remark would signal to the nurse that there is a teaching need for the family of a client diagnosed with schizophrenia? A. "We have taught him to use the bus so we do not have to drive him everywhere." B. "We give positive recognition to him whenever he does even simple things well." C. "We watch him closely of signs of illness associated with relapse." D. "We always reprimand him whenever his behavior is bothersome."

Answer: D An important need of families caring for the severely and persistently mentally ill is psychoeducation to help them understand the disease process. Families need to be prepared to meet the many concerns related to safety, communication, medication compliance, and symptom management. Family interventions are now considered an evidence-based practice, with research showing improved outcomes of decreased relapses and rehospitalizations for clients whose families participate. The only negative approach to the client and his/her needs is reflected in the option that suggests reprimanding when bothersome.

rapport

As in any relationship, rapport can be nurtured by demonstrating genuineness and empathy, developing positive regard, showing consistency, and offering assistance in problem solving and in providing support

Using Maslow's model of needs, the nurse providing care for an anxious client identifies which intervention as being a priority?

Assessing the client for strengths upon which a nurse-client relationship can be based

A 15-year-old is hospitalized after a suicide attempt. This adolescent lives with the mother, stepfather, and several siblings. When performing a family assessment, the nurse must first determine: a. how the family expresses and manages emotion. b. names and relationships of the family's members. c. the communication patterns between the patient and parents. d. the meaning that the patient's suicide attempt has for family members.

B

When a nurse assesses a family, which family task has the highest priority for healthy family functioning? a. Allocation of family resources b. Physical maintenance and safety c. Maintenance of order and authority d. Reproduction of new family members

B

Which scenario best illustrates scapegoating within a family? a. The identified patient sends messages of aggression to selected family members. b. Family members project problems of the family onto one particular family member. c. The identified patient threatens separation from the family to induce feelings of isolation and despair. d. Family members give the identified patient nonverbal messages that conflict with verbal messages.

B

The nurse who provides therapeutic milieu management supports the clients best by concentrating on which client need? A) Opportunity to act out fears and frustrations B) Providing a safe place to practice coping skills C) Meeting their physical as well as emotional needs D) Encouraging group communication about existing problems

B A therapeutic milieu can serve as a real-life training ground for learning about the self and practicing communication and coping skills in preparation for a return to the community. The other options are considered components of a therapeutic milieu.

What is the primary advantage of using a case manager when considering the planning and implementation of client care? A) Increases collaborative practice. B) Enhances resource management. C) Increases client satisfaction with care. D) Promotes evidence-based psychiatric nursing.

B Case management coordinates and monitors the effectiveness of services appropriate for the client. While the other options are true statements, none describes the primary advantage of the case manager model of health care delivery.

A client was admitted to the behavioral health unit for evaluation and diagnosis after being found wandering the streets. His personal hygiene is poor, and his responses to questions are bizarre and inappropriate. The client's constitutional rights are violated when the nurse makes which statement? A) "We will help you make decisions that will keep you safe." B) "I am going to help you shower, so you will not smell so bad." C) "Your pocket knife and nail clippers will be kept in the nurses' station." D) "You will be having a number of tests to help us learn about your condition."

B Every client has the right to be treated with dignity. This statement is demeaning. All of the other statements support the client's rights.

When a nurse says, "I work with a mobile mental health unit," what assumption can a client accurately make about the care being provided? A) The patients who are convicted criminals sentenced to home confinement. B) Care is provided to clients in unconventional settings. C) Care is provided by a preferred provider for a large HMO. D) The patients are provided for by a clinical specialist with the visiting nurse service.

B Mobile mental health units travel throughout the community, seeing clients on their own "turf," such as in shelters, on street corners, in homes, and at factories.

Which of the following patients would be appropriate to refer to a partial hospitalization program (PHP)? A) A depressed patient with a suicidal plan B) A patient being discharged from an inpatient alcohol rehabilitation unit C) A client who has stopped taking his or her antipsychotic medication and is neglecting his or her basic needs D) Jeff, who has mild depression symptoms and is starting outpatient therapy

B PHP is for patients who may need a "step-down" environment from inpatient status or for those who are being diverted from hospitalization with intensive, short-term care from which they return home each day. This patient would be a good candidate after completing alcohol rehab; PHP could possibly help prevent relapse in the early stages after rehab. This patient can be managed with regular outpatient therapy and does not need intensive short-term therapy such as PHP. Someone who is suicidal would require inpatient hospitalization for safety as would someone who is decompensated and not caring for herself. A patient exhibiting mild depression would be managed with outpatient therapy and would not need intensive short-term therapy such as PHP.

Ian makes the following statements to you while admitting him. Which statement indicates and increased likelihood of violent behavior? A. "When I get mad, I want to be left alone" B. "Last time I was in here I ended up in seclusion for punching my roommate C. "My old man was meek and mild, and I've always said I'm not going to be like him" D. "My girlfriend says I yell way too much, and she's threatened to leave me."

B. "Last time I was in here I ended up in seclusion for punching my roommate

A nurse is assessing a client in an inpatient mental health unit. Which of the following findings should the nurse expect if the client is in the preassaultive stage of violence? (select all that apply) A. Lethargy B. Defensive responses to questions C. Disorientation D. Facial grimacing E. Agitation

B. Defensive responses to questions D. Facial grimacing E. Agitation

When you approach Katy, what considerations should you take? A. Stand close to Katy for reassurance and to convey caring. B. Have other staff as backup, and stand far enough away to avoid injury. C. Take Katy to her room so you can speak with her alone. D. Call security and wait until they arrive before approaching Katy.

B. Have other staff as backup, and stand far enough away to avoid injury. Safety considerations for staff include enlisting other staff to be present, keeping a safe distance from the patient, and approaching the patient in a nonthreatening or nonconfrontational manner. The other options do not allow for staff safety; security personnel may escalate the patient's behavior and should be kept in the background until needed to assist. Furthermore, the patient has an immediate need to be assisted by staff if possible without waiting for security.Cognitive Level: Analyze (Analysis)Nursing Process: PlanningNCLEX: Safe and Effective Care EnvironmentText page: 520

Which neurotransmitter imbalance has been shown to be related to impulsive aggression? A. Low levels of ã-aminobutyric acid B. Low levels of serotonin C. High levels of dopamine D. High levels of acetylcholine

B. Low levels of serotonin Low serotonin levels have been implicated in several research studies as being a factor in impulsive aggression.REF: 516

A nurse is caring for a client who is speaking in a loud voice with clenched fists. Which of the following actions should the nurse take? A. Insist that the client stop yelling B. Request that other staff members remain close by C. Move as close to the client as possible D. Walk away from the client

B. Request that other staff members remain close by

Which nursing diagnosis is the priority when planning care for a client who displays considerable anger and occasional aggression? A. Social isolation B. Risk for other-directed violence C. Ineffective coping: overwhelmed D. Ineffective coping: maladaptive

B. Risk for other-directed violence Risk for other-directed violence is the priority diagnosis. The nurse then must determine which of two other diagnoses—ineffective coping: overwhelmed or ineffective coping: maladaptive—is appropriate. Social isolation is not an initial concern.REF: 519

An angry client frequently loses patience with the nurses and shouts at them while they perform a complicated dressing change. Which plan could they create to intervene effectively in this behavior? A. Tell him they will not change his dressing if he is going to abuse them. B. When the client begins to become abusive, leave the room promising to return in 20 minutes when he has regained control. C. Assure him they will complete the dressing change as quickly as possible. D. Explain that they are professionals and unused to being shouted at by people they are trying to help.

B. When the client begins to become abusive, leave the room promising to return in 20 minutes when he has regained control. The nurse is using behavioral techniques to reinforce desirable behavior (spending time with the client when he is calm) and limit reinforcement of undesirable behavior (leaving when he is acting out anger).REF: Page 521, 525

When a client diagnosed with a cognitive deficit experiences a catastrophic reaction, the priority intervention is to A. decrease sensory stimuli. B. smile and call the client by name. C. take the client to the bathroom. D. calmly ask the client what's wrong. b

B. smile and call the client by name. Getting the client's attention by calling his or her name is necessary. Smiling is necessary to convey the lack of a threat.REF: Page 527

One older concept that is being used currently that may help in violence reduction in patients is: A. aired grievances. B. trauma-informed care. C. shared governance. D. learned helplessness.

B. trauma-informed care.' Trauma-informed care is an older concept of providing care that has been reintroduced. It is based on the notion that disruptive patients often have histories that include violence and victimization. These traumatic histories can impede patients' ability to self-soothe, result in negative coping responses, and create a vulnerability to coercive interventions (e.g., restraint) by staff. Trauma-informed care focuses on the patients' past experiences of violence or trauma and the role it currently plays in their lives. The other options do not refer to a care concept that helps reduce violence.Cognitive Level: Understand (Comprehension)Nursing Process: PlanningNCLEX: Psychosocial IntegrityText page: 517

What is the premise underlying behavioral therapy?

Behavior is learned and can be modified

Select the best question for the nurse to ask to assess a family's ability to cope. a. "What strengths does your family have?" b. "Do you think your family copes effectively?" c. "Describe how you successfully handled one family problem." d. "How do you think the current family problem should be resolved?"

C

Which of the following patients meets the criteria for an involuntary admission to a psychiatric mental health unit? A) A 23-year-old college student who has developed symptoms of anxiety and is missing classes and work B) A 30-year-old accountant who has developed symptoms of depression C) A 26-year-old kindergarten teacher who is not in touch with reality and was found wandering in and out of traffic on a busy road D) A 76-year-old retired librarian who is experiencing memory loss and some confusion at times

C Inpatient involuntary admission is reserved for patients who are at risk for self-harm or who cannot adequately protect themselves from harm because of their illness (e.g., a psychotic patient). The other options can all be managed at this point in the community setting and don't meet criteria (risk of harm to self and/or others) for admission.

Which criterion must be met to refer a client to a partial hospitalization program? A) The client is hospitalized at night in an inpatient setting. B) The client must be able to provide his or her own transportation daily. C) The client is able to return home each day. D) The clients are all recovering from an addiction.

C Returning home each day is a criterion because doing so allows the person to test out new skills and gradually re-enter the family and society. None of the remaining options are true statements regarding partial hospitalization programs.

The psychiatric community health nurse engages in secondary prevention when implementing which intervention? A) Visiting a homeless shelter to provide mental health screenings for its clients B) Discussing the need for proper nutrition with a depressed new mother C) Providing stress reduction seminars at the local senior center D) Visiting the home of a client currently displaying manic behavior

C Secondary prevention is aimed at reducing the prevalence of psychiatric disorders. Early identification of problems, screening, and prompt and effective treatment are hallmarks of this level. While it does not stop the actual disorder from beginning, it is intended to delay or avert progression. None of the other options are focused on early identification of problems.

You are caring for Malcolm, an 83 y/o African American patient with Alzheimer's disease. Malcolm exhibits agitated behavior at times, especially when he feels he is missing work, and he sometimes attempts to leave the unit to " get to the school where I teach. " which of the following interventions is appropriate for de-escalating Malcolm's agitation. A. Medicate Malcolm with PRN medication at regular intervals to prevent agitation B. Repeatedly explain to Malcolm that he is retired and no longer teaches as the repetition will reinforce the patient's orientation C . Use validation therapy and ask Malcolm about the school and his job D. Reduce stimulation In the environment by having Malcolm sit by himself in his room until the agitation passes

C . Use validation therapy and ask Malcolm about the school and his job

Which statementt about violence and nursing is accurate? A. Unless working in psychiatric mental health settings, nurses are unlikely to experience patient violence B. To date, no legislation exists that addresses workplace violence against nurses C. Emergency, psychiatric, and step-down units have the highest rates of violence towards staff D. Violence primarily affects inexperienced or unskilled staff who cannot calm their patients

C. Emergency, psychiatric, and step-down units have the highest rates of violence towards staff

A nurse is caring for a client in an inpatient mental health facility who gets up from a chair and throws it across the day room. Which of the following is the priority nursing action? A. Encourage the client to express her feelings B. Maintain eye contact with the client C. Move the client away from others D. Tell the client that the behavior is not acceptable

C. Move the client away from others

The factor most likely to contribute to a client's escalating anger is A. watching violence on television. B. another client's depressed mood. C. a staff member telling him that he is inappropriate. D. a staff member asking him to help another client.

C. a staff member telling him that he is inappropriate. Punitive, threatening, accusatory, or challenging statements to the client should be avoided; rather, the nurse should determine what is behind the client's feelings and behaviors.REF: 520

The client at highest risk for violence directed at others is one who A. has a history of recurrent severe depression. B. is in an alcohol rehabilitation program. C. has delusions of persecution. D. who has somatic symptoms for which no organic basis is found.

C. has delusions of persecution. The client who perceives others to be against him may lash out if he feels threatened.REF: Page 518

When working with an angry client, it is best to A. encourage the client to fully explore and express his or her anger. B. help the client deny and repress the feelings of anger C. help the client reframe the anger-producing situation. D. ignore the client's anger and change the subject.

C. help the client reframe the anger-producing situation. De-escalation occurs more quickly with this strategy than when other approaches are used.REF: 518-519

An adolescent male is swearing and shouting at his physician, who refused to give him a pass to leave the unit. This behavior A. is acceptable if directed at staff but not when directed at other clients. B. may reduce tension and prevent the client from physically acting out. C. is a major indicator that the client may become physically aggressive. D. can be attributed to lack of parental controls applied at an early age.

C. is a major indicator that the client may become physically aggressive. Physical aggression is preceded by anger, which may be expressed by swearing and shouting, pacing, and other menacing behaviors.REF: Page 520

A nurse attempts to intervene verbally when an angry client initially threatens to throw a chair but quickly focuses the anger toward the nurse. Several staff members gather behind the nurse, but then the client shouts, "I will calm down when that nurse isn't in my face." The nurse best demonstrates the ability to help the client deescalate by A. continuing to manage the situation personally. B. telling the client, "It isn't safe for me to leave the room." C. moving to the rear of the staff group. D. apologizing for upsetting the client.

C. moving to the rear of the staff group. There is no need for the nurse to stand her ground to save face. The goal is to deescalate the situation. When the client makes a request that can be met without compromising safety, granting the request is acceptable.REF: Page 520-521

Peter, a 21-year-old patient, asks you, "What's wrong with my brain that I have such a problem with aggression?" Your response is based on the knowledge that: A. the prevailing theory is that diminishment of stress hormones causes anger and aggression. B. no abnormalities of the brain have been identified that correlate with anger and aggression. C. the limbic system, the prefrontal cortex, and neurotransmitters have been implicated in playing a part in aggression. D. personality type plays a much greater part in anger and aggression than physical factors.

C. the limbic system, the prefrontal cortex, and neurotransmitters have been implicated in playing a part in aggression. These have all been implicated by research as playing a part in anger and aggression. The other responses are untrue.Cognitive Level: Apply (Application)Nursing Process: ImplementationNCLEX: Psychosocial IntegrityText page: 516

An emergency department nurse prepares to assist with examination of a sexual assault victim. What equipment will be needed to collect and document forensic evidence?

Camera Body map DNA swabs

A woman said, "I can't take anymore! Last year my husband had an affair, and now we don't communicate. Three months ago, I found a lump in my breast. Yesterday my daughter said she's quitting college." What is the nurse's priority assessment a.Identify measures useful to help improve the couple's communication. b.The patient's feelings about the possibility of having a mastectomy c.Whether the husband is still engaged in an extramarital affair d.Clarify what the patient means by "I can't take anymore."

Clarify what the patient means by "I can't take anymore."

In what is the client participating when attending group therapy to enhance coping skills? Cognitive behavior therapy Occupational therapy Physical therapy Recreational therapy

Cognitive behavior therapy Coping skills are taught and enhanced by participating in cognitive behavioral groups that focus on symptom management. Occupational therapy provides an opportunity to practice life skills that may have been delayed or altered. Physical therapy focuses on physical conditioning and mobility. Recreational therapy activities are used to improve emotional, physical, cognitive, and social well-being. p. 56

What is a function of a neuron?

Conduction of electrical impulses

An emergency department nurse prepares to assist with evidence collection for a sexual assault victim. Prior to photographs and pelvic examination, what documentation is important?

Consent signed by the patient Patients have the right to refuse legal and medical examination. Consent forms are required to proceed with these steps.

A woman says, "I can't take anymore. Last year my husband had an affair, and now we do not communicate. Three months ago, I found a lump in my breast. Yesterday my daughter said she's quitting college and moving in with her boyfriend." Which issue should the nurse focus on during crisis intervention a.The possible mastectomy b.The disordered family communication c.The effects of the husband's extramarital affair d.Coping with the reaction to the daughter's events

Coping with the reaction to the daughter's events

A family expresses helplessness related to dealing with a mentally ill member's odd behaviors, mood swings, and argumentativeness. An effective nursing intervention for this family would be to: a. express sympathy for their situation. b. involve local social service agencies. c. explain symptoms of relapse. d. role-play difficult situations

D

Parents of a teenager recently diagnosed with serious mental illness express dismay. One parent says, "Our child acts so strangely that we don't invite friends to our home. We quit taking vacations. Sometimes we don't get any sleep." Which nursing diagnosis best applies? a. Impaired parenting b. Dysfunctional grieving c. Impaired social interaction d. Interrupted family processes

D

It is not always guaranteed that all clients who are voluntarily admitted to a behavioral health unit will have the right to which privilege? A) Refusal of treatment. B) To send and receive mail. C) To seek legal counsel. D) To access all personal possessions.

D A client has the right to keep personal belongings unless they are dangerous. Items such as sharp objects, glass containers, and medication are usually removed from the client's possession and kept in a locked area to be used by the client under supervision or returned at discharge. The remaining options are civil rights afforded to all clients.

The primary goal and benefit of assertive community treatment (ACT) is demonstrated by which situation? A) A client and family members attend counseling sessions together at a neighborhood clinic B) Implementation of a more flexible work schedule for staff C) Improved reimbursement for services provided in the community D) A client diagnosed with schizophrenia has avoided being rehospitalization for 16 months.

D A primary goal of ACT is working intensely with the patient in the community to prevent rehospitalization. The other options are not goals of ACT.

In order to be most effective, the community mental health nurse involved in assertive community treatment (ACT) needs to possess which characteristic? A) Knowledge of both national and local political activism B) The ability to cross service systems C) An awareness of own cultural and personal values D) Creative problem-solving and intervention skills

D Creative problem-solving and intervention skills are the hallmark of care provided by the ACT team.

Which situation demonstrates the nurse functioning in the role of advocate? A) Providing one-to-one supervision for a client on suicide precautions B) Co-leading a medication education group for clients and families C) Attending an in-service education program to obtain recertification in cardiopulmonary resuscitation D) Negotiating with the client's HMO for extension of a 3-day hospitalization to 5 days

D In the inpatient setting, case managers on the hospital team communicate daily or weekly with the client's insurer and provide the treatment team guidance regarding the availability of resources. In the community, multiple levels of intervention are available within case management service, ranging from daily assistance with medications to ongoing resolution of housing and financial issues.

What function is shared by advanced practice and general practice psychiatric nurses? A) Prescriptive authority B) Admitting privileges C) Offers consultation services D) Membership on a multidisciplinary team

D Nurses at both levels are expected to collaborate with multidisciplinary teams; only the advanced practice nurse has prescriptive authority and admitting privileges and can provide consultation.

You are working in the emergency department. You notice Matt, your patient's husband, pacing in the hallway, muttering to himself, and looking angrily around the emergency department. Which of the following statements to Matt may help prevent escalation and/or violence? A. "You need to stay with your wife. She needs you." B. "Hey, what's up buddy? You look pissed." C."I am calling security to deal with your behavior." D. You appear upset. Can I help you with anything?"

D. "You appear upset. Can I help you with anything?" Approaching a patient or a visitor with a calm, sincere, and caring manner can de-escalate a situation because the person may feel you are interested in helping. The other responses will not prevent escalation and may in fact anger the person further.Cognitive Level: Analyze (Analysis)Nursing Process: ImplementationNCLEX: Safe and Effective Care EnvironmentText page: 520

A nurse is conducting group therapy with a group of clients. Which of the following statements made by a client is an example of aggressive communication? A. "I wish you would not make me angry" B. "I feel angry when you leave me." C. "It makes me angry when you interrupt me." D. "You'd better listen to me"

D. "You'd better listen to me"

Anger can best be defined as A. an unhealthy way of releasing anxiety. B. doing intentional harm to others. C. an expression of conflict with others. D. a normal response to a perceived threat.

D. a normal response to a perceived threat.

A client waiting to see the physician is pacing and looking both angry and tense. When it's determined that the client won't be seen for another 30 minutes, the nurse addresses the client's agitation by A. telling the client that pacing will not help the rate at which clients are seen. B. adjusting the appointment schedule to allow the client to be seen next. C. empathizing with the long wait and asking the client if he would mind sitting down until his turn comes. D. explaining to the client what caused the back-up and suggesting that he has time to go to the coffee shop.

D. explaining to the client what caused the back-up and suggesting that he has time to go to the coffee shop. Taking time to explain to clients and offering measures that will provide comfort can be helpful in reducing tension and anger associated with waiting.REF: Page 521

A client experiencing manic hyperactivity stands up, glares challengingly at clients and staff, and shouts, "This food is garbage! I'll fight anyone who says it's not!" The nurse's most relevant assessment is that the client A. is upset with the quality of the food. B. is getting rid of tension in a harmless way. C. is frustrated by limits imposed by hospitalization. D.has a high potential for other-directed violence.

D. has a high potential for other-directed violence. The client's offers to fight are suggestive of a high potential for violence. Clients may have coping skills that are adequate for day-to-day events in their lives but are overwhelmed by the stresses of illness or hospitalization. Other clients may have a pattern of maladaptive coping, which is marginally effective and consists of a set of coping strategies that have been developed to meet unusual or extraordinary situations.REF: Page 516-517

A client has a history of demonstrating aggression physically. An appropriate short-term goal to help the client manage this anger is to A. strike objects rather than people. B. limit aggression to verbal outbursts. C. isolate in lieu of striking people. D. identify situations that precipitate hostility.

D. identify situations that precipitate hostility. The identification of situations that create hostile feelings must occur if the client is to develop new coping strategies.REF: Page 527

A client has been placed in seclusion to control aggressive behavior. Care while the client is secluded should include A. observation every 30 minutes. B. releasing the client every 8 hours. C. releasing the client every 8 hours. D. providing for nutrition and hydration.

D. providing for nutrition and hydration. Clients must be given meals on schedule and frequently offered cold liquids in paper cups (at least every 2 hours; hourly if the client is highly hyperactive).REF: Page 521

Nurses coping with angry clients may find it helpful to remember that anger and aggression begin as feelings of A. isolation. B. confidence. C. competence. D. vulnerability.

D. vulnerability. The progression is vulnerability, perception of event as a threat, arousal, and then uneasiness and anxiety.REF: 526

Freud believed that individuals cope with anxiety by implementing which mechanism?

Defense mechanisms

A nurse interviews a patient abducted and raped at gunpoint by an unknown assailant. The patient says, "I can't talk about it. Nothing happened. I have to forget." What is the patient's present coping strategy?

Denial The patient's statements reflect use of denial, an ego defense mechanism. This mechanism may be used unconsciously to protect the person from the emotionally overwhelming reality of the rape. The patient's statements do not reflect somatization, compensation, or projection.

A troubled adolescent pulled out a gun in a school cafeteria, fatally shooting three people and injuring many others. Hundreds of parents come to the school after hearing news reports. After police arrest the shooter, which action should occur next a.Ask police to encircle the school campus with yellow tape to prevent parents from entering. b.Announce over the loudspeakers, "The campus is now secure. Please return to your classrooms." c.Require parents to pass through metal detectors and then allow them to look for their children in the school. d.Designate zones according to the alphabet and direct students to the zones based on their surnames to facilitate reuniting them with their parents.

Designate zones according to the alphabet and direct students to the zones based on their surnames to facilitate reuniting them with their parents.

What responsibilities does the psychiatric mental health registered nurse carry out? Develop, implement, and evaluate plans of care Maintain oversight of restraint and seclusion Coordinate care by the treatment team Gather data and identify the psychiatric diagnosis Prescribe psychotropic medications for clients Monitor behavior, affect, and mood

Develop, implement, and evaluate plans of care Maintain oversight of restraint and seclusion Coordinate care by the treatment team Monitor behavior, affect, and mood The responsibilities of the psychiatric mental health registered nurse include developing, implementing, and evaluating plans of care; maintaining oversight of restraint use and seclusion; coordinating care provided for the client by the treatment team; and monitoring behavior, affect, and mood. Psychiatric mental health registered nurses do not diagnose the mental health condition but do identify the nursing diagnosis. Psychiatric mental health registered nurses do not prescribe psychotropic drugs; they do monitor and report drug effectiveness to health care providers. p. 57

When an emergency department nurse teaches a victim of rape-trauma syndrome about reactions that may occur during the long-term phase of reorganization, which symptoms should be included?

Development of fears and phobias Feelings of numbness Flashbacks, dreams These reactions are common to the long-term phase. Victims of rape frequently have a period of increased motor activity rather than decreased motor activity during the long-term reorganization phase. Syncopal episodes would not be expected.

A client experiencing an exacerbation of schizophrenia with psychotic features is reluctant to seek help despite family encouragement. What is the most common reason a client may not seek help for a mental illness? Lack of financial stability results in refusal of care by a health care provider. Disorganized thoughts impede the ability to recognize the need for care. Lack of family support results in the client's need for independence. The client is afraid of accumulating a large hospital bill.

Disorganized thoughts impede the ability to recognize the need for care. A client with a mental illness such as schizophrenia with psychotic features often experiences disorganized thinking that impedes the ability to recognize the need for care. Lack of financial stability is unlikely to cause a health care provider not to treat the client. Nonprofit hospitals accept any client regardless of the ability to pay. There is no indication that the client has lack of family support or is fearful of owing money for health care. p. 52

Which client problem would be most suited to the use of interpersonal therapy?

Dysfunctional grieving

Empathy

Empathy is a complex multidimensional concept that has moral, cognitive, emotional, and behavioral components.

The nurse is planning care for a 14-year-old. The nurse demonstrates an understanding of the developmental task appropriate for this client by providing which experience?

Encouraging them to talk about their school plans to help achieve identity

A nursing diagnosis for a client with a psychiatric disorder serves what purpose with considering the plan of care?

Establishing a framework for selecting appropriate interventions

The nurse planning care for a mentally ill client bases interventions on which concept?

Every client has a certain degree of resilience

The nurse is determining discharge living arrangements for a hospitalized mental health client. Which behavior demonstrated by the client would fulfill requirements for financial reimbursement for placement in a psychiatric home environment? Expresses paranoia regarding police Engages in numerous compulsive rituals Experiences both auditory and visual hallucinations Experiences panic attacks when among strangers outside of the home

Experiences panic attacks when among strangers outside of the home To qualify for reimbursement, clients must have a psychiatric diagnosis, be under the care of a primary care practitioner, and be homebound. Because the client experiences panic attacks outside of the home, he or she may be homebound and therefore qualify for financial reimbursement. Paranoia regarding police, compulsive rituals, and auditory and visual hallucinations do not necessarily qualify a client for financial reimbursement in a psychiatric home environment. p. 54

The nurse is writing discharge instructions of a mental health client. What information should discharge instructions include? Select all that apply. Follow-up appointments Medication instructions Education and prescriptions Instructions on arranging temporary housing Physician's home phone number

Follow-up appointments Medication instructions Education and prescriptions Discharge instructions for clients with mental illness should include follow-up appointments, medication instructions, and education and prescriptions. The social worker should make housing arrangements for the client prior to discharge. The nurse should never give clients the physician's home phone number without permission from the physician. p. 56

As Election Day nears, a mental health nurse studies the position statements of various candidates for federal offices. Which candidate's plan would the nurse interpret as most supportive of services for clients diagnosed with mental illness? Full-parity insurance coverage for mental illness Coverage for biologically based mental illnesses Reimbursement for initial treatment of addictions Managed care oversight for mental illness services

Full-parity insurance coverage for mental illness Full-parity insurance coverage ensures payment for mental health disorders equal to that of physical disorders; this is most supportive of services for clients diagnosed with mental illness. Coverage for biologically based mental illnesses is less supportive because it excludes mental health disorders that may not have a biological basis. Reimbursement for initial treatment of addictions limits coverage to a specific type of mental health disorder and for a limited amount of time. Managed care oversight for mental illness services does not ensure parity of coverage. p. 58

Genuineness

Genuineness, or self-awareness of one's feelings as they arise within the relationship and the ability to communicate them when appropriate, is a key ingredient in building trust

Which communication technique will the nurse use more in crisis intervention than traditional counseling a.Role modeling b.Giving direction c.Information giving d.Empathic listening

Giving direction

What is the most common method of inpatient suicide? Drowning Self-inflicted gunshot Hanging Cutting wrists

Hanging The most common method of inpatient suicide is hanging. Drowning is unlikely in the health care setting. Clients are searched for weapons such as knives or firearms upon entry into the organization, so suicide by gunshot or cutting wrists is unlikely. p. 56

Narcissism

Having to find weakness, helplessness, and/or disease in patients to feel helpful, at the expense of recognizing and supporting patients' healthier, stronger, and more competent features

How can a nurse best differentiate whether an Asian client is demonstrating a mental illness after having attempted suicide?

Identify the client's culture's view regarding suicide

The nurse providing anticipatory operant conditioning guidance to the mother of a toddler should advise that childhood temper tantrums are best handled by which intervention?

Ignoring the tantrum and giving attention when the child acts appropriately

A patient experienced sudden cardiac death (SCD) while hospitalized and survived. The nurse expects that what preventive treatment will be prescribed? External pacemaker An electrophysiologic study (EPS) Medications to prevent dysrhythmias Implantable cardioverter-defibrillator (ICD)

Implantable cardioverter-defibrillator (ICD) An ICD is the most common approach to preventing recurrence of SCD. An external pacemaker may be used in the hospital, but will not be used for the patient living daily life at home. An EPS may be done to determine if a recurrence is likely and determine the most effective medication treatment. Medications to prevent dysrhythmias are used, but are not best for prevention of SCD. p. 733

A nurse expresses an exclusive belief in the biological model for mental illness when stating "it's the only one I really believe." What conclusion should be drawn from this statement?

In believing only in the biological model, other influences on mental health including cultural, environmental, social, and spiritual influences are not taken into account.

When discussing the trend of treating mental health clients in community care environments, the nurse identifies which treatment-related event as the trigger for the shift away from traditional hospitalization? Increase in available psychopharmacological agents Increase in voluntary commitments to traditional hospital settings Increased diagnosis of clients with serious and persistent mental illnesses Increase in Medicare and Medicaid coverage for clients in psychiatric hospitals

Increase in available psychopharmacological agents The treatment-related event that triggered the shift away from traditional hospitalization was the increased availability of psychopharmacological agents. Gradually, more psychopharmacological agents were added to treat psychosis, depression, anxiety, and other disorders. The treatment of mental illness expanded from specialists in psychiatry to general practitioners. Increased voluntary commitment to traditional hospital settings did not occur. There was no increase in clients being diagnosed with mental illness. Medicare and Medicaid did not provide coverage for clients in psychiatric hospitals during this time. Test-Taking Tip: Read the question carefully before looking at the answers: (1) Determine what the question is really asking, and look for key words; (2) read each answer thoroughly, and see if it completely covers the material the question asks; and (3) narrow the choices by immediately eliminating answers you know are incorrect. p. 53

A student falsely accused a college professor of sexual intimidation. The professor tells the nurse, "I cannot teach nor do any research. My mind is totally preoccupied with these false accusations." What is the priority nursing diagnosis a.Ineffective denial related to threats to professional identity b.Deficient knowledge related to sexual harassment protocols c.Impaired social interaction related to loss of teaching abilities d.Ineffective role performance related to distress from false accusations

Ineffective role performance related to distress from false accusations

Which client care service provides structure in which clients eat meals, receive medication, attend activities, and participate in individual and group therapies on a schedule? Psychiatric rehabilitation services Psychiatric home care Intensive outpatient programs Inpatient care

Inpatient care Inpatient care provides structure, in which clients eat meals, receive medications (if necessary), attend activities, and participate in individual and group therapies on a schedule. Psychiatric rehabilitation services focus on the end goal of integration into society and include coordinated care from a variety of disciplines. Psychiatric home care can be provided by any medical health professional, but typically by nurses with previous inpatient care experience. Intensive outpatient programs function as intermediate programs between inpatient and outpatient care by primary health care providers, nurses, and social workers. Test-Taking Tip: Start by reading each of the answer options carefully. Usually at least one of them will be clearly wrong. Eliminate this one from consideration. Now you have reduced the number of response choices by one and improved the odds. Continue to analyze the options. If you can eliminate one more choice in a four-option question, you have reduced the odds to 50/50. While you are eliminating the wrong choices, recall often occurs. One of the options may serve as a trigger that causes you to remember what a few seconds ago had seemed completely forgotten. p. 56

A client presents to the clinic and states, "I've been on heroin for the past 5 years. It has destroyed my life. I want to quit but I get very sick when I go cold turkey. Can you help me?" Which plan of care would most meet this client's needs? 12-step program at the local church Drug counseling and education Inpatient care for detoxification Self-help group

Inpatient care for detoxification To begin recovery from heroin addition, the client needs to be admitted to a detoxification unit to stabilize physical symptoms during withdrawal. The 12-step program is an outpatient group therapy approach that is helpful in maintaining sobriety. Drug counseling and education should be done once the client undergoes detoxification. Self-help groups are not appropriate for the client who needs to stop taking heroin in a controlled and safe environment. Test-Taking Tip: Key words or phrases in the question stem such as first, primary, early, or best are important. Similarly, words such as only, always, never, and all in the alternatives are frequently evidence of a wrong response. No real absolutes exist in life; however, every rule has its exceptions, so answer with care. p. 58

A pediatric psychiatric registered nurse is explaining treatment options to the parents of a 12-year-old child. Which options are available for pediatric psychiatric care? Inpatient pediatric psychiatric care Outpatient pediatric psychiatric care Pediatric residential treatment centers Geriatric psychiatric care Veteran's administration centers Forensic psychiatric care

Inpatient pediatric psychiatric care Outpatient pediatric psychiatric care Pediatric residential treatment centers A child in need of psychiatric care has the same options as adults; however, they take place in a pediatric setting. Pediatric psychiatric care includes inpatient services, outpatient services, and residential treatment centers. Geriatric centers are specialized for the care of elderly psychiatric clients. Veteran's administration centers manage clients who are mentally affected by their military experiences. Forensic psychiatric care takes place in the prison system. p. 57

Intimate distance

Intimate distance in the United States is 0 to 18 inches and is reserved for those we trust most and with whom we feel most safe

The nurse educator is teaching a group of student nurses about the 1999 Olmstead decision. What was the main outcome of the Olmstead decision? It declared psychiatric clients must be deemed incompetent to receive care. It suggested that psychiatric clients be mandated into residential treatment centers. It deemed that mental institutions were safe and effective treatment options. It described mental illness as a disability and institutionalization as a violation of the client's rights.

It described mental illness as a disability and institutionalization as a violation of the client's rights. In the 1999 Olmstead decision, the Supreme Court ruled that institutionalizing clients with mental illness was in violation of the Americans with Disabilities Act. Institutionalization was described as "unjustified isolation." Clients who are declared incompetent receive care despite objections. The Olmstead decision did not mandate clients into residential treatment centers or describe mental institutions as safe and effective treatment options. Test-Taking Tip: Multiple choice questions can be challenging, because students think they will recognize the right answer when they see it or that the right answer will somehow stand out from the other choices. This is a dangerous misconception. The more carefully the question is constructed, the more each of the choices will seem like the correct response. p. 53

Which statement best describes the Diagnostic and Statistical Manual, fifth edition DSM-5?

It is a medical psychiatric assessment system

The nurse is caring for an adult client who experienced severe physical abuse from age of 2 through 12. What information should the nurse provide the client concerning the function of the "Id" and the ability to function as an adult?

It is the source of one's survival instincts

A nurse is providing care to a 28-year-old patient diagnosed with bipolar disorder who admitted in a manic state. According to Maslow's Hierarchy of Needs theory, the nurse should identify which patient symptom as having priority?

Lack of sleep

The community health nurse plays an important role in prevention and management of mental illness. What service is the community health nurse most likely to perform? Conducting data analysis on individual clients for diagnosis of mental illness Leading an educational session for a community group on stress-reduction techniques Obtaining and recording vital signs on a group of school-aged children with attention-deficit/hyperactivity disorder (ADHD) Perform lice screenings for school-age children within the community

Leading an educational session for a community group on stress-reduction techniques The community health nurse's role in mental health care is focused on prevention and management of mental illness. The community is the nurse's client. As such, leading an educational session on stress-reduction techniques for a community group is an example of community nursing care to prevent or manage stress. Conducting data analysis on individual clients for mental illness is the role of a mental health registered nurse. Obtaining and recording vital signs on a group of school-aged children with ADHD is the role of a school nurse or pediatric nurse specialist. Performing lice screenings is done within school systems and at individual physicians' offices. p. 54

A prescription for which medication would require the nurse to monitor the client for potential development of the side effect of hypothyroidism?

Lithium

Identify safety considerations for inpatient room design on a psychiatric care unit. Strong, sturdy closet rods and hooks Hand-held sprinkler showerheads Locked windows Removal of shoestrings Platform beds

Locked windows Removal of shoestrings Platform beds Safety considerations for inpatient psychiatric rooms include locked windows to deter jumping, removal of shoestrings and other devices that could be used for hanging, and platform beds to prevent possible crushing. Closet rods and hooks, towel bars, and shower rods are constructed to break if subjected to more than minimal weight to prevent hanging. Flush-mounted sprinkler and showerheads are used rather than hand-held sprinklers that could possibly be fashioned into a hanging device. Test-Taking Tip: After you have eliminated one or more choices, you may discover that two of the options are very similar. This can be helpful, because it may mean that one of these look-alike answers is the best choice and the other is a very good distractor. Test both of these options against the stem. Ask yourself which one completes the incomplete statement grammatically and which one answers the question more fully and completely. The option that best completes or answers the stem is the one you should choose. Here, too, pause for a few seconds, give your brain time to reflect, and recall may occur. pp. 54, 56

After celebrating the fortieth birthday, an individual becomes concerned with the loss of youthful appearance. What type of crisis has occurred a.Reactive b.Situational c.Maturational d.Adventitious

Maturational

The nurse admits a client experiencing hallucinations and delusional thinking to an inpatient mental health unit. The plan of care will require which service to occur first? Social history Psychiatric history Medical assessment Psychological evaluation

Medical assessment Medical assessment is the first step of inpatient care to first rule out comorbid conditions. Social history and psychiatric history should be taken after the client has been deemed physiologically stable. The appropriate member of the treatment team should conduct a full psychological evaluation after admission. p. 56

Role-playing is associated with which type of psychotherapy?

Modeling

A 49-year-old patient diagnosed with schizophrenia at 22 years old is prescribed risperidone. Which nursing assessment is the priority for this patient?

Monitoring for abnormal involuntary movements

What is the difference between intensive outpatient programs (IOPs) and partial hospitalization programs (PHPs)? Only IOPs function as intermediate steps between inpatient and outpatient care. More time is spent with clients in PHPs than in IOPs. Only PHPs are located within hospitals. Clients are more closely monitored for relapse in IOPs than in PHPs.

More time is spent with clients in PHPs than in IOPs. The difference between IOPs and PHPs is the amount of time spent with the clients. Both groups tend to be Monday through Friday. IOPs are usually half a day, while PHPs are longer (about 6 hours per day). Both function as intermediate steps between inpatient and outpatient care. Both are usually located within general hospitals, psychiatric hospitals, or in community settings. Clients are closely monitored for relapse in both programs. p. 54

Which agency provides coordination in the event of a terrorist attack a.Food and Drug Administration (FDA) b.Environmental Protection Agency (EPA) c.National Incident Management System (NIMS) d.Federal Emergency Management Agency (FEMA)

National Incident Management System (NIMS)

What does the concept of "least restrictive environment" mean? Mobility is discouraged, and the client is restrained. Restraints are only used with the client's permission. Necessary care is provided while permitting personal freedom. Necessary care is provided in the outpatient setting only.

Necessary care is provided while permitting personal freedom. The "least restrictive environment" means the client receives necessary care while being allowed the greatest personal freedom. Mobility is not discouraged; rather, the focus and intent are on client safety. As such, a client may be restrained for safety and continually assessed for readiness to remove them. Client permission is not required for restraint use. The "least restrictive environment" addresses clients in an inpatient setting, such as a hospital, or an outpatient setting, such as a nursing home. 52

An insulin-dependent client with diabetes presents to the emergency room confused, incoherent, and hallucinating. What is the priority assessment for this client? Determine the client's mental health history. Place the client in physical restraints for safety. Obtain a finger stick glucose reading. Consult the on-call psychiatrist.

Obtain a finger stick glucose reading. The priority assessment in this situation is to assess the client's glucose level. The client's signs and symptoms could be due to hypoglycemia. Physical problems are to be ruled out before mental health issues are addressed. The client's mental health history is not a priority. A client should never be placed in restraints unless there is threat of harm to the self or others and lesser methods have not been effective. The psychiatrist should not be consulted until a physical problem is ruled out. p. 56

One treatment goal for a hospitalized client is to be discharged to a residential treatment environment. Which priority intervention will the nurse include in the client's plan of care? Art therapy to reduce the effects of the illness Recreational therapy to improve social well-being Physical therapy to address any existing musculoskeletal disabilities Occupational therapy to assist in assuming skills needed to regain independence

Occupational therapy to assist in assuming skills needed to regain independence Residential treatment programs are structured short- or long-term 24-hour living environments in which individuals are provided with varying levels of supervision and support. Because hospitalization may lessen a client's independence, occupational therapy to improve the client's ability to perform activities of daily living will be a priority in this client's plan of care. Art therapy, recreational therapy, and physical therapy may be necessary as well but these would be of lower priority than occupational therapy to reestablish independence. Test-Taking Tip: You have at least a 25% chance of selecting the correct response in multiple choice items. If you are uncertain about a question, eliminate the choices you believe are wrong, and then call on your knowledge, skills, and abilities to choose from the remaining responses. p. 56

A client is visiting the primary health care provider (PCP). The client complains of difficulty sleeping. He states, "Those snakes on the floor scare me. There are so many! My daughter says there are no snakes, but she is trying to get my money." What are the disadvantages of mental illness being treated by PCPs? PCP appointment time constraints Complicated assessments when psychiatric disorders accompany physical complaints Limited PCP training in psychiatry Greater comfort in being treated in a familiar setting PCPs lacking expertise in diagnosis and treatment of psychiatric disorders Minimal stigma attached to receiving psychiatric care from PCPs

PCP appointment time constraints Limited PCP training in psychiatry PCPs lacking expertise in diagnosis and treatment of psychiatric disorders Disadvantages of being treated for mental illness by the PCP include appointment time constraints, limited training in psychiatry, and lack of expertise in the diagnosis and treatment of psychiatric disorders. Appointments with the PCP are usually scheduled in 15-minute increments, not permitting enough time for a detailed psychiatric assessment. Psychiatric disorders are often accompanied by physical complaints, but the PCP may overemphasize the physical over the psychiatric. PCPs may have limited training in psychiatry and lack expertise required to diagnose and manage these disorders. Benefits of PCP care are that the client may feel greater comfort in receiving treatment in a familiar setting, and the degree of stigma attached to mental illness might be minimized. p. 53

In order to reduce fragmented care and improve services, patient-centered medical homes (PCMHs) were developed and received strong support from the Affordable Care Act of 2010. What are the key characteristics of PCMHs? Select all that apply. Patient-centered care Comprehensive care Duplicated services Coordination of care Improved access to care Continuous evidence-based care

Patient-centered care Comprehensive care Coordination of care Improved access to care Continuous evidence-based care PCMHs have five key characteristics: 1) patient-centered care, 2) comprehensive care, 3) coordination of care, 4) improved access to care and needed services, and 5) a systems approach making use of evidence-based practice, provided in a continuous feedback loop of evaluation and quality improvement. PCMHs are intended to eliminate fragmented or duplicated care by using a patient-centered approach providing access to physical health, behavioral health, and supportive community and social services. p. 53

What individuals with mental illness are paid or volunteer to use their experiences to provide recovery-oriented services and support others with mental illness? Recreation therapists Art therapists Peer specialists Licensed professional counselors

Peer specialists Peer specialists are individuals with serious mental illness who receive training to use their experience to provide recovery-oriented services and support others with mental illness. Recreation therapists are typically bachelor's prepared and may be licensed by the state or be nationally certified. Recreational therapists provide activities used to improve emotional, physical, cognitive, and social well-being. Art therapists are prepared at the master's level in art therapy and registered through a professional organization. Licensed professional counselors possess a master's degree in psychology, counseling, or a related field and are licensed by the state. They assess and diagnose psychiatric conditions and provide individual, family, and group counseling. p. 55

personal distance

Personal distance (18 to 40 inches) is for personal communications such as those with friends or colleagues.

An adult comes to the crisis clinic after termination from a job of 15 years. The patient says, "I don't know what to do. How can I get another job Who will pay the bills How will I feed my family" Which nursing diagnosis applies a.Hopelessness b.Powerlessness c.Chronic low self-esteem d.Disturbed thought processes

Powerlessness

Date rape drugs- GBH

Produces relaxation, euphoria, disinhibition Onset: 5-20 min Duration: 1-1.5 hr. Side Effects: Resp. depression, Vomiting, seizures, unconsciousness, coma

What services may be available as part of residential treatment programs? Psychoeducation Physical therapy Vocational training Activities of daily living training Recreational therapy

Psychoeducation Vocational training Activities of daily living training Clients in residential treatment programs may receive psychoeducation about the disease process and medications, vocational training, and training for activities of daily living. Physical therapy and recreational therapy are not considered part of the residential treatment program. p. 56

The client with a recent onset of bipolar disorder is admitted to the mental health unit. The nurse is most likely to arrange which type of group therapy for the client? Occupational therapy Skilled nursing care Psychoeducational group therapy Cognitive behavioral therapy

Psychoeducational group therapy Psychoeducational group therapy focuses on specific psychiatric disorders, medication, goal setting, life planning, and recovery, which would greatly improve the newly diagnosed client's quality of life. Occupational therapy gives clients an opportunity to practice life skills that have been delayed, hampered, or eroded, which is not indicated for this client. Skilled nursing care is appropriate for clients who require continued therapy after discharge from acute care. Cognitive behavioral therapy groups focus on coping skills. p. 56

public distance

Public distance (12 feet or more) relates to public space (e.g., public speaking). In public space one may hail another, and the parties may move about while communicating with one another.

A client tell the mental health nurse "I am terribly frightened! I hear whispering in my head that someone is going to kill me." Which criteria of mental health can the nurse assess as lacking?

Rational Thinking

What does psychiatric rehabilitation emphasize? Recovery and integration into society Reduction in physiological symptoms Need for aggressive interventions Reintegration of multiple personalities into one

Recovery and integration into society Psychiatric rehabilitation is a social model that emphasizes and supports recovery and integration into society. Reduction of psychological symptoms may be an aspect of individual care but is not the emphasis of this model. Aggressive interventions are not provided in the psychiatric rehabilitation model. Reintegration of multiple personalities into one is not the emphasis of psychiatric rehabilitation. p. 54

A client diagnosed with major depressive disorder tells the community mental health nurse, "I usually spend all day watching television. If there's nothing good to watch, I just sleep or think about my problems." What is the nurse's most appropriate action? Suggest that the client instead call some friends. Refer the client for counseling with a recreation therapist. Refer the client for counseling with an occupational therapist. Tell the client that watching television and thinking about problems worsens depression.

Refer the client for counseling with a recreation therapist. A recreation therapist can help the client find activities to do during free time that may better improve emotional, physical, cognitive, and social well-being. Suggesting that the client call friends could make the client feel worse if this is not possible given the client's support system or level of motivation for social engagement. Occupational therapists work with clients to develop the practical and necessary skills of daily independent living. Advising the client that watching television and thinking about problems will only make depression worse conveys judgment without helping the client find better health-promoting activities. p. 55

A client with newly diagnosed bipolar disorder is in need of minimal supervision and support, psychoeducation for symptom management, and understanding of lithium therapy. What is the best care setting to meet this client's needs? Intensive inpatient hospitalization Residential treatment center Intensive outpatient program General medical floor

Residential treatment center This client is best suited to a residential treatment center. The residential treatment centers are short- or long-term 24-hour living environments that provide varying levels of supervision and support, individualized psychoeducation, and access to community support services to achieve maximal independence. The intensive inpatient hospitalization is for clients with suicidal ideation, aggressive impulses, or other issues that require crisis stabilization and behavior modification. Because this client is stable and needs minimal supervision, the intensive outpatient program is unnecessary. A general medical floor is not equipped to fully address the client's mental health needs. p. 56

Which nursing intervention demonstrates the theory behind operant conditioning?

Rewarding the client with a token for avoiding an argument with another client

Following the death of his spouse, an elderly client presents to the emergency room with chest pain, sweaty palms, and dizziness. What is the priority of care in this situation? Administer an antianxiety medication Obtain an order for an antidepressant medication Rule out possible cardiac disease Ask the client if he has a suicide plan

Rule out possible cardiac disease The priority of care in this situation is to rule out cardiac disease. Once the client is medically cleared, depression and anxiety should be addressed. Administering an antianxiety medication may be beneficial but is not the priority in this situation. There is not enough information to indicate the need for an antidepressant medication. The client should be assessed for suicide risk after he is medically cleared. p. 52

Identify the goals of inpatient psychiatric care. Reduction of hallucinations Prevention of delusions Regulation of repetitive behaviors Safety Stabilization Crisis intervention

Safety Stabilization Crisis intervention The goals of inpatient psychiatric care include client safety, stabilization, and crisis intervention. Reduction in hallucinations may be an individual client goal. Prevention of delusions, although desirable, may not be a realistic goal for some clients. Regulation of repetitive behaviors may be an individual goal. pp. 54, 56

A client states, "I will always be alone because nobody could ever love me." The nurse recognizes that the client is expressing what cognitive-behavioral concept?

Schema

The quality of care provided by state hospitals has improved dramatically. What is the clinical role of state hospitals? Provide services for homeless clients Limit exposing the community to the client Extend services once provided in early mental institutions Serve the most seriously ill clients

Serve the most seriously ill clients The clinical role of state hospitals is to serve the most seriously ill clients. Providing services for homeless clients is not the focus or intent of state hospitals. Limiting exposure to the community is not consistent with providing high quality health services, and it violates the Americans with Disabilities Act. Services provided at present day psychiatric hospitals are not intended to become an extension of the early institutions. p. 56

Six months ago, a woman had a prophylactic double mastectomy because of a family history of breast cancer. One week ago, this woman learned her husband was involved in an extramarital affair. The woman tearfully says to the nurse, "What else can happen" What type of crisis is this person experiencing a.Maturational b.Adventitious c.Situational d.Recurring

Situational

Which theorist is associated with behavioral therapy?

Skinner

social distance

Social distance (4 to 12 feet) is applied to strangers or acquaintances, often in public places or formal social gatherings.

The mental health nurse is working with another team member to find temporary housing for a psychiatric client. Which health care team member is most suited to assist in locating appropriate temporary housing? Social worker Licensed professional counselor Occupational therapist Psychiatric mental health advanced practice nurse

Social worker The team member best suited to assist the nurse with locating appropriate temporary housing is the social worker. Social workers may provide counseling and plan for supportive services such as housing, health care, and treatment after the client returns to the community. Licensed professional counselors assess and diagnose psychiatric conditions and provide individual, family, and group counseling. Occupational therapists assist clients to develop or regain independent living skills, activities of daily living, and role performance that have been affected by mental disorders. The psychiatric mental health advanced practice nurse provides assessment, diagnosis, and treatment in outpatient settings. p. 55

The mental health status of a particular client can best be assessed by considering which factor?

Status placement on a continuum from health to illness

The nurse educator is teaching a group of nursing students about the president's vision for mental health care in America. The nurse educator discusses the President's New Freedom Commission on Mental Health. What was the purpose of this group? Studying the relationship between mental illness and physical symptoms Studying the mental health system and making recommendations for its transformation Examining the number of people with mental illness in the United States Collecting data on the prevalence of mental illness in homeless Americans

Studying the mental health system and making recommendations for its transformation The President's New Freedom Commission on Mental Health was charged with studying the mental health system and making recommendations for its transformation. The groups did not study the relationship between mental illness and physical symptoms, examine the number of people with mental illness, or collect data on the number of homeless people with mental illness. pp. 58-59

Which factors may influence the level of psychiatric care given to clients in the current health care system? Substance abuse problems Familial support Medical problems Financial stability Acceptance of treatment Potential for relapse

Substance abuse problems Medical problems Acceptance of treatment Potential for relapse Decisions for the level of care given to psychiatric clients may be influenced by such factors as concurrent psychiatric or substance abuse problems, medical problems, acceptance of treatment, social supports, and disease chronicity or potential for relapse. Familial support consideration, although helpful, may be an unnecessary consideration depending on other available support services and client acceptance. Financial stability is not a consideration when determining appropriate and needed care. p. 53

A rape victim says to the nurse, "I always try to be so careful. I know I should not have walked to my car alone. Was this attack my fault?" Which communication by the nurse is most therapeutic?

Support the victim to separate issues of vulnerability from blame. Although the victim may have made choices that made her vulnerable, she is not to blame for the rape. Correcting this distortion in thinking allows the victim to begin to restore a sense of control. This is a positive response to victimization. The distracters do not permit the victim to begin to restore a sense of control or offer use of non-therapeutic communication techniques. In this interaction, the victim needs to talk about feelings rather than prevention.

Which situation demonstrates use of primary care related to crisis intervention a.Implementation of suicide precautions for a depressed patient b.Teaching stress reduction techniques to a first-year college student c.Assessing coping strategies used by a patient who attempted suicide d.Referring a patient with schizophrenia to a partial hospitalization program

Teaching stress reduction techniques to a first-year college student

termination phase

Termination is discussed during the first interview, and again during the working stage at appropriate times. Termination may occur when the patient is discharged or when the student's clinical rotation ends Summarizing the goals and objectives achieved in the relationship • Discussing ways for the patient to incorporate into daily life any new coping strategies learned during the time spent with the nurse • Reviewing situations that occurred during the time spent together • Exchanging memories, which can help validate the experience for both nurse and patient and facilitate closure of that relationship

A victim of a sexual assault who sits in the emergency department is rocking back and forth and repeatedly saying, "I can't believe I've been raped." This behavior is characteristic of which stage of rape-trauma syndrome?

The acute phase reaction The victim's response is typical of the acute phase and shows cognitive, affective, and behavioral disruptions. This response is immediate and does not include a display of behaviors suggestive of the long-term (reorganization) phase, anger, or a delayed reaction.

A community mental health nurse is preparing to address a national parent-teacher organization. Which statement concerning the availability of residential treatment centers for emotionally disturbed children in the United States should the nurse include? Centers have increased but only marginally since 2004. The numbers of centers grew substantially between 2004 and 2008. Availability of these centers has declined steadily over the last decade. Availability of residential treatment beds has remained steady over the last decade.

The numbers of centers grew substantially between 2004 and 2008. The number of residential treatment centers for emotionally disturbed children increased from 33,835 to 50,063 with available beds per 100,000 civilian population increasing from 11.4 to 16.5 for the years between 2004 and 2008. This increase is significant, not marginal. Availability of centers has not declined, nor has availability of treatment beds remained stagnant; both increased as the quantity of centers increased. p. 55

At the last contracted visit in the crisis intervention clinic, an adult says, "I've emerged from this a stronger person. You helped me get my life back in balance." The nurse responds, "I think we should have two more sessions to explore why your reactions were so intense." Which analysis applies a.The patient is experiencing transference. b.The patient demonstrates need for continuing support. c.The nurse is having difficulty terminating the relationship. d.The nurse is empathizing with the patient's feelings of dependency.

The nurse is having difficulty terminating the relationship.

The nursing diagnosis Rape-trauma syndrome applies to a rape victim in the emergency department. Select the most appropriate outcome to achieve before discharging the patient.

The patient agrees to a follow-up appointment with a rape victim advocate. Agreeing to keep a follow-up appointment is a realistic short-term outcome. The victim is in the acute phase; the distracters are unlikely to be achieved during the limited time the victim is in an emergency department.

Therapeutic encounter

The type of relationship that occurs may be informal and not extensive, such as when the nurse and patient meet for only a few sessions. However, even though it is brief, the relationship may be substantial, useful, and important for the patient.

What is a common misconception about psychiatric nurses in acute care? They have just as much education as registered nurses on other care units. They just talk to clients and lose their nursing skills. They are very good at monitoring effects of psychotropic drugs. They may be uncomfortable inserting an intravenous catheter (IV) or administering IV medications.

They just talk to clients and lose their nursing skills. A common misconception about psychiatric nurses is that they "just talk" to clients. Therapeutic communication is a nursing skill that takes time to master. Psychiatric nurses are experts in therapeutic communication. Psychiatric nurses have just as much education as registered nurses in other types of health care. Psychiatric nurses are experienced at monitoring intended psychotropic drug effectiveness. Psychiatric nurses may be uncomfortable performing skills that are not common in the mental healthcare setting such as inserting IV catheters or administering IV medications. p. 57

Transference

Transference is the process whereby a person unconsciously and inappropriately displaces (transfers) onto individuals in his or her current life those patterns of behavior and emotional reactions that originated in relation to significant figures in childhood

A patient tells the nurse, "I had severe chest pain six days ago." The nurse identifies that which cardiac biomarker will be most helpful in determining whether the patient had a myocardial infarction at the time the patient experienced chest pain? Troponin Myoglobin Homocysteine Creatine kinase

Troponin Troponin is a serum cardiac marker that is detectable in the blood up to two weeks after myocardial injury and is used to diagnose a myocardial infarction. Troponin has two subtypes: cardiac-specific troponin T (cTnT) and cardiac-specific troponin I (cTnI). Serum levels of cTnT and cTnI increase 4 to 6 hours after the onset of myocardial injury, peak at 10 to 24 hours, and return to baseline over 10 to 14 days. Myoglobin is a protein found in skeletal and cardiac muscle. It is a sensitive indicator of early myocardial injury but is not specific for cardiac muscle; therefore it is not used to diagnose a myocardial infarction. Myoglobin peaks and returns to normal in 3 to 15 hours. Homocysteine is a protein. High levels of homocysteine may indicate an increased risk for coronary artery disease. It is not used to diagnose myocardial infarction. CK levels begin to rise about 6 hours after an MI, peak at about 18 hours, and return to normal within 24 to 36 hours.https://eolscontent.elsevier.com/105WLB4FDG8/image/1688_LewisFigure33-10.png p. 722

patient-centered partnership

a patient's allowance for control over his or her health care decisions

social relationship

a relationship that is primarily initiated for the purpose of friendship, socialization, enjoyment, or accomplishment of a task

Insurance will not pay for continued private hospitalization of a mentally ill patient. The family considers transferring the patient to a public hospital but expresses concern that the patient will not get any treatment if transferred. Select the nurse's most helpful reply. a. "By law, treatment must be provided. Hospitalization without treatment violates patients' rights." b. "All patients in public hospitals have the right to choose both a primary therapist and a primary nurse." c. "You have a justifiable concern because the right to treatment extends only to provision of food, shelter, and safety." d. "Much will depend on other patients, because the right to treatment for a psychotic patient takes precedence over the right to treatment of a patient who is stable."

a. "By law, treatment must be provided. Hospitalization without treatment violates patients' rights."

A person in the community asks, "Why aren't people with mental illness kept in state institutions anymore?" Select the nurse's best response. a. "Less restrictive settings are available now to care for individuals with mental illness." b. "There are fewer persons with mental illness, so less hospital beds are needed." c. "Most people with mental illness are still in psychiatric institutions." d. "Psychiatric institutions violated patients' rights."

a. "Less restrictive settings are available now to care for individuals with mental illness."

8. Group members are having difficulty deciding what topic to cover in today's session. Which nurse leader response reflects autocratic leadership? a. "We are talking about fear of rejection today." b. "Let's go around the room and make suggestions for today's topic." c. "I will let you come to a conclusion together about what to talk about." d. "I'll work with you to find a suitable topic for today."

a. "We are talking about fear of rejection today."

6. Group dynamics can vary widely and at times members are capable of disrupting the group process. Which of the following participant traits may indicate a need for additional support for a new nurse facilitator? Select all that apply. a. A member with paranoid delusions b. A quietly tearful participant expressing suicidal thoughts c. An angry woman who raises her voice d. A calm but ineffective communicator

a. A member with paranoid delusions b. A quietly tearful participant expressing suicidal thoughts

A nurse is concerned that an agency's policies are inadequate. Which understanding about the relationship between substandard institutional policies and individual nursing practice should guide nursing practice? a. Agency policies do not exempt an individual nurse of responsibility to practice according to professional standards of nursing care. b. Agency policies are the legal standard by which a professional nurse must act and therefore override other standards of care. c. Faced with substandard policies, a nurse has a responsibility to inform the supervisor and discontinue patient care immediately. d. Interpretation of policies by the judicial system is rendered on an individual basis and therefore cannot be predicted.

a. Agency policies do not exempt an individual nurse of responsibility to practice according to professional standards of nursing care.

1. Which individuals are most at risk for displaying aggressive behavior? Select all that apply. a. An adolescent embarrassed in front of friends. b. A young male who feels rejected by the social group. c. A young adult depressed after the death of a friend. d. A middle-aged adult who feels that concerns are going unheard. e. A patient who was discovered telling a lie.

a. An adolescent embarrassed in front of friends. b. A young male who feels rejected by the social group. d. A middle-aged adult who feels that concerns are going unheard. e. A patient who was discovered telling a lie.

9. Twenty-four-hour observation is a good choice for restraint in which of the following patients? a. An inmate with suicidal ideation on hospice care b. A sex offender in the psychiatric intensive care unit c. An aggressive female with antisocial personality disorder d. An inmate diagnosed with paranoid schizophrenia

a. An inmate with suicidal ideation on hospice care

A patient in alcohol rehabilitation reveals to the nurse, "I feel terrible guilt for sexually abusing my 6-year-old before I was admitted." Select the nurse's most important action. a. Anonymously report the abuse by phone to the local child protection agency. b. Reply, "I'm glad you feel comfortable talking to me about it." c. File a written report with the agency's ethics committee. d. Respect nurse-patient relationship confidentiality.

a. Anonymously report the abuse by phone to the local child protection agency.

3. Which intervention(s) should the nurse implement when helping a patient expresses anger in an inappropriate manner? Select all that apply. a. Approach the patient in a calm, reassuring manner. b. Provide suggestions regarding acceptable ways of communicating anger. c. Warn the patient that being angry is not a healthy emotional state. d. Set limits on the angry behavior that will be tolerated. e. Allow any expression of anger as long as no one is hurt.

a. Approach the patient in a calm, reassuring manner. b. Provide suggestions regarding acceptable ways of communicating anger. d. Set limits on the angry behavior that will be tolerated.

4. What group would benefit most from a laissez-faire leader? a. Art group b. Grief group c. Social skills group d. Anger management group

a. Art group

Which action by a psychiatric nurse best applies the ethical principle of autonomy? a. Exploring alternative solutions with the patient, who then makes a choice. b. Suggesting that two patients who were fighting be restricted to the unit. c. Intervening when a self-mutilating patient attempts to harm self. d. Staying with a patient demonstrating a high level of anxiety.

a. Exploring alternative solutions with the patient, who then makes a choice.

3. What advantages does group therapy have over those of individual therapy? Select all that apply. a. Groups are less expensive than one-to-one therapy. b. Groups provide an opportunity to learn from others. c. Groups are homogeneous in composition. d. Feedback is available from the group leader and group members. e. Interpersonal skills can be practiced in a safe environment.

a. Groups are less expensive than one-to-one therapy. b. Groups provide an opportunity to learn from others. d. Feedback is available from the group leader and group members. e. Interpersonal skills can be practiced in a safe environment.

4. Which guidelines should direct nursing care when deescalating an angry patient? Select all that apply. a. Intervene as quickly as possible b. Identify the trigger for the anger c. Behave calmly and respectfully d. Recognize the patient's need for increased personal space e. Demands are agreed to as long as they won't result in harm to anyone

a. Intervene as quickly as possible b. Identify the trigger for the anger c. Behave calmly and respectfully d. Recognize the patient's need for increased personal space

1. Which outcome would be appropriate for a group session on medication education? Select all that apply. a. Patient will identify three side effects of prescribed medication. b. Patient will verbalize the purpose of taking the medication. c. Patient will acknowledge and accept the financial cost of prescribed medications. d. Patient will correctly identify time of day and dose for each prescribed medication. e. Patient will list two dangerous drug-drug and drug-food interactions for prescribed medications.

a. Patient will identify three side effects of prescribed medication. b. Patient will verbalize the purpose of taking the medication. d. Patient will correctly identify time of day and dose for each prescribed medication. e. Patient will list two dangerous drug-drug and drug-food interactions for prescribed medications.

In order to release information to another health care facility or third party regarding a patient diagnosed with a mental illness, the nurse must obtain a. a signed consent by the patient for release of information stating specific information to be released. b. a verbal consent for information release from the patient and the patient's guardian or next of kin. c. permission from members of the health care team who participate in treatment planning. d. approval from the attending psychiatrist to authorize the release of information.

a. a signed consent by the patient for release of information stating specific information to be released.

A patient comes to the crisis center saying, "I'm in a terrible situation. I don't know what to do." The triage nurse can initially assume that the patient is: a.suicidal. b.anxious and fearful. c.misperceiving reality. d.potentially homicidal.

anxious and fearful.

Six months ago, a woman had a prophylactic double mastectomy because of a family history of breast cancer. One week ago, this woman learned her husband was involved in an extramarital affair. The woman tearfully, "What else can happen" If the woman's immediate family is unable to provide sufficient support, the nurse should: a.suggest hospitalization for a short period. b.ask what other relatives or friends are available for support. c.tell the patient, "You are a strong person. You can get through this crisis." d.foster insight by relating the present situation to earlier situations involving loss.

ask what other relatives or friends are available for support.

7. A nurse named Darryl has been hired to work in a psychiatric intensive care unit. He has undergone training on recognizing escalating anger. Which statement indicates that he understands danger signs in regard to aggression? a. "I need to be aware of patients who are withdrawn and sitting alone." b. "An obvious change in behavior is a risk factor for aggression." c. "Patients who seek constant attention are more likely to be violent." d. "Patients who talk to themselves are the most dangerous."

b. "An obvious change in behavior is a risk factor for aggression."

An adolescent hospitalized after a violent physical outburst tells the nurse, "I'm going to kill my father, but you can't tell anyone." Select the nurse's best response. a. "You are right. Federal law requires me to keep clinical information private." b. "I am obligated to share that information with the treatment team." c. "Those kinds of thoughts will make your hospitalization longer." d. "You should share this thought with your psychiatrist."

b. "I am obligated to share that information with the treatment team."

2. A newly admitted male patient has a long history of aggressive behavior toward staff. Which statement by the nurse demonstrates the need for more information about the use of restraint? a. "If his behavior warrants restraints, someone will stay with him the entire time he's restrained." b. "I'll call the primary provider and get an as needed (prn) seclusion/restraint order." c. "If he is restrained, be sure he is offered food and fluids regularly." d. "Remember that physical restraints are our last resort."

b. "I'll call the primary provider and get an as needed (prn) seclusion/restraint order."

An aide in a psychiatric hospital says to the nurse, "We don't have time every day to help each patient complete a menu selection. Let's tell dietary to prepare popular choices and send them to our unit." Select the nurse's best response. a. "Thanks for the suggestion, but that idea may not work because so many patients take MAOI (monoamine oxidase inhibitor) antidepressants." b. "Thanks for the idea, but it's important to treat patients as individuals. Giving choices is one way we can respect patients' individuality." c. "Thank you for the suggestion, but the patients' bill of rights requires us to allow patients to select their own diet." d. "Thank you. That is a very good idea. It will make meal preparation easier for the dietary department."

b. "Thanks for the idea, but it's important to treat patients as individuals. Giving choices is one way we can respect patients' individuality."

Select the example of a tort. a. The plan of care for a patient is not completed within 24 hours of the patient's admission. b. A nurse gives a prn dose of an antipsychotic drug to an agitated patient because the unit is short-staffed. c. An advanced practice nurse recommends hospitalization for a patient who is dangerous to self and others. d. A patient's admission status changed from involuntary to voluntary after the patient's hallucinations subside.

b. A nurse gives a prn dose of an antipsychotic drug to an agitated patient because the unit is short-staffed.

Which nursing intervention demonstrates false imprisonment? a. A confused and combative patient says, "I'm getting out of here, and no one can stop me." The nurse restrains this patient without a health care provider's order and then promptly obtains an order. b. A patient has been irritating and attention seeking much of the day. A nurse escorts the patient down the hall saying, "Stay in your room, or you'll be put in seclusion." c. An involuntarily hospitalized patient with suicidal ideation runs out of the psychiatric unit. The nurse rushes after the patient and convinces the patient to return to the unit. d. An involuntarily hospitalized patient with homicidal ideation attempts to leave the facility. A nurse calls the security team and uses established protocols to prevent the patient from leaving.

b. A patient has been irritating and attention seeking much of the day. A nurse escorts the patient down the hall saying, "Stay in your room, or you'll be put in seclusion."

7. The nurse is caring for four patients. Which patients would not be appropriate to consider for inpatient group therapy? (Select all that apply.) The patient who: a. Has limited financial and social resources b. Is experiencing acute mania c. Has few friends on the unit d. Is preparing for discharge tomorrow e. Does not speak up often, yet listens to others

b. Is experiencing acute mania

5. The nurse tells group members that they will be working on expressing conflicts during the current group session. Which phase of group development is represented? a. Planning (formation) phase b. Orientation phase c. Working phase d. Termination phase

b. Orientation phase

Which action by a nurse constitutes a breach of a patient's right to privacy? a. Documenting the patient's daily behavior during hospitalization b. Releasing information to the patient's employer without consent c. Discussing the patient's history with other staff during care planning d. Asking family to share information about a patient's pre-hospitalization behavior

b. Releasing information to the patient's employer without consent

A nurse prepares to administer a scheduled intramuscular injection of an antipsychotic medication to an outpatient diagnosed with schizophrenia. As the nurse swabs the site, the patient shouts, "Stop! I don't want to take that medicine anymore. I hate the side effects." Select the nurse's best action. a. Assemble other staff for a show of force and proceed with the injection, using restraint if necessary. b. Stop the medication administration procedure and say to the patient, "Tell me more about the side effects you've been having." c. Proceed with the injection but explain to the patient that there are medications that will help reduce the unpleasant side effects. d. Say to the patient, "Since I've already drawn the medication in the syringe, I'm required to give it, but let's talk to the doctor about delaying next month's dose."

b. Stop the medication administration procedure and say to the patient, "Tell me more about the side effects you've been having."

A patient diagnosed with schizophrenia believes a local minister stirred evil spirits. The patient threatens to bomb a local church. The psychiatrist notifies the minister. Select the answer with the correct rationale. The psychiatrist a. released information without proper authorization. b. demonstrated the duty to warn and protect. c. violated the patient's confidentiality. d. avoided charges of malpractice.

b. demonstrated the duty to warn and protect.

A nurse finds a psychiatric advance directive in the medical record of a patient currently experiencing psychosis. The directive was executed during a period when the patient was stable and competent. The nurse should a. review the directive with the patient to ensure it is current. b. ensure that the directive is respected in treatment planning. c. consider the directive only if there is a cardiac or respiratory arrest. d. encourage the patient to revise the directive in light of the current health problem.

b. ensure that the directive is respected in treatment planning

Two hospitalized patients fight whenever they are together. During a team meeting, a nurse asserts that safety is of paramount importance, so treatment plans should call for both patients to be secluded to keep them from injuring each other. This assertion a. reinforces the autonomy of the two patients. b. violates the civil rights of both patients. c. represents the intentional tort of battery. d. correctly places emphasis on safety.

b. violates the civil rights of both patients.

A patient experiencing psychosis asks a psychiatric technician, "What's the matter with me?" The technician replies, "Nothing is wrong with you. You just need to use some self-control." The nurse who overheard the exchange should take action based on a. the technician's unauthorized disclosure of confidential clinical information. b. violation of the patient's right to be treated with dignity and respect. c. the nurse's obligation to report caregiver negligence. d. the patient's right to social interaction.

b. violation of the patient's right to be treated with dignity and respect.

According to current information what factor is associated with the most disabling mental disorders?

biological influences

A voluntarily hospitalized patient tells the nurse, "Get me the forms for discharge. I want to leave now." Select the nurse's best response. a. "I will get the forms for you right now and bring them to your room." b. "Since you signed your consent for treatment, you may leave if you desire." c. "I will get them for you, but let's talk about your decision to leave treatment." d. "I cannot give you those forms without your health care provider's permission."

c. "I will get them for you, but let's talk about your decision to leave treatment."

In which situations would a nurse have the duty to intervene and report? (Select all that apply.) a. A peer has difficulty writing measurable outcomes. b. A health care provider gives a telephone order for medication. c. A peer tries to provide patient care in an alcohol-impaired state. d. A team member violates relationship boundaries with a patient. e. A patient refuses medication prescribed by a licensed health care provider.

c. A peer tries to provide patient care in an alcohol-impaired state. d. A team member violates relationship boundaries with a patient

8. An effective method of preventing escalation in an environment with violent offenders is to develop a level of trust through: a. A casual authoritative demeanor b. Keeping patients busy c. Brief, frequent, nonthreatening encounters d. Threats of seclusion or punishment

c. Brief, frequent, nonthreatening encounters

6. John Patrick is a widower with four daughters. He has enjoyed a healthy relationship with all of them until they reached puberty. As each girl began to mature physically, he acted in an aggressive manner, beating her without provocation. John Patrick is most likely acting on: a. Self-protective measures b. Stress of raising four daughters c. Frustration of unhealthy desire d. Motivating his daughters to be chaste

c. Frustration of unhealthy desire

Which actions violate the civil rights of a psychiatric patient? The nurse (Select all that apply) a. performs mouth checks after overhearing a patient say, "I've been spitting out my medication." b. begins suicide precautions before a patient is assessed by the health care provider. c. opens and reads a letter a patient left at the nurse's station to be mailed. d. places a patient's expensive watch in the hospital business office safe. e. restrains a patient who uses profanity when speaking to the nurse.

c. opens and reads a letter a patient left at the nurse's station to be mailed. e. restrains a patient who uses profanity when speaking to the nurse.

After leaving work, a nurse realizes documentation of administration of a prn medication was omitted. This off-duty nurse phones the nurse on duty and says, "Please document administration of the medication for me. My password is alpha1." The nurse receiving the call should a. fulfill the request promptly. b. document the caller's password. c. refer the matter to the charge nurse to resolve. d. report the request to the patient's health care provider.

c. refer the matter to the charge nurse to resolve.

Which individual diagnosed with a mental illness may need involuntary hospitalization? An individual a. who has a panic attack after her child gets lost in a shopping mall. b. with visions of demons emerging from cemetery plots throughout the community. c. who takes 38 acetaminophen tablets after the person's stock portfolio becomes worthless. d. diagnosed with major depression who stops taking prescribed antidepressant medication.

c. who takes 38 acetaminophen tablets after the person's stock portfolio becomes worthless.

A new antidepressant is prescribed for an elderly patient diagnosed with major depressive disorder, but the dose is more than the usual geriatric dose. The nurse should a. consult a reliable drug reference. b. teach the patient about possible side effects and adverse effects. c. withhold the medication and confer with the health care provider. d. encourage the patient to increase oral fluids to reduce drug concentration.

c. withhold the medication and confer with the health care provider.

orientation phase

can last for a few meetings or can extend over a longer period. It is the first time the nurse and the patient meet, and they are strangers to each other.

Which branch of epidemiology is the nurse involved in when seeking outcomes for patients whose depression was treated with ECT?

clinical

9. A patient continues to dominate the group conversation despite having been asked to allow others to speak. What is the most appropriate group leader response? a. "You are monopolizing the conversation." b. "When you talk constantly, it makes everyone feel angry." c. "You are supposed to allow others to talk also." d. "When you speak out of turn, I am concerned that others cannot participate equally."

d. "When you speak out of turn, I am concerned that others cannot participate equally."

2. What question by the nurse leader is helpful in managing a monopolizing member of a group? a. "You seem angry. Is there something you want to discuss with the group?" b. "Would it be helpful if you had time to think about the question?" c. "Would you tell us about experiences that have frightened you?" d. "Who else would like to share feelings about this issue?"

d. "Who else would like to share feelings about this issue?"

5. Which comorbid condition would result in cautious use of a selective serotonin reuptake inhibitors for a patient with chronic aggression? a. Asthma b. Anxiety disorder c. Glaucoma d. Bipolar disorder

d. Bipolar disorder

In a team meeting a nurse says, "I'm concerned about whether we are behaving ethically by using restraint to prevent one patient from self-mutilation, while the care plan for another self-mutilating patient requires one-on-one supervision." Which ethical principle most clearly applies to this situation? a. Beneficence b. Autonomy c. Fidelity d. Justice

d. Justice

A newly admitted acutely psychotic patient is a private patient of the medical director and a private-pay patient. To whom does the psychiatric nurse assigned to the patient owe the duty of care? a. Medical director b. Hospital c. Profession d. Patient

d. Patient

A patient experiencing psychosis became aggressive, struck another patient, and required seclusion. Select the best documentation. a. Patient struck another patient who attempted to leave day room to go to bathroom. Seclusion necessary at 1415. Plan: Maintain seclusion for 8 hours and keep these two patients away from each other for 24 hours. b. Seclusion ordered by physician at 1415 after command hallucinations told the patient to hit another patient. Careful monitoring of patient maintained during period of seclusion. c. Seclusion ordered by MD for aggressive behavior. Begun at 1415. Maintained for 2 hours without incident. Outcome: Patient calmer and apologized for outburst. d. Patient pacing, shouting. Haloperidol 5 mg given PO at 1300. No effect by 1315. At 1415 patient yelled, "I'll punch anyone who gets near me," and struck another patient with fist. Physically placed in seclusion at 1420. Seclusion order obtained from MD at 1430.

d. Patient pacing, shouting. Haloperidol 5 mg given PO at 1300. No effect by 1315. At 1415 patient yelled, "I'll punch anyone who gets near me," and struck another patient with fist. Physically placed in seclusion at 1420. Seclusion order obtained from MD at 1430.

10. The nurse is planning care, which includes a dual-diagnosis group. Which patient would be appropriate for this group? The patient with: a. Depression and suicidal tendencies b. Anxiety and frequent migraine headaches c. Bipolar disorder and anorexia nervosa d. Schizophrenia and alcohol abuse

d. Schizophrenia and alcohol abuse

Which documentation of a patient's behavior best demonstrates a nurse's observations? a. Isolates self from others. Frequently fell asleep during group. Vital signs stable. b. Calmer; more cooperative. Participated actively in group. No evidence of psychotic thinking. c. Appeared to hallucinate. Frequently increased volume on television, causing conflict with others. d. Wore four layers of clothing. States, "I need protection from evil bacteria trying to pierce my skin."

d. Wore four layers of clothing. States, "I need protection from evil bacteria trying to pierce my skin."

What is the legal significance of a nurse's action when a patient verbally refuses medication and the nurse gives the medication over the patient's objection? The nurse a. has been negligent. b. committed malpractice. c. fulfilled the standard of care. d. can be charged with battery.

d. can be charged with battery.

A family member of a patient with delusions of persecution asks the nurse, "Are there any circumstances under which the treatment team is justified in violating a patient's right to confidentiality?" The nurse should reply that confidentiality may be breached a. under no circumstances. b. at the discretion of the psychiatrist. c. when questions are asked by law enforcement. d. if the patient threatens the life of another person.

d. if the patient threatens the life of another person.

Which patient meets criteria for involuntary hospitalization for psychiatric treatment? The patient who a. is noncompliant with the treatment regimen. b. fraudulently files for bankruptcy. c. sold and distributed illegal drugs. d. threatens to harm self and others.

d. threatens to harm self and others.

Which individual diagnosed with mental illness may need emergency or involuntary admission? The individual who a. resumes using heroin while still taking naltrexone (ReVia). b. reports hearing angels playing harps during thunderstorms. c. does not keep an outpatient appointment with the mental health nurse. d. throws a heavy plate at a waiter at the direction of command hallucinations.

d. throws a heavy plate at a waiter at the direction of command hallucinations.

hyperarousal

difficulty concentrating or falling asleep, easily startled, tense, angry outbursts, this makes it difficult for victim to complete ADLs, tasks

follow up care for sexual assault victims

discharge with written material for community support groups follow up visit at 2, 4, and 6 weeks STD and pregnancy screens complete hep B vaccines if indicated counseling emergency contraception emotional/psychological state should be reassessed with a personal PHONE CALL or personal contact within 24-48 hours of discharge

paralinguistics,

encompasses voice loudness, pitch, rate, and fluency

what term is used to identify the quantitative study of the distribution of mental disorders in human populations?

epidemiology

A nurse works a rape telephone hotline. Communication with potential victims should focus on:

explaining immediate steps victims should take. The telephone counselor establishes where the victim is and what has happened and provides the necessary information to enable the victim to decide what steps to take immediately. Counseling is not the focus until immediate problems are resolved. The victim remains anonymous. The other distracters are inappropriate or incorrect because counselors are trained to be empathetic rather than sympathetic.

Date rape drugs- Ketamine

hallucinogen, dream like state, amnesia complete compliance of victim Onset: 30 sec IV 20 min PO Duration: 30--60 minutes Side Effects: confusion, resp. compromise, paranoia, combativeness,drooling

When a tumor of the cerebellum is present, the nurse should expect that the client would initially demonstrate which sign/symptom?

impaired balance

The principle most useful to a nurse planning crisis intervention for any patient is that the patient: a.is experiencing a state of disequilibrium. b.is experiencing a type of mental illness. c.poses a threat of violence to others. d.has high potential for self-injury.

is experiencing a state of disequilibrium.

The medication prescribed for a client acts by blocking reuptake of both serotonin and norepinephrine. The nurse evaluates the treatment as successful when observing which client behavior?

laughing at a joke

The nurse responsible for the care of a client prescribed clonazepam should evaluate treatment being successful when the client demonstrates which behavior?

less anxiety

An unconscious teenager is treated in the emergency department. The teenager's friends suspect a rape occurred at a party. Priority action by the nurse should focus on:

maintaining physiologic stability. Because the patient is unconscious, the risk for airway obstruction is present. The nurse's priority will focus on maintaining physiologic stability. The distracters are of lower priority than preserving physiological functioning.

The nurse is working with a client experiencing depression stemming from low self-esteem. The client is distrustful of unit staff and "just wants to go home." Initially what is the nurses priority?

making the client feel physically and emotionally safe

Date rape drugs- Rohypnol

potent when COMBINED with alcohol sedation, muscle relaxation Onset: 10-30 minutes Duration: 2-12 hours Side Effects: resp. depression, GI concerns

Before a victim of sexual assault is discharged from the emergency department, the nurse should:

provide referral information verbally and in writing. Immediately after the assault, rape victims are often disorganized and unable to think well or remember instructions. Written information acknowledges this fact and provides a solution. The distracters violate the patient's right to privacy, evidence a rescue fantasy, and offer a platitude that is neither therapeutic nor effective.

A victim of a sexual assault comes to the hospital for treatment but abruptly decides to decline treatment and leaves the facility. While respecting the person's rights, the nurse should:

provide written information about physical and emotional reactions the person may experience. All information given to a patient before he or she leaves the emergency department should be in writing. Patients who are anxious are unable to concentrate and therefore cannot retain much of what is verbally imparted. Written information can be read and referred to later. Patients may not be kept against their will or coerced into treatment. This constitutes false imprisonment.

The incoherent thought and speech patterns of the client diagnosed with schizophrenia are related to the brain's inability to perform which function?

regulate conscious mental activity

re-experiencing

repeated reliving of the event that interfere with daily activity flashbacks, frightening thoughts, memories/dreams

A nurse cares for a rape victim who was given a drink that contained flunitrazepam (Rohypnol) by an assailant. Which intervention has priority? Monitoring for:

respiratory depression. Monitoring for respiratory depression takes priority over hypotonia, seizures, or coma.

Acute distress disorder

severe numbing, derealization, inability to remember stressful event, fear, helplessness, horror occurs within 1 month of extreme stress

Which function is classified as a circadian rhythm?

sleep cycle

While conducting the initial interview with a patient in crisis, the nurse should: a.speak in short, concise sentences. b.convey a sense of urgency to the patient. c.be forthright about time limits of the interview. d.let the patient know the nurse controls the interview.

speak in short, concise sentences.

A victim of spousal violence comes to the crisis center seeking help. Crisis intervention strategies the nurse uses will focus on: a.supporting emotional security and reestablishing equilibrium. b.long-term resolution of issues precipitating the crisis. c.promoting growth of the individual. d.providing legal assistance.

supporting emotional security and reestablishing equilibrium.

rape trauma syndrome

sustained and maladaptive response to sexual assault guilt, shame, depression, suicide, helplessness, substance abuse, paranoia etc. acute phase- confusion, disbelief long term reorganization phase- development of fears, phobias, numbness, flashbacks, increased motor activity

Homeostasis is promoted by interaction between the brain and internal organs and is mediated by component of the nervous system?

the autonomic nervous system

working phase

the nurse and patient together identify and explore areas in the patient's life that are causing problems.

therapeutic relationship (patient-centered partnership

the relationship is consistently focused on the patient's problem and needs

Countertransference

the tendency of the nurse to displace onto the patient feelings related to people in his or her past.

A patient with a history of asthma presents to the emergency department with wheezing, chest pain, and shortness of breath. The nurse reviews the patient's medication profile and identifies that which type of medication may be responsible for the patient's symptoms? Short-acting nitrate β-adrenergic blocker Calcium channel blocker Angiotensin II receptor blocker

β-adrenergic blocker The patient with a history of asthma should avoid β-adrenergic blockers because their side effects include wheezing from bronchospasm. Short-acting nitrates are the first line treatment for a patient with angina and can be used safely in patients. Calcium channel blockers are used in patients if β-blockers are contraindicated, poorly tolerated, or do not control anginal symptoms. Angiotensin II receptor blockers can be given to the patient safely because they have no harmful effects on the respiratory system. p. 717

The nurse provides education to a patient that receives a prescription for sublingual nitroglycerin for treatment of angina. Which statement made by the patient indicates the need for further teaching? "I cannot take sildenafil and nitroglycerin at the same time." "I can take a tablet 5 to 10 minutes before starting an activity that causes angina." "After taking a tablet, I may experience a headache, dizziness, or flushing." "After the bottle is open, the tablets lose potency and should be replaced every 12 months."

"After the bottle is open, the tablets lose potency and should be replaced every 12 months." After the bottle is open, the tablets lose potency and should be replaced every 6 months. The patient can take the medication prophylactically before starting an activity that is known to cause angina. In these cases the patient can take a tablet or spray 5 to 10 minutes before beginning the activity. Headache, dizziness, or flushing may occur after taking the medication. Patients should be instructed not to combine nitroglycerin with drugs used for erectile dysfunction (e.g., sildenafil [Viagra]) as severe hypotension can occur. p. 716

A 29-year-old patient has been admitted following a suicide attempt. Which nursing statement illustrates the concept of patient advocacy?

"Dr. Raye, I notice you ordered fluoxetine for this patient. During the admission interview, the client stated they will refuse the medication because of adverse effects they experienced previously"

Which instructions should the nurse provide to a patient that takes nitroglycerin? "Place the nitroglycerin tablet under the tongue." "Change your position slowly after taking the medicine." "Store nitroglycerin tablets in a warm and bright environment." "Replace nitroglycerin tablets annually once the bottle is opened." "Discard nitroglycerin tablets if they do not cause a tingling sensation."

"Place the nitroglycerin tablet under the tongue." "Change your position slowly after taking the medicine." "Discard nitroglycerin tablets if they do not cause a tingling sensation." Nitroglycerin tablets should be placed under the tongue and allowed to dissolve. Nitroglycerin causes orthostatic hypotension on administration, so the patient should be advised to change position slowly. Nitroglycerin should cause a tingling sensation when administered; if there is not a tingling sensation, the medication should be discarded. The patient should be instructed to store nitroglycerin away from light and heat to protect it from degradation. Once the nitroglycerin bottle is opened, the tablets lose their potency and need to be replaced every six months. p. 716

A patient is admitted to the hospital with chronic stable angina. The nurse provides teaching about pain associated with the diagnosis. What information should the nurse include? "The pain typically occurs at rest and usually lasts more than 10 minutes." "The pain is triggered by activities of daily living; therefore they will need to be limited." "The pain may last for a few minutes and stop when the precipitating factor is resolved." "The pain is triggered by smoking; therefore a smoking cessation program will be recommended."

"The pain may last for a few minutes and stop when the precipitating factor is resolved." The pain of chronic stable angina usually lasts for only a few minutes and typically subsides when the precipitating factor is resolved (e.g., resting, calming down, using nitroglycerin, etc.). The pain of unstable angina occurs at rest or with minimal exertion and lasts for more than 10 minutes. Microvascular angina is triggered by activities of daily living, such as shopping, dressing, working, etc. Prinzmetal angina is triggered by smoking and increased levels of certain substances (e.g., histamine, epinephrine, and cocaine). p. 712

Which severe mental illness is recognized across cultures?

-Schizophrenia -Bipolar disorder

Parents of a mentally ill teenager say, "We have never known anyone who was mentally ill. We have no one to talk to because none of our friends understand the problems we are facing." Select the nurse's most helpful intervention. a. Refer the parents to a support group. b. Build the parents' self-concept as coping parents. c. Teach the parents techniques of therapeutic communication. d. Facilitate achievement of normal developmental tasks of the family.

A

Which documentation of family assessment indicates a healthy and functional family? a. Members provide mutual support. b. Power is distributed equally among all members. c. Members believe there are specific causes for events. d. Under stress, members turn inward and become enmeshed.

A

A patient with stable angina receives a prescription for isosorbide dinitrate. What medication education should the nurse provide to the patient? A. "Change positions slowly after taking this medication." B. "This is a short-acting nitrate that may produce a headache." C. "You cannot take sublingual nitroglycerin while taking this medication." D. "You will be scheduled for a 10- to 14-hour nitrate-free period every day." E. "You should take this medication as needed when experiencing chest pain."

A and D The patient receiving nitrates is often scheduled for a 10- to 14-hour nitrate-free period every day to limit tolerance to the long-acting nitroglycerin (NTG). Orthostatic hypotension is caused by venous dilation, and patients should be provided with instructions about safety precautions while taking the medication. Remind patients that taking a long-acting NTG preparation should not keep them from using a translingual or sublingual (SL) NTG if angina develops. The medication is a long-acting nitrate that is taken regularly. The short-acting nitrates are taken as needed. The nurse should warn the patient that headache is a side effect but should not tell the patient that the medication is a short-acting nitrate. p. 716

Which client assessment data demonstrates parity related to mental health care?

A client's mental health coverage is equal to his/her medical/surgical coverage

A wife believes her husband is having an affair. Lately, he has been disinterested in romance and working late. The husband has an important, demanding project at work. The mother asks her teen, "What have you noticed about your father?" The teen later mentions this to the father, who says, "Tell your mother that I can't deal with her insecurities right now." Family therapy should focus on: (select all that apply) a. identifying and reducing the cognitive distortion in each parent's perceptions. b. confronting the family with the need for honest, direct, assertive communication. c. helping the parents find ways to cope more effectively with their stress and fears. d. supporting the teen to redirect the parents when they try to communicate through her. e. convincing the mother that her fear of an affair is due to her own insecurities and unfounded. f. helping the husband understand how others might misinterpret the changes in his behavior.

A, C, D, F

The nurse provides basic physical activity guidelines for a patient that is being discharged following acute coronary syndrome (ACS). The nurse recommends following the FITT formula, which includes which guidelines?

A: Physical activity that is regular, rhythmic, and repetitive. The FITT formula includes physical activities that are regular, rhythmic, and repetitive, using large muscles to build up endurance. The increase in heart rate (HR) should not exceed 20 beats/min over the resting HR. The physical activity session should build up to 30 minutes. There is no weightlifting included in the formula. p. 732

A therapy group adds new members as others leave. What type of group is evident? a. Open b. Closed c. Homogeneous d. Heterogeneous

ANS: A An open group is a group that adds members throughout the life of the group as other members leave and as more persons who would benefit from the group become available. A closed group does not add new members; the membership is established at the beginning and, except for the occasional losses as some members leave, does not change thereafter. A homogeneous group includes members who are similar, and a heterogeneous group includes dissimilar members; not enough data are provided here to determine which applies in this case.

1. Who led reform efforts to correct types of inhumane practices in the care of criminals, those with mental disorders, and victims of the Civil War? a. Dorothea Dix b. Philippe Pinel c. Benjamin Rush d. Clifford Beers

ANS: A Dorothea Dix led reform efforts to correct types of inhumane practices in the care of criminals, those with mental disorders, and victims of the Civil War.

12. A client who was deinstitutionalized during the 1960s may have experienced difficulties living in the community because of: a. Insufficient community mental health services b. Difficulties connecting with community resources c. Lack of affordable housing d. Increased cost of care

ANS: A Families were not prepared for treatment responsibilities. There were no education and support programs for families. Staff in nursing homes lacked skills to treat people with mental disorders, and clients had little or no supervision in independent settings.

A leader plans to start a new self-esteem building group. Which intervention would be most helpful for assuring mutual respect within the group? a. Describe the importance of mutual respect in the first session and make it a group norm. b. Exclude potential members whose behavior suggests they are likely to be disrespectful. c. Give members a brochure describing the purpose, norms, and expectations of the group. d. Explain that mutual respect is expected and confront those who are not respectful.

ANS: A It is helpful to motivate members to behave respectfully by describing how mutual respect benefits all members and is necessary for the group to be fully therapeutic. Setting a tone and expectation of mutual respect from the outset is the most helpful intervention listed. Excluding members because of how they might behave could exclude members who would have been appropriate, depriving them of the potential benefits of the group. Conveying expectations by brochure is less effective than doing so orally, because it lacks the connection to each member a skilled leader can create to motivate members and impart the expectation of respect. Confronting inappropriate behavior is therapeutic but only addresses existing behavior rather than preventing all such undesired behavior.

3. Many clients with mental disorders were institutionalized because: a. There was a continued fear of people with mental disorders. b. The success rate was higher with hospital treatment. c. It was easier to stabilize clients with psychotropic medications. d. It provided a less stressful environment for clients to recuperate.

ANS: A Many clients with mental disorders were institutionalized because of a continued fear of people with mental disorders.

Three members of a therapy group share covert glances as other members of the group describe problems. When one makes a statement that subtly criticizes another speaker, the others nod in agreement. Which group dynamic should the leader suspect? a. Some members are acting as a subgroup instead of as members of the main group. b. Some of the members have become bored and are disregarding others. c. Three members are showing their frustration with slower members. d. The leadership of the group has been ineffective.

ANS: A Subgroups, small groups isolated within a larger group and functioning separately from it, sometimes form within therapy groups. When this occurs, subgroup members are cohesive with other subgroup members but not with the members of the larger group. Members of the subgroup may be bored or frustrated or expressing passive aggression, but the primary dynamic is the splitting off from the main group.

23. When a nurse is involved with a group who advocates for improved and effective psychiatric services and consumer empowerment, the nurse is working with the: a. National Alliance for Mentally Ill (NAMI) Consumer Council b. National Mental Health Consumers Association (NMHCA) c. National Association of Psychiatric Survivors (NAPS) d. Consumer/Survivor Mental Health Research and Policy Work Group

ANS: A The NAMI Consumer Counsel advocates for improved and effective psychiatric services and consumer empowerment.

A patient has talked constantly throughout the group therapy session, often repeating the same comments. Other members were initially attentive then became bored, inattentive, and finally sullen. Which comment by the nurse leader would be most effective? a. Say to everyone, "Most of you have become quiet. I wonder if it might be related to concerns you may have about how the group is progressing today." b. Say to everyone, "One person has done most of the talking. I think it would be helpful for everyone to say how that has affected your experience of the group." c. Say to everyone, "I noticed that as our group progressed, most members became quiet, then disinterested, and now seem almost angry. What is going on?" d. Say to the talkative patient, "You have been doing most of the talking, and others have not had a chance to speak as a result. Could you please yield to others now?"

ANS: A The most effective action the nurse leader can take will be the one that encourages the group to solve its own problem. Pointing out changes in the group and asking members to respond to them lays the foundation for a discussion of group dynamics. Asking members to respond to the talkative patient puts that patient in an awkward position, likely increasing her anxiety. As anxiety increases, monopolizing behavior tends to increase as well, so this response would be self-defeating. Asking members what is going on is a broader opening and might lead to responses unrelated to the issue that bears addressing; narrowing the focus to the group process more directly addresses what is occurring in the group. Focusing on the talkative patient would be less effective and involves the leader addressing the issue instead of members first attempting to do so themselves (giving them a chance to practice skills such as assertive communication).

During group therapy, one patient says to another, "When I first started in this group, you were unable to make a decision, but now you can. You've made a lot of progress. I am beginning to think that maybe I can conquer my fears too." Which therapeutic factor is evident by this statement? a. Hope b. Altruism c. Catharsis d. Cohesiveness

ANS: A The patient's profession that he may be able to learn to cope more effectively reflects hope. Groups can instill hope in individuals who are demoralized or pessimistic. Altruism refers to doing good for others, which can result in positive feelings about oneself. Catharsis refers to venting of strong emotions. Cohesion refers to coming together and developing a connection with other group members.

6. Why must a nurse have specialized knowledge and skills in mental health illness and problems in order to provide effective nursing care for this population? a. Services for people with mental disorders are inadequate. b. The effects of mental health are universal. c. Mental health disorders affect small sectors of the population. d. Current health objectives cannot be met without the influence of nursing.

ANS: A There are two universal truths that require nurses to have specialized knowledge and skills in this area: services for people with mental disorders are inadequate in all countries, and mental illness has a major impact on families, communities, and nations.

3. A nurse is referring a client with a mental illness to a community group for additional information and support. Where would the nurse most likely refer a client? Select all that apply. a. Alcoholics Anonymous b. Obsessive-Compulsive Foundation c. Schizophrenics Anonymous d. Anxiety Disorders Association of America

ANS: A, B, C, D Alcoholics Anonymous, Obsessive-Compulsive Foundation, Schizophrenics Anonymous, and Anxiety Disorders Association of America are all sources of information and help for people with mental illness.

MULTIPLE RESPONSE 1. According to the Surgeon Generals report, how can people receive assistance for mental health problems? Select all that apply. a. Specialty mental health systems b. General medical or primary care sectors c. Human service sectors d. Voluntary support networks

ANS: A, B, C, D The Surgeon Generals report defined specialty mental health systems, general medical or primary care sectors, human service sectors, and voluntary support networks as ways through which people could receive assistance.

2. A nurse is using Healthy People 2020 as a guide to develop community programming for mental health. Which objectives for mental health would be found in this document? Select all that apply. a. Reduce suicide rates. b. Increase employment of persons with severe mental illness. c. Increase the proportion of children treated for mental health problems. d. Decrease the number of juvenile facilities that screen new admissions for mental health problems.

ANS: A, C Reducing suicide attempts and increasing the proportion of children with mental health problems who receive treatment are two objectives for mental health targeted by Healthy People 2020.

7. An employee of the National Institute for Mental Health would most likely be involved in the: a. Expansion of psychiatric units in general hospitals b. Development of education programs for community mental health treatment c. Legislation and advocacy for the rights of people with mental disorders d. Deinstitutionalization of patients

ANS: B After the National Mental Health Act was passed in 1946, the National Institute of Mental Health (NIMH) administered its programs. Objectives included development of education and research programs for community mental health treatment approaches.

5. A nurse working in a psychiatric institution during the hospital expansion era would have most likely worked in which location? a. Near small communities with access to families and activities b. In rural areas removed from family and social activities c. Near urban areas with access to families and low-paying jobs d. In urban areas without access to families and jobs

ANS: B During the hospital expansion era, most psychiatric institutions were located in rural areas removed from family and social activities.

22. Which intervention is most appropriate for a nurse to implement when working with older adults and their caregivers? a. Work with legislators to advocate for policies which support families. b. Refer clients and families to community support groups to reduce stress. c. Involve clients and families in case management programs to coordinate care. d. Conduct depression screenings with clients and families on a regular basis.

ANS: B Family caregivers and older adults are at risk for health disruptions. Involvement in a community support group can help reduce stress of caregiving and provide networking opportunities for the older adult.

9. According to the National Institutes of Mental Health (NIMH), which is the leading cause of disability among adults? a. Schizophrenia b. Major depression c. Obsessive-compulsive disorder d. Anxiety

ANS: B Major depression is pervasive and is the leading cause of disability among adults ages 15 to 44.

24. A nurse who is working with persons with serious mental illness should recognize that: a. Persons with severe mental illness require institutionalization until they are functional. b. Inadequate community resources have caused problems with homelessness among this population. c. Serious mental illness originates from childhood events and therapy is ineffective. d. Motivation influences a persons ability to earn a living and purchase necessary medications.

ANS: B Many people with serious mental illness live in poverty because they lack the ability to maintain a suitable standard of living. Brief hospital stays and inadequate community resources have resulted in an increased number of persons with serious mental illness living on the streets or in jail.

16. An example of primary prevention concerning mental health is: a. Group psychotherapy b. Stress reduction education c. Case management d. Monitoring illness symptoms

ANS: B Primary prevention refers to the reduction of health risks, thus stress reduction education is the correct response. Group psychotherapy and case management are secondary preventions, and monitoring illness symptoms is a tertiary prevention.

25. Which intervention would a nurse most likely use when implementing relapse management? a. Use a holistic view of the system. b. Identify triggers. c. Understand the individuals personality. d. Provide crisis intervention.

ANS: B Recognizing triggers that may lead to illness helps the consumer manage the illness and promotes recovery. Examples of triggers are poor social skills, hopelessness, and poor symptom management.

11. A nurse is working with a client who is concerned about discrimination in the workplace based on a current mental health diagnosis. Which legislation has prohibited discrimination for persons with mental disorders? a. National Mental Health Act b. American with Disabilities Act c. Developmental Disabilities Act d. Protection and Advocacy for Mentally Ill Individuals Act

ANS: B The Americans with Disabilities Act prohibited discrimination and promoted opportunities for persons with mental disorders.

The nurse is planning a new sexuality group for patients. Which location would best enhance the effectiveness of this group? a. The hospital auditorium b. A small conference room c. A common area, such as a day room d. The corner of the music therapy room

ANS: B The conference room would provide a quiet, private area with few distractions, separate from other patient areas and effective for teaching and learning about a private topic. The auditorium is too large, and members' anxiety or lack of trust might lead them to spread out too far from each other, interfering with group process. The day room and the music therapy room are too busy and exposed, reducing privacy and increasing distractions.

During a therapy group that uses existential/Gestalt theory, patients shared feelings that occurred at the time of their admission. After a brief silence, one member says, "Several people have described feeling angry. I would like to hear from members who had other feelings." Which group role is evident by this comment? a. Energizer b. Encourager c. Compromiser d. Self-confessor

ANS: B The member is filling the role of encourager by acknowledging those who have contributed and encouraging input from others. An energizer encourages the group to make decisions or take an action. The compromiser focuses on reducing or resolving conflict to preserve harmony. A self-confessor verbalizes feelings or observations unrelated to the group.

13. Which factor has influenced the advancement in the treatment of mental illness? a. The movement of clients out of mental institutions to the community b. A better understanding of the neurobiology of mental illness c. More interest in helping persons with mental illness d. A change in the culture about what constitutes mental illness

ANS: B Two major movements have influenced the treatment of mental illness: consumer advocacy and a better understanding of neurobiology

8. Approximately what percentage of the adult population (ages 18 and older) suffers from a mental disorder in a given year in the United States? a. 6 b. 14 c. 26 d. 30

ANS: C Approximately 26% of the adult population has a mental disorder in the United States.

15. A nurse categorizes mental illness as a biopsychosocial disorder. What is meant by this classification? a. Mental illness is an abnormal brain vasculature that can be detected with angiography. b. Antipsychotic drugs can be used for all types of mental illness. c. Experience and psychosocial factors affect the etiology and treatment of mental illness. d. Mental illness can be cured with the right drugs and therapy.

ANS: C Biochemical changes of the brain are being studied as causes of mental illness.

21. When providing care to an African American population in the community, which would be the most appropriate intervention for the nurse to implement? a. Build on the cultural traditions of the community. b. Develop a support system in a nearby community. c. Integrate mental health services into primary care settings. d. Focus on migrant health issues.

ANS: C Nurses can promote the mental health of this population by integrating mental health services into primary care settings, providing services in community centers, collaborating with faith communities, providing education to decrease the stigma, working toward the provision of safer communities, and recruiting members of this population to work as community mental health providers. Focusing on migrant health issues would occur with the Latino population. Building on cultural traditions is important in the Native American population. Developing a support system outside of the community would not be an appropriate intervention for any population.

A patient in a group therapy session listens to others and then remarks, "I used to think I was the only one who felt afraid. I guess I'm not as alone as I thought." This comment is an example of: a. altruism. b. ventilation. c. universality. d. group cohesiveness.

ANS: C Realizing that one is not alone and that others share the same problems and feelings is called universality. Ventilation refers to expressing emotions. Altruism refers to benefitting by being of help to others. Group cohesiveness refers to the degree of bonding among members of the group.

A nurse at the well child clinic realizes that many parents have misconceptions about effective ways of disciplining their children. The nurse decides to form a group to address this problem. What should be the focus of the group? a. Support b. Socialization c. Health education d. Symptom management

ANS: C The nurse has diagnosed a knowledge deficit. The focus of the group should be education. Support and socialization are beneficial but should not be the primary focus of the group, and symptoms are not identified for intervention here.

2. The first major piece of legislation to influence mental health services in the United States was the: a. National Mental Health Act b. Mental Health Study Act c. Social Security Act d. Protection and Advocacy for Mentally Ill Individuals Act

ANS: C This was in response to economic and social problems of the era and shifted the responsibility of care for ill people from the state to the federal government.

During a group therapy session, a newly admitted patient suddenly says to the nurse, "How old are you? You seem too young to be leading a group." Select the nurse's most appropriate response. a. "I am wondering what leads you to ask. Please tell me more." b. "I am old enough to be a nurse, which qualifies me to lead this group." c. "My age is not pertinent to why we are here and should not concern you." d. "You are wondering whether I have enough experience to lead this group?"

ANS: D A question such as this is common in the initial phase of group development when members are getting to know one another, dealing with trust issues, and testing the leader. Making explicit the implied serves to role model more effective communication and prompts further discussion of the patient's concern. Asking the patient to tell the leader more about the question focuses on the reason for the member's concern rather than on the issue raised (the experience and ability of the leader) and is a less helpful response. "I am old enough to be a nurse" and "age is not pertinent" are defensive responses and fail to address the patient's valid concern.

20. A nurse is counseling a client following an unexpected loss. If given adequate support and adaptation, the client will most likely: a. Recover from the crisis and become mentally ill b. Avoid the loss and potential mental illness c. Be able to ignore the grief d. Resume previous lifestyle in spite of sadness

ANS: D When people do not have adequate resources, there is an increased risk of altered mental health. However, when given adequate support and adaptation, most persons will resume their lifestyles.

Which outcome would be most appropriate for a symptom-management group for persons with schizophrenia? Group members will: a. state the names of their medications. b. resolve conflicts within their families. c. rate anxiety at least two points lower. d. describe ways to cope with their illness.

ANS: D An appropriate psychoeducational focus for patients with schizophrenia is managing their symptoms; coping with symptoms such as impaired memory or impaired reality testing can improve functioning and enhance their quality of life. Names of medications might be appropriate for a medication education group but would be a low priority for symptom management. Addressing intra-family issues would be more appropriate within a family therapy group or possibly a support group. Rating anxiety lower would be an expected outcome for a stress-management group.

4. An individual living during which era would have viewed physical and mental illness as interrelated, resulting from physical conditions? a. Ancient times b. Middle Ages c. Colonial d. Greco-Roman

ANS: D During the ancient times, Middle Ages, and colonial times, mental illness was viewed as resulting from supernatural forces.

A patient tells members of a therapy group, "I hear voices saying my doctor is poisoning me." Another patient replies, "I used to hear voices too. They sounded real, but I found out later they were not. The voices you hear are not real either." Which therapeutic factor is exemplified in this interchange? a. Catharsis b. Universality c. Imitative behavior d. Interpersonal learning

ANS: D Here a member gains insight into his own experiences from hearing about the experiences of others through interpersonal learning. Catharsis refers to a therapeutic discharge of emotions. Universality refers to members realizing their feelings are common to most people and not abnormal. Imitative behavior involves copying or borrowing the adaptive behavior of others.

Which remark by a group participant would the nurse expect during the working stage of group therapy? a. "My problems are very personal and private. How do I know people in this group will not tell others what you hear?" b. "I have enjoyed this group. It's hard to believe that a few weeks ago I couldn't even bring myself to talk here." c. "One thing everyone seems to have in common is that sometimes it's hard to be honest with those you love most." d. "I don't think I agree with your action. It might help you, but it seems like it would upset your family."

ANS: D In the working stage, members actively interact to help each other accomplish goals, and because trust has developed, conflict and disagreement can be expressed. Focusing on trust and confidentiality typically occur in the orientation phase as part of establishing group norms. Commonality and universality are also themes typically expressed in the orientation phase, whereas reflecting on progress is a task addressed in the termination phase.

10. A client who has been diagnosed with major depression works with other mental health consumers to advocate for establishment of additional self-help services for individuals and families with mental illness in a community. This client is likely involved with which organization? a. Community Support Program (CSP) b. National Institute of Mental Health (NIMH) c. Mental Health Study d. National Alliance for the Mentally Ill (NAMI)

ANS: D NAMI was the first consumer group to advocate for better services. This consumer advocacy group worked to establish education and self-help services for individuals and families with mental illness.

17. An example of tertiary prevention concerning mental health is: a. Screening for anxiety b. Depression education c. Counseling following a natural disaster d. Coordination transition from the hospital to the community

ANS: D Tertiary prevention efforts attempt to restore and enhance functioning, thus coordination of transition from the hospital to the community is tertiary prevention. Screening and counseling are secondary prevention activities, and depression education is a primary prevention.

Guidelines followed by the leader of a therapeutic group include focusing on recognizing dysfunctional behavior and thinking patterns, followed by identifying and practicing more adaptive alternate behaviors and thinking. Which theory is evident by this approach? a. Behavioral b. Interpersonal c. Psychodynamic d. Cognitive-behavioral

ANS: D The characteristics described are those of cognitive-behavioral therapy, in which patients learn to reframe dysfunctional thoughts and extinguish maladaptive behaviors. Behavioral therapy focuses solely on changing behavior rather than thoughts, feelings, and behaviors together. Interpersonal theory focuses on interactions and relationships. Psychodynamic groups focus on developing insight to resolve unconscious conflicts.

14. A nurse considers the effects of biology and environment, or nature and nurture, on the development of mental illness when providing care for clients with mental illness. Which nursing model/theory is being applied? a. Community mental health model b. Holistic model c. Systems theory d. Diathesis-stress model

ANS: D The diathesis-stress model integrates the effects of biology and environment on the development of mental illness. Certain genes or genetic combinations produce a predisposition to a disorder. When an environmental stressor challenges an individual with a predisposition to a disorder, the expression of a mental disorder may result.

19. Which role would be most appropriate for an undergraduate-level prepared nurse working with a mental health population? a. Prescribe medications and have hospital admission privileges. b. Work as a case manager for large groups of persons with mental illness. c. Assess clients in acute psychiatric hospital settings. d. Provide basic primary, secondary, and tertiary services.

ANS: D The undergraduate levelprepared nurse is prepared to implement primary, secondary, and tertiary services. Roles of the nurse may include clinician, educator, and coordinator.

A young female member in a therapy group says to an older female member, "You are just like my mother, always trying to control me with your observations and suggestions." Which therapeutic factor of a group is evident by this behavior? a. Instillation of hope b. Existential resolution c. Development of socializing techniques d. Corrective recapitulation of the primary family group

ANS: D The younger patient is demonstrating an emotional attachment to the older patient that mirrors patterns within her own family of origin, a phenomenon called corrective recapitulation of the primary family group. Feedback from the group then helps the member gain insight about this behavior and leads to more effective ways of relating to her family members. Instillation of hope involves conveying optimism and sharing progress. Existential resolution refers to the realization that certain existential experiences such as death are part of life, aiding the adjustment to such realities. Development of socializing techniques involves gaining social skills through the group's feedback and practice within the group.

A nurse interviews a homeless parent with two teenage children. To best assess the family's use of resources, the nurse should ask: a. "Can you describe a problem your family has successfully resolved?" b. "What community agencies have you found helpful in the past?" c. "What aspect of being homeless is most frightening for you?" d. "Do you feel you have adequate resources to survive?"

B

A parent is admitted to a chemical dependency treatment unit. The patient's spouse and adolescent children attend a family session. Which initial assessment question should the nurse ask of family members? a. "What changes are most important to you?" b. "How are feelings expressed in your family?" c. "What types of family education would benefit your family?" d. "Can you identify a long-term goal for improved functioning?"

B

A parent is admitted to a chemical dependency treatment unit. The patient's spouse and adolescent children participate in a family session. What is the most important aspect of this family's assessment? a. Spouse's codependent behaviors b. Interactions among family members c. Patient's reaction to the family's anger d. Children's responses to the family sessions

B

A wife believes her husband is having an affair. Lately, he has been disinterested in romance and working late. The husband has an important, demanding project at work. The mother asks her teen, "What have you noticed about your father?" The teen later mentions this to the father, who says, "Tell your mother that I can't deal with her insecurities right now." Which family dynamic is evident? a. Multigenerational dysfunction b. Triangulation c. Enmeshment d. Blaming

B

An adult diagnosed with schizophrenia lives with elderly parents. The patient was recently hospitalized with acute psychosis. One parent is very anxious, and the other is ill because of the stress. Which nursing diagnosis is most applicable to this scenario? a. Ineffective family coping related to parental role conflict b. Caregiver role strain related to the stress of chronic illness c. Impaired parenting related to patient's repeated hospitalizations d. Interrupted family processes related to relapse of acute psychosis

B

An adult recently diagnosed with AIDS is hospitalized with pneumonia. The patient and family are very anxious. Select the best outcome to add to the plan of care for this family. a. Describe the stages of the anticipatory grieving process. b. Identify and describe effective methods for coping with anxiety. c. Recognize ways dysfunctional communication is expressed in the family. d. Examine previously unexpressed feelings related to the patient's sexuality.

B

Parents of a teenager recently diagnosed with serious mental illness express dismay. One parent says, "Our hopes for our child's future are ruined. We probably won't ever have grandchildren." The nurse will use interventions to assist with: a. denial. b. grieving. c. acting out. d. manipulation.

B

Two divorced people plan to marry. The man has a teenager, and the woman has a toddler. This family will benefit most from: a. role-playing opportunities for conflict resolution regarding discipline. b. guidance about parenting children at two developmental levels. c. formal teaching about problem-solving skills. d. referral to a family therapist.

B

A married couple has two biologic children who live with them as well as a child from the wife's first marriage. What type of family is evident? a. Homogeneous b. Extended c. Blended d. Nuclear

C

A married couple has two children living in the home. Recently, the wife's mother moved in. This family should be assessed as: a. nuclear. b. blended. c. extended. d. alternative.

C

A parent became unemployed 6 months ago. The parent has subsequently been verbally abusive toward the spouse and oldest child. The child ran away twice, and the spouse has become depressed. What is the most appropriate nursing diagnosis for this family? a. Impaired parenting related to verbal abuse of oldest child b. Impaired social interaction related to disruption of family bonds c. Ineffective community coping related to fears about economic stability d. Disabled family coping related to insecurity secondary to loss of family income

D

A parent says, "My son and I argue constantly since he started using drugs. When I talk to him about not using drugs, he tells me to stay out of his business." What is the nurse's first most appropriate action? a. Educate the parent about stages of family development. b. Report the son to law enforcement authorities. c. Refer the son for substance abuse treatment. d. Make a referral for family therapy.

D

The parent of an adolescent diagnosed with mental illness asks the nurse, "Why do you want to do a family assessment? My teenager is the patient, not the rest of us." Select the nurse's best response. a. "Family dysfunction might have caused the mental illness." b. "Family members provide more accurate information than the patient." c. "Family assessment is part of the protocol for care of all patients with mental illness." d. "Every family member's perception of events is different and adds to the total picture."

D

Which example of behavior in a family system demonstrates double-bind communication? a. A mother tells her daughter, "You make me so mad that sometimes I wish I had never had you." b. A teenager tells her father, "You are treating me like a baby when you tell me I must be home by 10 PM on a school night." c. A son tells his mother, "You worry too much about what might happen. Nothing has happened yet, so why worry?" d. A wife tells her husband, "You go ahead with your bowling trip. Try not to worry about me falling on my crutches while I'm alone at home."

D

Which information is the nurse most likely to find when assessing the family of a patient with a serious mental illness? a. The family exhibits many characteristics of dysfunctional families. b. Several family members have serious problems with their physical health. c. Power in the family is maintained in the parental dyad and rarely delegated. d. Stress from living with a mentally ill member has challenged the family's function.

D

Which scenario best demonstrates a healthy family? a. One parent takes care of children. The other parent earns income and maintains the home. b. A family has strict boundaries that require members to address problems within the family. c. A couple requires their adolescent children to attend church services 3 times a week. d. A couple renews their marital relationship after their children become adults.

D

After being identified as high risk for traditional bypass surgery, a patient is scheduled for a minimally invasive direct coronary artery bypass (MIDCAB). The nurse recognizes that which steps are involved in the procedure? A robot is used to replace the mitral valve. Small incisions are made between the ribs. Cardiac catheterization is performed during the procedure. A mechanical stabilizer is placed to immobilize the operative site. A thoracoscope or robotic assistance is used to dissect the internal mammary artery from the chest.

Small incisions are made between the ribs. A mechanical stabilizer is placed to immobilize the operative site. A thoracoscope or robotic assistance is used to dissect the internal mammary artery from the chest. Minimally invasive direct coronary artery bypass (MIDCAB) offers patients with disease of the left anterior descending or right coronary artery an approach to surgical treatment that does not involve a sternotomy and cardiopulmonary bypass (CPB). It involves several small incisions between the ribs to dissect the internal mammary artery (IMA) with a thoracoscope or with robotic assistance. The heart is then slowed or stopped temporarily with adenosine, which is assisted by a mechanical stabilizer to immobilize the operative site. The IMA is then sutured to the coronary artery. A robot is used to replace the mitral valve during robot-assisted cardiothoracic surgery. Transmyocardial laser revascularization involves cardiac catheterization. Mechanical stabilizers are used on a beating heart during off-pump coronary artery bypass. p. 725

An individual is found to consistently wear only a bathrobe and neglect the cleanliness of his apartment. When neighbors ask him to stop his frequent outbursts of operatic arias, he acts outraged and tells them he must sing daily and will not promise to be quieter. This behavior supports what conclusion about this client?

The client is not conforming with social norms


Related study sets

Exam 4: Adrenal Disorders (NCLEX)

View Set

TX Prin. of Real Estate TWO Ch. Two 2.5.2

View Set

Who is this? Blooket Question Set

View Set

US History Chapter 14 Practice Quiz

View Set